================================================================

THEMES ABORDES :

================================================================


[1] PRESENTATION ----------------


  1. 1.1 Quelle est l'histoire de (La)TeX ?
  2. Quels sont les principes de base de LaTeX ?
  3. Comment faire ses premiers pas ?
  4. Comment sont gérées les options de package ?

[2] DOCUMENTATION -----------------


  1. Que puis je lire sur TeX ?
  2. Que puis je lire sur LaTeX2.09 ?
  3. Que puis je lire sur LaTeX2e ?
  4. Que puis je lire sur AMS-LaTeX ?
  5. Que puis-je lire sur la typographie ?
  6. Où trouver une bibliographie complète ?
  7. Que puis-je lire sur Metafont ?

[3] SITES WEB -------------


  1. Où trouver des infos sur le WEB ?
  2. Où trouver cette FAQ ?
  3. Existe t'il des listes de discussion francophones ?
  4. Où trouver d'autres FAQ LaTeX ?

[4] SOURCES TEX ET LATEX ------------------------


  1. Où trouver les sources pour Unix ?
  2. Où trouver les sources pour VMS ?
  3. Où trouver les sources pour DOS, OS/2, Windows3.x/95/NT ?
  4. Où trouver les sources pour MacOS ?
  5. Où trouver d'autres sources ?

[5] EDITER LATEX ----------------


  1. Quel éditeur de texte puis je utiliser ?
  2. Qu'est ce que Lollipop ?

[6] GESTION DE LA MISE EN PAGE ------------------------------


  1. Comment modifier l'interligne d'un document ?
  2. Comment gérer un document recto-verso ?
  3. Comment modifier le style des titres ?
  4. Comment obtenir un document multicolonnes ?
  5. Comment composer une brochure ?
  6. Comment définir un format de document ?
  7. Comment redéfinir les marges d'un document ?
  8. Comment changer l'orientation d'un document ?
  9. Comment justifier verticalement un paragraphe ?
  10. Comment modifier la commande \caption ?
  11. Comment modifier un changement de page ?
  12. Comment obtenir des colonnes parallèles ?
  13. Comment définir l'espace de début de paragraphe ?
  14. Comment supprimer la date sur une page de titre ?
  15. Comment mettre en valeur la première lettre d'un chapitre ?
  16. Comment préserver les tabulations en mode verbatim ?
  17. Comment modifier les entêtes de chapitre ?
  18. Comment définir des tabulations ?
  19. Comment obtenir des lettres accentuées dans tabbing ?
  20. Comment encadrer du texte ?
  21. Comment gérer des URL www ?
  22. Comment mettre en page des exercices dont les solutions sont reportées dans un autre paragraphe ?
  23. Comment positionner un objet dans une page ?
  24. Comment lier le placement des flottants aux sections ?
  25. Comment griser le fond (background) d'un paragraphe ?
  26. Comment modifier l'espace inter-colonnes ?
  27. Comment modifier les environnements de liste ?
  28. Comment souligner plusieurs lignes ?
  29. Comment réaliser des onglets ?
  30. Comment réaliser des QCM ?
  31. Comment modifier l'orientation des flottants ?
  32. Comment faire référence plusieurs fois à la même note de bas de page ?
  33. Comment éviter les orphelins en début ou fin de page ?
  34. Comment définir de nouveaux flottants ?
  35. Comment utiliser la commande \caption hors d'un environnement flottant ?
  36. Comment encadrer un objet flottant ?
  37. Comment changer l'orientation d'un caption ?
  38. Comment mettre en page un programme ?
  39. Comment obtenir un espace insécable ?
  40. Comment insérer une page blanche ?
  41. Comment supprimer l'indentation des paragraphes ?
  42. Comment modifier l'espacement entre caractères ?
  43. Comment supprimer certaines coupures de mots ?
  44. Comment mettre en forme des équations chimiques ?
  45. Comment mettre en page un calendrier ?
  46. Comment forcer un caption sur plusieurs lignes ?
  47. Comment générer des barres de modification dans un document ?
  48. Comment encadrer du texte verbatim ?
  49. Comment écrire du texte en forme de losange ou autre ?
  50. Comment isoler les flottants par un trait horizontal ?
  51. Comment insérer un espace vertical dans une page ?
  52. Comment insérer un espace horizontal dans un texte ?

[7] GESTION DES TABLEAUX ------------------------


  1. Comment gérer les tableaux de plus d'une page ?
  2. Comment modifier l'orientation d'un tableau ?
  3. Comment nommer un tableau ?
  4. Comment modifier l'épaisseur des lignes d'un tableau ?
  5. Comment griser des cellules d'un tableau ?
  6. Comment changer la fonte d'une colonne ?
  7. Comment créer des notes de bas de page dans un tableau ?
  8. Comment écrire un texte sur plusieurs colonnes ?
  9. Comment passer certaines cellules en reverse vidéo ?
  10. Comment fixer la largeur d'une colonne ?
  11. Comment écrire un texte sur plusieurs lignes ?
  12. Comment diviser une cellule par une diagonale ?
  13. Comment définir une colonne en mode mathématique dans un tableau ?
  14. Comment modifier le nombre de tableaux par page ?
  15. Comment aligner des données sur le point décimal ?
  16. Comment obtenir des lignes partielles dans un tableau ?
  17. Comment éviter que du texte de grande taille atteigne le cadre des cellules ?
  18. Comment fixer la largeur d'un tableau ?
  19. Comment tracer des traits discontinus ?

[8] GESTION DES FIGURES -----------------------


  1. Comment inclure une figure ?
  2. Comment placer des figures côte à côte ?
  3. Comment modifier le nombre de figures par page ?
  4. Comment superposer du texte sur des figures ?
  5. Comment réaliser des captures d'écran ?
  6. Comment tracer une courbe ?
  7. Comment est géré le positionnement des figures ?
  8. Comment placer une légende à côté d'une figure ?
  9. Comment insérer des figures dans multicol ?
  10. Comment faire apparaître toutes les figures en fin de document ?
  11. Comment insérer des images Mathematica ?
  12. Comment modifier la taille d'une bounding box ?
  13. Comment obtenir une figure avec un titre non numéroté ?
  14. Comment redéfinir le style de caption ?
  15. Comment fondre une image dans du texte ?
  16. Comment réaliser des diagrammes en bâtons ?
  17. Comment faire un organigramme ?
  18. Comment centrer une figure très large ?

[9] INCLUSION DE FICHIERS -------------------------


  1. Comment inclure des fichiers en mode verbatim ?
  2. Comment gérer un document par parties ?
  3. Comment isoler une partie d'un fichier ps ou dvi ?
  4. Comment inclure un fichier PICT ?

[10] HAUTS ET BAS DE PAGES --------------------------


  1. Comment définir les hauts et bas de page ?
  2. Comment utiliser le mode verbatim dans une note de bas de page ?
  3. Comment mettre les notes de bas de page en fin de document ?
  4. Comment réduire les rappels de titres dans un haut ou bas de page ?
  5. Comment référencer une note de bas de page ?
  6. Comment supprimer la numérotation des pages ?
  7. Comment numéroter les pages par rapport à la dernière ?
  8. Comment supprimer le trait de séparation des notes de bas de page ?
  9. Comment modifier la numérotation des pages ?
  10. Comment supprimer les entêtes et bas de page de pages vierges ?
  11. Comment gérer des en-têtes avec des environnements verbatim multi-pages ?
  12. Comment utiliser \footnote dans un titre ?
  13. Comment placer les notes de bas de page les unes à côté des autres ?
  14. Comment réinitialiser le compteur de note de bas de page à chaque page ?

[11] LE FRANCAIS ET LATEX -------------------------


  1. Comment françiser un document LaTeX ?
  2. Comment corriger les coupures de mots accentués ?
  3. Comment utiliser les lettres accentuées ?

[12] MATHEMATIQUES ------------------


  1. Comment passer en mode mathématique ?
  2. Qu'est que AMS-LaTeX ?
  3. Comment écrire les symboles d'ensembles ?
  4. Comment modifier la numérotation des équations ?
  5. Comment aligner des équations ?
  6. Comment générer des vecteurs ?
  7. Comment écrire du texte en mode mathématique ?
  8. Comment ajuster la taille de délimiteurs ?
  9. Comment changer de fonte en mode mathématique ?
  10. Comment obtenir le L de la transformée de Laplace ?
  11. Comment réaliser un tableau en mode mathématique ?
  12. Comment obtenir d'autres symboles mathématiques ?
  13. Comment définir une fonction ?
  14. Comment définir une matrice ?
  15. Comment encadrer des formules ?
  16. Comment ajuster la longueur d'une flèche par rapport à un texte ?
  17. Comment obtenir des indices ou exposants à gauche ?
  18. Comment tracer des diagrammes commutatifs ?
  19. Comment ajuster la taille de certains opérateurs ?
  20. Comment mettre en page des algorithmes ?
  21. Comment mettre en page des formules longues ?
  22. Comment créer des unités de mesure ?
  23. Comment écrire proprement a/b ?
  24. Comment créer des notes de bas de page dans une formule mathématique ?
  25. Comment forcer le style d'un tableau à displaystyle ?
  26. Comment réduire les espaces gérés par eqnarray ?
  27. Comment réduire la taille des indices ?
  28. Comment aligner des indices ?
  29. Comment mettre en page des théorèmes ?

[13] REFERENCES CROISEES ------------------------


  1. Quelles sont les commandes de base ?
  2. Comment obtenir un renvoi à une page ?
  3. Comment obtenir des références croisées à partir de plusieurs sources ?
  4. Comment définir des liens hypertexte PDF sous LaTeX ?
  5. Comment faire référence à ses propres compteurs ?
  6. Comment faire des références extérieures à un fichier ?

[14] TABLE DES MATIERES -----------------------


  1. Comment générer une table des matières par chapitre ?
  2. Comment ajouter une entrée dans la table des matières ?
  3. Comment changer le titre de la table des matières ?
  4. Comment changer la profondeur de la table des matières ?
  5. Comment gérer des chapitres de préface, d'introduction, et de conclusion non numérotés ?
  6. Comment enlever la numérotation des pages de tdm ?

[15] BIBLIOGRAPHIE SOUS LATEX -----------------------------


  1. Comment construire une bibliographie ?
  2. Comment gérer plusieurs bibliographies ?
  3. Comment changer de langue dans une bibliographie ?
  4. Comment couper une chaîne de caractères ?
  5. Comment renvoyer une référence en note de bas de page ?
  6. Comment faire référence à un document ?
  7. Comment grouper des références multiples ?
  8. Comment changer le titre de la bibliographie ?
  9. Comment changer le style de la bibliographie ?
  10. Comment construire une bibliographie à partir de plusieurs fichiers .bib ?
  11. Comment utiliser la commande \cite dans un \caption ?
  12. Comment référencer une thèse française ou un mémoire ?
  13. Comment supprimer la virgule supplémentaire dans une liste d'auteurs ?
  14. Comment configurer la commande \cite ?
  15. Comment construire une liste d'auteurs ?
  16. Comment spécifier un tri dans une bibliographie ?
  17. Comment référencer les pages contenant des citations ?
  18. Où trouver des styles de bibliographie ?
  19. Comment faire des références croisées ?
  20. Comment citer une URL ?
  21. Comment définir des initiales à deux lettres ?
  22. Comment conserver les majuscules dans les titres ?
  23. Comment changer l'espace entre les item ?

[16] INDEX ----------


  1. Quelles sont les commandes de base ?
  2. Comment construire un index hiérarchique ?
  3. Quels sont les générateurs d'index ?
  4. Comment changer le style de certains mots indexés ?
  5. Comment changer le style des pages de référence ?
  6. Comment rappeler certains mots dans un haut de page ?
  7. Comment générer plusieurs index ?
  8. Qu'est ce que IdXTeX ?
  9. Qu'est ce que xindy ?

[17] GLOSSAIRE --------------


  1. Quelles sont les commandes de base ?
  2. Qu'est ce que GloTeX ?

[18] STYLES PREDEFINIS ----------------------


  1. Où trouver un style de thèse ?
  2. Comment faire son CV en LaTeX ?
  3. Où trouver un format de publication ?
  4. Où trouver un style de manuel de référence ?
  5. Où trouver un style de poster ?
  6. Comment créer son propre style ?

[19] CREATION DE TRANSPARENTS -----------------------------


  1. Quels sont les packages et styles existants ?
  2. Comment définir un contour oval pour des transparents ?
  3. Comment inclure des commentaires dans les transparents ?
  4. Comment modifier l'interligne sous seminar ?
  5. Comment définir des en-têtes et pieds de pages ?
  6. Comment modifier la taille du cadre d'un transparent ?
  7. Comment empêcher les figures de flotter ?
  8. Comment gérer la couleur avec seminar ?
  9. Comment imprimer des transparents en miroir ?

[20] LETTRES ET MAILING -----------------------


  1. Comment structurer une lettre ?
  2. Comment préparer un mailing ?
  3. Comment faire des références dans une lettre ?

[21] SYMBOLES ET LOGOS ----------------------


  1. Où trouver des symboles électroniques ?
  2. Comment dessiner des circuits électroniques ?
  3. Quelles sont les polices de symboles sous LaTeX ?
  4. Comment obtenir les symboles mâle et femelle ?
  5. Comment obtenir le symbole degré ?
  6. Où trouver des symboles astronomiques ?
  7. Où trouver une police de symboles phonétiques ?
  8. Où trouver des opérateurs de logique floue ?
  9. Comment obtenir le symbole de paragraphe ?
  10. Comment obtenir le caractère 'registered' ?
  11. Où trouver le symbole trade-mark ?
  12. Comment obtenir un underscore ?
  13. Où trouver le symbole radioactif ?
  14. Comment obtenir le logo LaTeX ?
  15. Comment obtenir le logo LateX2e ?
  16. Où trouver des chiffres entourés ?

[22] MUSIQUE ------------


  1. Comment écrire de la musique sous LaTeX ?
  2. Comment convertir du midifile en MusicTeX ?
  3. Existe-t'il une liste de discussion de musique ?
  4. Comment éditer un livre de chants ?

[23] CONVERSIONS DE FICHIERS ----------------------------


  1. Comment générer un fichier .ps à partir d'un .dvi ?
  2. Comment transformer du source C++ en LaTeX ?
  3. Qu'est ce que le "Literate Programming" ?
  4. Comment convertir du LaTeX en word ?
  5. Comment convertir du word en LaTeX ?
  6. Comment convertir du scribe en LaTeX ?
  7. Comment convertir du WordPerfect en LaTeX ?
  8. Comment convertir du LaTeX en RTF ?
  9. Comment convertir du RTF en (La)TeX ?
  10. Comment convertir du Excel en LaTeX ?
  11. Comment convertir du HTML en LaTeX ?
  12. Comment convertir du LaTeX en HTML ?
  13. Comment convertir un fichier dvi en ascii ?
  14. Comment convertir du WEB en LaTeX ?
  15. Comment convertir du TeX en Framemaker ?
  16. Comment enlever les balises LaTeX d'un document ?
  17. Comment convertir du SGML en (La)TeX ?
  18. Comment convertir du WinWord en LaTeX ?
  19. Comment convertir un fichier 8 bits en fichier 7 bits ?
  20. Comment convertir un fichier ChiWriter en TeX ?
  21. Où trouver une FAQ de convertisseurs (La)TeX/ Traitement de texte PC ?
  22. Comment convertir une image en police metafont ?
  23. Comment convertir un fichier postscript en ascii ?
  24. Comment convertir un fichier pdf en ascii ?
  25. Comment convertir du LaTeX en PDF ?
  26. Comment définir son propre format de sortie ?

[24] PREVIEWERS ET VIEWERS --------------------------


  1. Où trouver un previewer ?
  2. Où trouver un viewer ?

[25] LOGICIELS DE DESSINS -------------------------


  1. Où trouver un logiciel de dessin ?
  2. Comment inclure des formules Latex dans Xfig ?
  3. Comment gérer différents formats de figures ?

[26] CORRECTEURS ----------------


  1. Où trouver un correcteur orthographique ?
  2. Où trouver un vérificateur de syntaxe LaTeX ?

[27] ASSOCIATIONS -----------------


  1. Qu'est ce que l'association GUTenberg ?
  2. Qu'est ce que l'association TUG ?
  3. Qu'est ce que l'association AsTeX ?

[28] FONTES -----------


  1. Que signifient les sigles T1, mf, fd etc. ?
  2. Quels sont les attributs d'une fonte ?
  3. Comment utiliser une fonte ?
  4. Comment changer la forme d'une fonte ?
  5. Comment changer la taille d'une fonte ?
  6. Comment modifier la fonte des numéros de paragraphe ?
  7. Comment modifier la fonte du mode verbatim ?
  8. Comment réaliser des changements de fontes relatifs ?
  9. Comment mettre en évidence une portion de texte ?

[29] DIVERS -----------


  1. A quoi sert la commande \par ?
  2. Comment commenter une partie d'un source LaTeX ?
  3. Comment utiliser LaTeX sur des petites machines ?
  4. Comment visualiser des paramètres ?
  5. Comment visualiser des compteurs ?
  6. A quoi servent \makeatletter et \makeatother ?
  7. Comment numéroter les lignes d'un document ?
  8. A quoi sert la commande \special ?
  9. Comment réaliser des calculs avec les variables LaTeX ?
  10. Où trouver une fonte 9pt ?
  11. Comment automatiser les compilations LaTeX ?
  12. Comment obtenir des cadres gris ?
  13. Comment obtenir certaines abbréviations ?
  14. Comment gérer les espaces après une macro ?
  15. Où trouver les notices d'utilisation des packages ?
  16. Comment obtenir des listes d'objets flottants ?
  17. Comment connaître les versions des fichiers utilisés lors d'une compilation ?
  18. Pourquoi certaines commandes sont-elles indépendantes ?
  19. Comment installer un package ?
  20. Comment générer des codes barres ?
  21. Comment gérer des abbréviations ?
  22. Comment imprimer un fichier postscript sur une imprimante non postscript ?
  23. Comment surcharger une commande déjà existante ?
  24. Comment reporter l'exécution d'une commande à la fin d'une page ?
  25. Comment définir des scripts interactifs ?
  26. Comment identifier une version provisoire ?
  27. Comment obtenir des caractères barrés ?
  28. A quoi sert la commande \ensuremath ?
  29. A quoi sert la commande \newcommand ?
  30. Où trouver des hirondelles ?
  31. Comment tracer des lignes épaisses ?
  32. Comment imprimer le caractère ~ ?
  33. Comment visualiser tous les caractères d'une fonte ?
  34. A quoi sert % dans les macros ?
  35. Comment inclure l'heure dans un document ?
  36. Comment compter le nombre de mots d'un fichier ?
  37. Comment rendre inactif un caractère ?
  38. Comment utiliser le mode verbatim dans une commande ?
  39. Comment redéfinir la commande \year ?
  40. Qu'est ce que la magnification ?
  41. Comment sont gérés les postscripts dans LaTeX ?
  42. Pourquoi LaTeX n'accepte t'il pas tous les formats d'image ?
  43. A quoi sert la commande \newenvironment ?
  44. Comment récupérer le nom du fichier compilé ?

[30] AJOUTS -----------


  1. Quels sont les ajouts par rapport à la FAQ précédente ?
  2. Quelles sont les modifications effectuées ?

================================================================
[1] PRESENTATION
================================================================

 Quelle est l'histoire de (La)TeX ?
------------------------------------------

TeX (1978) est le formateur de texte de D. E. Knuth. A
l'origine, Knuth a développé TeX (en WEB cf. paragraphe 22.3)
notamment pour réaliser de beaux documents et écrire des
formules mathématiques.

LaTeX, écrit par L. Lamport (1982), est un jeu de macros par
dessus TeX, plus facile à utiliser que ce dernier. LaTeX a été
conçu pour rédiger des articles, des rapports, des thèses ou
des livres ou pour préparer des transparents. On peut insérer
dans le texte, des dessins, des tableaux, des formules
mathématiques et des images sans avoir à se soucier (ou
presque) de leur mise en page. Les documents produits avec
LaTeX et TeX sont d'une excellente qualité typographique.

Plain TeX écrit également par D. E. Knuth, était le premier jeu
(minimal) de macros par dessus TeX. De même Eplain, de K. Berry,
est un jeu de macros intermédiaire entre TeX et LaTeX.

Suite à une large diffusion de LaTeX beaucoup d'extensions ont
été créées par différents utilisateurs. Grâce à Murphy, ces
extensions ont introduit un certain nombre d'incompatibilités
et ont porté atteinte à la portabilité de LaTeX. C'est de cette
situation qu'est né le projet de normalisation LaTeX3, sous la
direction des gurus LaTeX : L. Lamport, F. Mittelbach, C.
Rowley, R. Schopf et tant d'autres... Pour plus de détails,
consulter :
http://www.cogs.susx.ac.uk/users/alanje/latex/latex3.html

Toutefois, pour ne pas perturber les actuels utilisateurs de
LaTeX, la version provisoire normalisée s'appelle LaTeX2e et
elle est compatible (dans la mesure du possible) avec les
anciens standards. Ainsi tous les documents écrits pour
LaTeX2.09 peuvent être compilés sous LaTeX2e en mode "LaTeX2.09
compatibility mode".

Les membres du projet LaTeX3 travaillent actuellement sur le
futur LaTeX3. Autrement dit, LaTeX2e ne devrait plus beaucoup
évoluer. 

Il existe aussi omega, une extension 16 bits de TeX qui utilise
unicode comme représentation interne et autorise ainsi la
composition de textes multi-lingues dans les langues telles que
l'arabe, le chinois ou les langues du continent indien. Pour
plus de renseignements, vous pouvez consulter : les cahiers
GUTenbeg, TUGboat, http://www.ens.fr/omega.html,
ftp://ftp.ens.fr/pub/tex/yannis/omega,
ftp://ftp.ens.fr/pub/tex/yannis/omega-babel, ... 

Signalons également NTS, un projet à très long terme qui vise
d'abord à enrichir TeX mais qui à terme, n'en gardera que les
concepts.


 Quels sont les principes de base de LaTeX ?
------------------------------------------

LaTeX peut être considéré comme un langage de programmation
dans le sens où le document que l'on veut créer doit être
décrit dans un fichier source (.tex) puis doit être compilé.
Ainsi, le compilateur LaTeX prend en entrée un fichier source
écrit en LaTeX et produit en sortie un fichier .dvi (device
independent). Ce fichier peut ensuite être converti en fichier
postscript avant d'être imprimé. Les fichiers dvi et postscript
peuvent être visualisés à l'écran à l'aide de (pre)viewers.

Le passage de LaTeX2.09 à LaTeX2e se traduit dans l'en-tête des
documents par :

\documentclass[options]{class}
\usepackage{style}
\usepackage{package}

au lieu de :

\documentstyle[options, styles, packages]{class}

Attention toutefois, certains styles LaTeX2.09 ne seront pas
reconnus par LaTeX2e.

Le fichier source (fichier.tex) doit comprendre un certain
nombre de commandes (balises) LaTeX qui vont permettre au
compilateur LaTeX de construire un fichier "device independent"
(.dvi). Ces commandes se caractérisent par le fait qu'elles
commencent toutes par un "backslash" (\), que leurs arguments
obligatoires apparaîssent entre accolades ({ et }) et que leurs
arguments optionnels apparaîssent entre crochets ([ et ]). Par
exemple, la structure minimale d'un rapport est en LaTeX2e :

\documentclass{classe}
\begin{document}
   Votre texte...
\end{document}

Les commandes LaTeX décrivent ainsi la classe d'un document, sa
structure, etc. Les principales classes de document disponibles
sont : 'article', 'report', 'book', 'letter' et 'slides'. Il
existe également un certain nombre d'options qui permettent de
modifier le style par défaut d'une classe (le format a4, la
taille de la fonte 12pt, etc). Ces classes et options 
permettent de disposer d'une structure de base pour un
document, mais libre à vous ensuite de définir vos propres
structures grâce aux styles offerts, aux packages disponibles
notamment sur les sites CTAN (cf. paragraphe 3.1) et/ou à vos
propres macros TeX et LaTeX.

L'intérieur d'un document de classe 'article', 'report' ou
'book' est ensuite structuré grâce aux balises disponibles de
type :
\part{}, \chapter{}, \(sub)*section{}, etc. Les lettres et
les transparents font appel à d'autres structures particulières
(cf. sections 18 et 19).

Les informations présentées dans ces structures peuvent être
mises sous différentes formes grâce à des environnements tels
que tabular ou itemize.

Exemple :
\documentclass[12pt]{report}
\usepackage{french}

\title{Mon premier document {\LaTeX} \\ 
	Qu'il est beau ! !}
\author{C'est moi l'auteur.}

\begin{document}

\maketitle
\tableofcontents

\part{Une partie.}
	\chapter{Un chapitre.}
	Texte...
	   \section{Une section.}
	      Texte...
	   \section{Une autre section.}
	      Texte...
	      \subsubsection{Avec une sous-section.}
		 Texte...
	      \subsubsection{Plus une autre.}
		 \begin{table}[htbp] 
		    \begin{center} 
		       \begin{tabular}{|c||c|} 
			  \hline 
			  donn\'ees & donn\'ees \\ 
			  \hline 
		       \end{tabular} 
		       \caption{Titre table. \label{table-}} 
		    \end{center} 
		 \end{table}
\part{Une courte deuxi\`eme partie.}
Texte...
\appendix
\chapter{Et une annexe pour finir.}
Texte...
\begin{itemize} 
   \item bla bla 1
   \item bla bla 2
\end{itemize} 
\end{document}

Les appels à des packages ainsi que les définitions de
nouvelles commandes sont placées dans le préambule du document
LaTeX (i.e. entre les balises \documentstyle (LaTeX2.09) ou
\documentclass (LaTeX2e) et la commande \begin{document}).


 Comment faire ses premiers pas ?
------------------------------------------

En plus de la présentation faite ci-dessus quelques notions
supplémentaires sont utiles à la compréhension de LaTeX.
Celles-ci concernent essentiellement la saisie d'un texte.

Lors que l'on désire travailler dans une langue comportant des
caractères accentués LaTeX propose des saisies un peu barbares
surtout aux yeux des débutants (\'{e} pour é par exemple) mais
qui permettent de conserver la portabilité du document ainsi
saisi sur tous systèmes (caractères codés sur 7 bits). En
revanche l'utilisation de fontes contenant des caractères
accentués (codées sur 8 bits) réduit cette portabilité (pour
plus de détails lire les questions 11.3 et 27.1).

Autre remarque importante du point de vue de la gestion des
espaces et des retours chariot inclus dans le fichier
source (.tex) d'un document. LaTeX gère tout seul les espaces :
il est inutile de taper plusieurs espaces de suite entre deux
mots, ils seront transformés en un seul dans le fichier .dvi.
En outre, UN retour chariot est considéré comme un espace, sauf
s'il est suivi d'un deuxième, il marque alors la fin d'un
paragraphe et le prochain sera indenté. On peut alors sauter
autant de lignes que l'on veut dans le texte, cela n'a aucun
effet supplémentaire. Un passage à la ligne peut être forcé par
\\ ou par \newline mais dans ce cas, la première ligne du
nouveau paragraphe ne sera pas indentée. La commande \par
permet de commencer un nouveau paragraphe en laissant un espace
vertical plus important et en indentant.

En LaTeX, tout ce qui suit un % n'est pas lu, jusqu'au prochain
retour chariot. (Au passage le caractère % peut alors être
obtenu par \%.)


 Comment sont gérées les options de package ?
------------------------------------------

Quand on utilise plusieurs packages : 
\usepackage{package1,package2}
et que l'on veut utiliser une option du package1 qui n'existe
pas pour le package2, on peut écrire :
\usepackage[option1]{package1}
\usepackage{pckge2}
Cela évite un message du type "unknown option1 for pckge2" qui
peut apparaître quand on écrit
\usepackage[option1]{package1,package2}.

On peut également écrire :
\documentclass[option1,gnagna]{article}
\usepackage{pckge1,pckge2}
qui permet de conserver l'ordre de chargement des packages mais
pas celui dans lequel seront exécutées les options par tel
package. Les options de classe sont globales et descendent à
toutes les extensions chargées si elles sont définies pour ces
extensions (certaines options sont par nature globales comme
draft, final, french, a4paper, dvips...).


================================================================
[2] DOCUMENTATION
================================================================


 Que puis je lire sur TeX ?
------------------------------------------

* "The TeXbook", de D. E. Knuth (Addison Wesley, 1984)

* "Le petit livre de TeX", de R. Seroul, (Interéditions, 1989)

* "Introduction to TeX", de N. Schwarz (Addison Wesley, 1989)

* "TeX for the beginner", de W. Snow (Addison Wesley, 1992)

* "TeX for the impatient", de P. Abrahams, K. Berry et K.
Hargreaves (Addison Wesley, 1990)

* "The advanced TeX book", de D. Salomon (Springer Verlag,
1995)

* "La maîtrise de TeX et LaTeX", de T. Lachand-Robert (Masson,
1995). Ce document peut être très utile à ceux qui veulent
programmer/comprendre/adapter des macros/packages LaTeX.

* "A TeX Primer for Scientists", de S. Sawyer et S. Krantz (CRC
Press,1995)

* "TeX by example: A Beginner's Guide", de A. Borde (Academic
Press, 1992).

* "TeX: The Program", de D. E. Knuth (Addison Wesley, 1986)


 Que puis je lire sur LaTeX2.09 ?
------------------------------------------

* "LaTeX, a Document Preparation System", de L. Lamport
(Addison Wesley, 1ère édition)

* "LaTeX reference manual", de L. Lamport (Addison Wesley)

* "Apprends LaTeX", de M. Baudoin (manuel de l'ENSTA)

* "LaTeX, Manuel utilisateur simplifié", de C. Simian (CNRS)

* "La maîtrise de TeX et LaTeX", de T. Lachand-Robert (Masson,
1995)


 Que puis je lire sur LaTeX2e ?
------------------------------------------

* Le fichier usrguide.tex de la distribution LaTeX décrit les
changements entre LaTeX2.09 et LaTeX2e.

* "LaTeX, a Document Preparation System", de L. Lamport
(Addison Wesley, 1994- 2nde édition)

* "The LaTeX companion", de M. Goossens, F. Mittelbach, et A.
Samarin (Addison Wesley, 1994) 

* "A Guide to LaTeX2e, document preparation for beginners and
advanced users", de H. Kopla & P.W. Daly (Addison Wesley, 1995)

Remarque : très bien pour qui cherche un guide très complet. En
	   particulier, il met en permanence l'accent sur ce qui 
	   est différent/spécifique entre LaTeX2e et LaTeX2.09.  

* "LaTeX guide pratique - version 2e", de C. Rolland, (Addison
Wesley, 1995) 

* "Joli manuel pour LaTeX2e", de B. Bayart (GEUT) à l'ESIEE. Ce
manuel est disponible sur CTAN dans /info/JMPL.ps.gz.

* "LaTeX2e, un apercu", de M. Goossens au CERN disponible sur
le WEB a http://www.loria.fr/tex.

* "Essential LaTeX", de J. Warbrick. Ce document très
pédagogique permet de réaliser un document LaTeX en quelques
minutes. Il est disponible sur CTAN (cf. paragraphe 3.1) ou a
http://www.loria.fr/tex/general/essential-latex++.ps

* M. Herrb a traduit en français "The not so short introduction
to LaTeX2e" de T. Oetiker. La version française est disponible
par ftp à ftp.laas.fr dans /pub/Logiciels/latex/flshort/ ou sur
http://www.laas.fr/~matthieu/cours/latex2e/. Elle comprend
quelque compléments français spécifiques.

Il y a là: 
flshort2e.dvi.gz    Le fichier DVI
flshort2e.ps.gz     Le fichier PostScript
flshort2e.tar.gz    Les sources LaTeX2e

* "Objectif LaTeX", de V. Gramet et J.P. Regourd (Masson, 1995)


 Que puis je lire sur AMS-LaTeX ?
------------------------------------------

* "Math into LaTeX:  An Introduction to LaTeX and AMS-LaTeX" de
G. Gratzer (Birkhauser, Boston, 1996).

* Il existe également une documentation "amsldoc.tex"
disponible sur CTAN dans /latex/packages/amslatex/math/.


 Que puis-je lire sur la typographie ?
------------------------------------------

* "Le développpement des caractères", de H. E. Meier, (Syntax
Press, Cham, Suisse).

* "Pour une sémiologie de la typographie", de G. Blanchard,
(édité par Remy Magermans en Belgique, vendu par "Rencontres de
Lure", BP 533 71010 Macon cedex).
Il sagit de la partie "illustrations" de la thèse de G.
Blanchard avec quand même un peu de texte... Le texte complet
n'est disponible qu'en italien: "L'eredita Gutenberg",
(Gianfranco Altieri Editore). Ouvrage fondamental...

* "Manuel de typographie élémentaire", de Y. Perousseaux,
(1995).

* "La chose imprimée", de Dreyfus et Richaudeau, (Retz, 1985).


 Où trouver une bibliographie complète ?
------------------------------------------

http://www.loria.fr/tex/texbib.html propose une bibliographie
tenue à jour.


 Que puis-je lire sur Metafont ?
---------------------------------------
"Metafont", de Knuth D.E. (Addison Wesley Longman).


================================================================
[3] SITES WEB
================================================================


 Où trouver des infos sur le WEB ?
------------------------------------------

* CTAN ou Comprehensive TeX Archive Network
Le système CTAN se propose de rassembler diverses informations
concernant TeX et son environnement suivant une organisation
commune. En particulier, tous les sites dits CTAN ou leurs
miroirs essaient de se synchroniser pour offrir des
informations cohérentes. En voici quelques uns : 

+ France :
par ftp à ftp.loria.fr dans ./pub/ctan/ ou 
sur http://www.loria.fr ou
sur http://www.ens.fr/gut

+ Allemagne :
par ftp à ftp.dante.de dans ./tex-archive/ ou
par mail à mail-server@ftp.dante.de  ou
sur http://www.dante.de

+ Grande-Bretagne
par ftp à ftp.tex.ac.uk dans ./tex-archive/ ou 
par mail à texserver@tex.ac.uk  ou
sur http://www.tex.ac.uk/UKTUG/home.html

+ Pays-Bas :
sur http://www.ntg.nl/ntg/ntg.html

+ Espagne :
sur http://gordo.us.es/Actividades/GUTH

+ Etats-Unis :
par ftp à ftp.cdrom.com dans /pub/tex/ctan/ ou 
sur http://www.cdrom.com/pub/tex/ctan/

Une liste complète des sites CTAN peut être trouvée sur CTAN 
dans /usergrps/info/usergrps.tex ou encore à
http://jasper.ora.com/CTANINFO/ctan.sites.html.

ATTENTION (08/96) : Le serveur d'archives (La)TeX aux Etats-Unis,
	ftp.shsu.edu, n'étant plus à jour par défaut de
	maintenance, il est fortement déconseillé désormais d'y
	récupérer des utilitaires (La)TeX. Il reste donc les 2
	serveurs CTAN << de base >>, l'anglais et l'allemand,
	ainsi que leurs fidèles copies en France dont celle du
	LORIA-CNRS à Nancy : ftp.loria.fr. 

La plupart des sites CTAN offrent une fonctionnalité QUOTE
SITE INDEX qui permet d'accéder plus rapidement à l'information
que l'on recherche sous forme d'expression régulière. En
france, le mieux est de consulter :
http://www.loria.fr/cgi-bin/ctan-index

* http://jasper.ora.com/ctan.html est une interface WWW plus
pratique pour accéder aux sites CTAN.

* http://www.loria.fr/tex/ présente le (La)TeX Navigator. On
peut y trouver de nombreux pointeurs (sur des docs dont cette
FAQ, des personnes, des packages, des outils LaTeX, des sites
ftp, CTAN, ...) et de nombreuses informations concernant Tex,
LaTeX, LaTeX2e, LaTeX3, AMS-LaTex, BibTeX, SliTeX, ....

* ftp : ftp.univ-rennes1.fr et
http://www.univ-rennes1.fr/ dans ./pub/GUTenberg proposent
également de nombreuses informations dont la dernière version
de OzTex pour Macintosh, la distribution GUTenberg pour PC
(DOS-GUT) ainsi que les distributions Unix basées sur MlTeX et
TeX et VMS. Ce site présente également un certain nombre
d'archives et de publications dont notamment les lettres et les
cahiers GUTenberg.

* Le TeX macro index de D. M. Jones disponible par ftp
anonyme à theory.lcs.mit.edu dans ./pub/tex/TeX-index
recense un certain nombre de macros et styles LaTeX
actuellement existants.

* Autres sources d'informations (résumés de commandes, guides) :
+ http://www.sd.monash.edu.au/~timm/pub/guides/lshort2e.dvi
présente un guide rapide.
+ http://molscat.giss.nasa.gov/LaTeX/ présente un sommaire des
commandes LaTeX
+ http://www.tug.org/interest.html donne des pointeurs sur TeX,
LaTeX et consorts.
+ Sur CTAN dans /info/latex2e-help-texinfo.

* Un catalogue des outils TeX et LaTeX, mis à jour régulièrement
est disponible sur :
http://www.dit.csiro.au/~gjw/texpkgs.html

Ce catalogue présente une liste des packages disponibles avec
une courte description de chacun. Il est mirroré par les sites
CTAN dans /help/Catalogue/. Une version gzip de ce catalogue
est disponible à :
http://www.cdrom.com/pub/tex/ctan/help/Catalogue/
catalogue.html.gz

Exemples :
+ US CTAN :
http://www.cdrom.com/pub/tex/ctan/help/Catalogue/catalogue.html
+ UK CTAN node at:
http://www.tex.ac.uk/tex-archive/help/Catalogue/catalogue.html
+ germany
ftp://ftp.dante.de/tex-archive/help/Catalogue/catalogue.html

* http://www-h.eng.cam.ac.uk/help/tpl/textprocessing/
LaTeX_intro.html présente une introduction à LaTeX2e et aux
mathématiques.


 Où trouver cette FAQ ?
------------------------------------------

* Cette FAQ sera mensuellement postée complète dans
fr.comp.text.tex, et sa table des matières dans comp.text.tex.

* Une version postscript de cette FAQ est disponible sur le 
LaTeX Navigator : http://www.loria.fr/tex/divers.html.

* Elle est disponible sur CTAN dans
/help/LaTeX-FAQ-francaise/.

* Elle est également accessible sur le WEB : 
+ http://www.loria.fr/tex/divers.html (LaTeX Navigator)
+ http://www.ams.org/tex/ (American Mathematical Society's TeX
Resources pages) 
+ par ftp à ftp.inria.fr dans /faq/fr.comp.text.tex/
(ce site possède entre autres un miroir avec le site
rtfm.mit.edu)
+ par ftp à ftp.univ-lyon1.fr dans
/pub/faq/by-name/fr/faq-latex-francaise 
+ http://diwww.epfl.ch/~jmonzani/FAQ_LaTeX.html site de l'Ecole
Polytechnique Federale de Lausanne.


 Existe t'il des listes de discussion francophones ?
------------------------------------------

* Il existe la liste gut@ens.fr (de l'association GUTenberg)
dont les archives sont stokées sur :
http://www.univ-rennes1.fr/LISTES/gut@ens.fr/arc/maillist.html
et sur lesquelles on peut faire des recherches par auteurs,
sujets, etc.

* Concernant le projet omega il y a la liste omega@ens.fr.


 Où trouver d'autres FAQs LaTeX ?
------------------------------------------

* http://www.cogs.susx.ac.uk/cgi-bin/texfaq2html
* ftp://rtfm.mit.edu/pub/usenet-by-hierarchy/comp/text/tex/
* sur CTAN :
usergrps/uktug/faq
usergrps/dante/de-tex-faq
help/comp-fonts-FAQ/
* Le cahier GUTenberg numéro 23


================================================================
[4] SOURCES TEX ET LATEX 
================================================================


 Où trouver les sources pour Unix ?
------------------------------------------

* La distribution GUTenberg est disponible par ftp à
ftp.univ-rennes1.fr dans /pub/GUTenberg/UNIX/. Elle est
basée sur MlTeX (adapté à LaTeX2e), TeX, Metafont. Les binaires
sont disponibles pour : 
+ Sun4 sous solaris 2.x et SunOS 4.1.x
+ IBM RS6000 sous AIX 3 et 4
+ Silicon graphics sous irix4, 5 et 6
+ HP7xx sous Hpux9 et 10
+ HP9000 en Hpux 10
+ DECalpha sous OSF/1
+ DECstation 3100 sous Ultrix
+ PC sous Linux
+ PC i86 en Solaris 2

* Différentes distributions source Unix de TeX sont disponibles
sur CTAN dans /systems/unix/.

* La distribution teTeX pour Unix, Linux ou Irix est disponible
sur CTAN dans /systems/unix/teTeX/(distrib/binaries/). A
l'origine, cette distribution a été developpée pour Linux. Il
existe également un package 'config' qui offre des fichiers de
configuration de cette distribution disponibles sur CTAN dans
/systems/unix/teTeX/contrib/. 


 Où trouver les sources pour VMS ?
------------------------------------------

* TeX pour VMS est disponible sur CTAN dans
/systems/vms/ puis Alpha/ ou VAX/ suivant l'architecture
utilisée.

* La distribution OpenVMS VAX et AXP de GUTenberg est disponible
par ftp à ftp.univ-rennes1.fr dans /pub/GUTenberg/vms/. Elle
s'appuie sur TeX et LaTeX2e.


 Où trouver les sources pour DOS, OS/2, Windows3.x/95/NT ?
------------------------------------------

* Une distribution TeX pour PC, incluant LaTeX, BibTeX,
previewers, et drivers est disponible par ftp anonyme à
vax.eedsp.gatech.edu dans ./pub/TeX/. 

* emTeX de E. Mattes, pour PC sous MS-DOS, Windows ou OS/2, est
disponible par ftp anonyme sur CTAN dans /systems/msdos/emtex.
Cette distribution inclu LaTeX, METAFONT, BibTeX, TeXcad... Il
existe également emtexgi sur CTAN dans
systems/msdos/emtex-contrib/emtexgi/ qui est une interface
Windows pour emTeX. Il existe une liste de discussion emTeX :
majordomo@physik.tu-berlin.de.

* Une version emTeX françisée de M. Lavaud
(Michel.Lavaud@univ-orleans.fr), distribuée par AsTeX (cf.
question 27.3) est disponible par ftp à ftp.univ-orleans.fr dans
/astex/ ou sur CTAN dans /pub/tex/PC/AsTeX/. L'installation de
la nouvelle version 2.2 est entièrement automatisée, ainsi que
la configuration des pilotes (dviscr, dviwin, dvips), GSview,
Gnuplot et les interfaces TeXShell, MicroEmacs et MenuTeX. Il y
a une fonction d'installation / desinstallation sélective avec
une vingtaine de types d'installation différents pour installer
tout ou partie de la distribution, et plusieurs modes
d'installation (ajouter, écraser, mettre à jour, désinstaller,
module par module).

Il existe même une liste de discussion AsTeX :
astex@univ-orleans.fr. Pour s'inscrire, il faut envoyer le
message HELP à listserv@univ-orleans.fr.

* 4AllTeX, très complet, nécessite 4DOS (un remplacement de
command.com) pour l'utilisation de ses .btm (.bat améliorés).
4AllTeX inclut une interface (TeXelmExtel) sous windows (Tex,
LaTeX2e, BibTeX, makeindex, ...) vous trouverez le pack complet
sur ftp.loria.fr dans /systems/.

* gTeX sous MS-DOS et windows est disponible sur CTAN dans
/systems/msdos/gtex/. Son avantage est de fonctionner en 32bits
à la fois sous Windows (3.x, 95 et NT) et sous Dos via un
extender fourni. Cette ditribution comprend micro-emacs,
dviwin, etc.

* Win32 MiKTeX de C. Schenk, pour windows 95 et NT, est
une version compilée de LaTeX2e (il supporte même les longs
noms de fichiers). MiKTeX est disponible par ftp à
ftp.tex.ac.uk dans /systems/win32/miktex/ou sur CTAN dans
/systems/win32/miktex/.

* DOS-GUT (distribution françisée par GUTenberg) est disponible
par ftp à ftp.univ-rennes1.fr dans /pub/GUTenberg/PC/DOS-GUT/.
Cette distribution est basée sur gtex et web2c. Elle comprend
l'éditeur TeXshell, TeX, TeX-XeT, plain, LaTeX, babel,
ArabicTeX, dviscr, dvips, metafont et GhostScript.

Remarque : la distribution DOS-GUT n'est plus activement
	   développée. Elle a été remplacée pas WIN-GUT.

* WIN-GUT est une version intégrée pour windows 95 et 3.1 par
P. Legrand. Elle permet la composition de textes français (soit
avec babel, soit avec french), américain et arabe (ArabicTeX).
Elle comprend DVIWIN, dvips, micro-emacs

* Il existe également Y&Y TeX System pour Windows. Pour plus
d'informations, vous pouvez consulter le site
http://www.YandY.com/.

* PCTeX pour DOS, Windows3.1 et Windows95/NT, qui est moins
puissant  que le précédent mais qui est très simple à mettre en
place. Pour plus de détails, consulter http://www.pctex.com.

* EMTEXGI de A. Cottrell est une nouvelle distribution
disponible à
http://www.wfu.edu/Academic-departments/Economics/ftp/
emtexgi.html.


 Où trouver les sources pour MacOS ?
------------------------------------------

* CMacTeX, shareware de de T. Kiffe, comprenant TeX, tex--xet,
bibtex, makein metafont, metapost, un previewer dvi, dvips,
ps2pdf, un driver PostScript et quelques utilitaires de gestion
de fontes est disponible sur CTAN dans /systems/mac/cmactex/.
Pour plus de renseignements, consulter :
http://www.math.tamu.edu/~tkiffe/cmactex.html.

* OzTex de A. Trevorrow, est disponible par ftp à
ftp.univ-rennes1.fr dans ./pub/gut/, sur CTAN dans
/systems/mac/oztex/ ou par ftp à midway.uchicago.edu dans
./pub/OzTeX/. Cette distribution shareware intègre un previewer
dvi et un driver postscript. Elle nécessite plus de 512K de
mémoire vive.

* Direct-TeX Pro 2.1 de W. Ricken, est une distribution
shareware qui possède un environnement intégré et multi-fenêtre
très pratique, modulable de surcroit. Elle intègre TeX--XeT
3.14159 ainsi que Metafont 2.718, et tous les utilitaires qui
permettent de passer en une passe d'un source (La)TeX au .ps
final. Il y a même un éditeur intégré, quoique petit.
Direct-TeX est disponible sur CTAN dans
/systems/mac/directtex/. Elle nécessite le système 7 et 8Mo de
mémoire vive. Une version françisée par GUTenberg est en
préparation.

* Euro-Oztex de Y. Haralambous, est la distribution
proposée par GUTenberg. Cette version françisée appelle une
contribution shareware à Trevorrow et Ricken. Elle est
disponible par ftp à ftp.univ-rennes1.fr dans
/pub/GUTenberg/mac/Euro-OzTeX/.

Remarque : Euro-Oztex est une vieille version de la distribution
	   de GUTenberg. Elle utilisait OzTeX 1.7. Elle a été
	   remplacé récemment par Mac-GUT, qui elle utilise
	   CMacTeX.

* Mac-GUT, basée sur CMacTeX, est la distribution shareware
proposée par GUTenberg (donc françisée). Elle  n'est disponible
à l'heure actuelle que sur le cd-rom TeX-Live distribué par
GUTenberg à ses adhérents. Pour plus de détails, consulter :
http://www.tug.org/texlive.html.


 Où trouver d'autres sources ?
------------------------------------------

* Atari 
TeX pour Atari ST est disponible par ftp à
atari.archive.umich.edu dans ./atari/tex/ ou à
ifi.informatik.uni-stuttgart.de dans ./pub/atari.st/tex/ ou sur
CTAN dans /systems/atari/. Pour tous renseignements contacter
atari@atari.archive.umich.edu par un mail "help".

* Amiga
+ PasTeX, implémentation de TeX 3.1 et METAFONT 2.7 sont
disponibles par ftp anonyme à merlin.etsu.edu dans 
/ab20/AMIGA/ ou à forwiss.uni-passau.de dans
/pub/amiga/tex ou sur CTAN dans /systems/amiga/.

+ On peut trouver également PasTeX 1.4 sur les mirroirs FTP
aminet (sunsite.cnam.fr. ftp.grolier.fr, ftp.netnet.net, ...)
dans le répertoire /pub/aminet/text/tex/. PasTeX 1.4 est le
portage de LaTeX2e. La distribution comprend également dvips.

+ XFig est disponible dans /pub/aminet/gfx/edit/.

+ Ghostscript est disponible dans /pub/aminet/gfx/show/.

+ Une version LaTeX2e a également été développée à l'ESIEE
http://www.esiee.fr/~tex/Install/Amiga/index.html

* Tandy 6000 
Pour tous renseignements contacter Ken Yap
(ken@syd.dit.csiro.au).

* TOPS-20
Une distribution TeX sur TOPS-20 est disponible par ftp
anonyme à ftp.math.utah.edu dans ./pub/tex/pub/web.


================================================================
[5] EDITER LATEX
================================================================


 Quel éditeur de texte puis-je utiliser ?
------------------------------------------

* (X-)Emacs est un éditeur sous Unix qui offre en standard un
mode d'édition, un peu fruste mais néanmoins pratique,
facilitant la composition de documents (La)TeX. Une extension à
emacs, AUC-TeX (disponible sur CTAN dans /support/auctex/ ou à
http://sunsite.auc.dk/auctex/), fournit de nombreuses facilités
supplémentaires (indentation automatique, messages d'erreur en
anglais compréhensible, gestion des documents multi-fichiers,
etc.)

Emacs reconnait automatiquement certaines extensions
(tex,sty...) dans un nom de fichier, et active le mode en
question automatiquement. Si votre fichier n'est pas reconnu
comme un document (La)TeX, vous pouvez spécifier sur la première
ligne de votre fichier : % -*-latex-*-

Les packages 'font-lock' et 'hilit19' ('hilit319' pour Xemacs),
permettant de choisir les couleurs et les polices mettant en
évidence la syntaxe d'un fichier, sont utilisables avec les
modes (La)TeX.

Le package (standard) 'imenu' donne accès à un menu listant les
en-têtes de section du document, et permet de retrouver
celles-ci facilement dans un grand document. Une extension à ce
mécanisme permet de mieux visualiser la structure du document,
en indentant les sous-sections. La dernière version diffusée est
disponible à l'URL : 
http://fillmore.univ-mlv.fr/~dirson/files/ydi-imenu.el

* GNU emacs et AUCTeX peuvent également être utilisés sous
MS-DOS ou OS/2.

* JED est un clone multi-plateformes proche d'emacs. JED est en
fait un emacs allégé qui offre des facilités dans l'édition de
fichier (La)TeX. Il tourne sous Unix/VMS/Dos/Win. Il est
disponible à http://space.mit.edu/~davis.

* Xcoral-2.5, éditeur sous Unix offre des fonctionnalités de
même type que emacs. Il est disponible par ftp à ftp.inria.fr
dans /X/contrib-R5/clients/ ou à ftp.x.org dans /contrib/. 

Xcoral est un éditeur multi-fenêtres pour X Window System,
offrant un certain  nombre de facilités notamment pour écrire
des programmes C, C++, et des documents Latex (2.09) ou Html.

Cet éditeur comprend un interprêteur Ansi C 'built-in' qui
permet aux utilisateurs d'étendre ses fonctionnalités
facilement. Un manuel d'aide complet indexé est disponible
on-line.

* Alpha est un éditeur Macintosh shareware assez proche d'emacs.
Il est disponible entre autres par ftp à 
ftp://www.cs.umd.edu/pub/faculty/keleher/Alpha/. 

Cet éditeur est hautement configurable grâce à un langage de
programmation intégré, Tcl. Il possède entre autres un mode
LaTeX très convivial. Alpha permet aussi une interaction avec le
compilateur (OzTeX ou Direct-TeX) en lancant la compilation
d'une combinaison de touches. La dernière version d'Alpha est la
6.5. Elle comprend la version 3.2 des macros freeware Alpha
LaTeX de T. Scavo. 

Pour plus de renseignements, voir : 
+ http://www.cs.umd.edu/~keleher/alpha.html 
+ http://www.cs.umd.edu/~keleher/latex_guide/latex_guide.html

* MicroEmacs (inclu dans la distribution DOS-GUT), sous windows,
permet d'éditer et de gérer des documents TeX.

* Eddi4TeX, sous MS-DOS ou OS/2, est un éditeur spécifiquement
conçu pour TeX, il offre la couleur, vérifie la syntaxe. Il
est disponible sur CTAN dans /systems/msdos/e4t/ ou
/systems/OS2/epmtex/.

* Scientific Word pour windows est un éditeur qui permet presque
de visualiser un document LaTeX en WYSIWYG (What You See Is What
You Get). Ce produit est commercial.

* LyX est un traitement de texte sous X11 qui offre une sortie
LaTeX2e. Il est presque WYSIWYG. LyX présente les avantages
d'être petit, rapide et gratuit. LyX est encore en
développement. Une version NON définitive est disponible à
http://www-pu.informatik.uni-tuebingen.de/users/ettrich ou
par ftp à sunsite.unc.edu dans /pub/Linux/X11/xapps/editors ou
par ftp à ftp.via.ecp.fr dans /pub/lyx/ ou
WWW http://www.lehigh.edu/~dlj0/LyriX.html ou
WWW http://www-pu.informatik.uni-tuebingen.de/~ettrich.
Il existe également une liste de discussion : lyx@via.ecp.fr.

Parmi les fonctionnalites, on trouve:
- éditeur d'équations,
- éditeur de tables,
- inclusion d'images au format EPS,
- correction d'orthographe,
- etc.

* TeXShell, écrit par J. Schlegelmilch, est disponible sur CTAN
dans /systems/msdos/texshell/ts271.zip. C'est un éditeur Windows
sous dos qui offre une coloration des mots clés LaTeX, une aide
en ligne et d'autres petites fonctionnalités telles que les
compilations associées à des boutons. La distribution DOS-GUT
utilise TeXshell et offre un MicroEmacs françisé.

Il existe également TeXShell pour X Window system (Tcl/Tk)
disponible par ftp à sunsite.unc.edu dans
/pub/Linux/apps/tex/tsYYMMDD.tgz.

* WINEDT95,logiciel Shareware pour Windows 95, est un éditeur
avec menu TeX/LaTeX qui permet de repérer les commandes LaTeX et
de compter les délimiteurs. Il est disponible sur CTAN dans
/support/winedt/.

* wintex95 disponible sur CTAN dans /systems/win32/wtex95/ est
un éditeur flexible offrant un coloriage automatique de la
syntaxe LaTeX.

* Cicero est un traitement de texte sous X11R6 et Motif2.0. Le
package comprend des fontes X postscript, ghostscript, TeX
(dvips) et Cicero. Pour plus de renseignements, consulter :
http://zeus.informatik.uni-frankfurt.de/~weis/cicero.html

* X-Window Shell pour TeX (OpenLook ou Xaw/Xaw3d) disponible
sur
http://www.tm.bi.ruhr-uni-bochum.de/personal/marc/xtexshell.html.

* xtem: une interface graphique offrant des fonctionnalités
TeX/LaTeX disponible sur
http://ftp.lrw.uni-bremen.de/xtem/xtem_texmenu.html.

* STEAD "Sympathetic Tk-based Editor for Average Dummies" sous
Unix (ressemble à Alpha sur mac) est disponible par ftp à
ftp.ensta.fr dans /pub/tcl/contrib_ensta/. La particularité de
cet éditeur est qu'il est lui même écrit en texte pur ! En
effet, il est entièrement écrit en langage TCL et TK (son
extension permettant d'écrire par script une application avec
bouton, menu, ... sous Xwindows). Une conséquence est que cet
éditeur nécessite la présence de l'interpréteur TCL (appelé wish
et disponible sur ftp.ibp.fr fichier
/pub/tcl/distrib/tk3.6.tar.gz (il y a tout ce qu'il faut avec :
la doc, des exemples, ...) Le répertoire contient également le
projet du livre sur TCL... 

STEAD est un éditeur simple d'utilisation, contextuel et
configurable.

Autre avantage : une fois l'interpréteur disponible, l'éditeur
tourne aussi bien sur SUN3 (pas vite !), SUN4, HP, et même sur
les PC avec UNIX BSD.

* Funtek, de V. Vidal, sous X-Windows system et Motif, est un
éditeur texte orienté latex, il traite le source latex page à
page, permet un accès aux symboles spéciaux, et une construction
de tableaux  automatique. La version actuelle est une béta.
Pour plus de détails consulter :
http://fillmore.univ-mlv.fr/~vidalc.

* D'autres éditeurs sous dos, Mac et windows95 sont présentés
dans http://www.jumbo.com/pages/utilities/dos/editor
http://www.jumbo.com/pages/utilities/mac/util/text
http://www.jumbo.com/pages/dtp/windows95/editor


 Qu'est ce que Lollipop ?
------------------------------------------

C'est un jeu de macro de V. Eijkhout, destiné à faciliter
l'écriture de macros TeX. Il est disponible sur CTAN dans
/macros/lollipop/.


================================================================
[6] GESTION DE LA MISE EN PAGE
================================================================


 Comment modifier l'interligne d'un document ?
------------------------------------------

* Pour modifier l'espace interligne d'un document on peut
utiliser la commande \linespread (solution non recommandée).
Par exemple, \linespread{1.6} permet de doubler l'intervalle
par défaut.

* \renewcommand{\baselinestretch}{1.2} placé dans le
préambule permet d'obtenir le même résultat pour tout le
document.

* Il existe également les packages 'doublespace' (pour
LaTeX2.09) et 'setspace' (pour LaTeX2e) disponibles sur CTAN
dans /macros/latex2.09/contrib/misc/ pour l'un et 
/macros/latex/contrib/other/misc/ou
/macros/latex/contrib/supported/setspace/ pour l'autre.
setspace.sty définit les environnements singlespace,
onehalfspace et doublespace. L'utilisation de ces styles est
recommandée parce que plus robuste (gestion des tableaux, des
notes de bas de page, ...).

* Pour réduire l'interligne d'un paragraphe (à celui de small
par exemple) sans modifier la taille de la fonte on peut
utiliser \small{\normalsize texte à interligne réduit}\par ou
encore {\advance\baselineskip -1pt le texte \par}


 Comment gérer un document recto-verso ?
------------------------------------------

* En LaTeX 2.09 il faut passer twoside comme option au
documentstyle. Ensuite, pour forcer les entêtes de chapitre à
commencer sur une page impaire, il faut inclure la commande
\cleardoublepage avant chaque début de chapitre.

Exemple :
\documentstyle[twoside]{report}
puis :
\cleardoublepage 
\chapter{Introduction.}

* LaTeX2e prévoit directement les options de classe twoside et
openright.

Exemple : 
\documentclass[twoside,openright]{report}


 Comment modifier le style des titres ?
------------------------------------------

Les définitions de \section, \sub(sub)section, etc, se
trouvent dans report.cls, article.cls, book.cls.

ATTENTION : Il est vivement conseillé de ne pas modifier
            directement ces classes mais de redéfinir 
	    un fichier.sty avec les nouvelles commandes ou
	    d'utiliser \makeatletter et \makeatother.

Exemple :
La syntaxe de définition d'une nouvelle section est :
\renewcommand\section{\@startsection {section}{1}{\z@}%
	{-3.5ex \@plus -1ex \@minus -.2ex}%
	{2.3ex \@plus.2ex}%
	{\reset@font\Large\bfseries}}

Explication :
+ La commande \@startsection permet de gérer : la table des
	matières, la numérotation des titres, les références, 
	les titres des sections dans l'en-tête, etc...  
+ {section} indique qu'il s'agit d'une section 
+ {1} indique son niveau dans la table des matières 
+ {\z@} indique son niveau d'indentation (zero) 
+ {-3.5ex \@plus -1ex \@minus -.2ex} définit l'espace qui sera
	ajouté au dessus du titre 
+ {2.3ex \@plus.2ex} définit l'espace qui sera ajouté en dessous
	du titre. Si ce nombre est négatif alors il s'agit d'un
	espacement horizontal, pour avoir des titres "en ligne",
	comme ça :
	
	Titre de ma section.        Nous allons parler... bla,
	bla, bla...
+ \@plus et \@minus permettent de jouer sur l'élasticité de ces
	espaces
+ {\reset@font\Large\bfseries} sont les commandes de mises en
	forme du titre. 	

Pour augmenter l'espace avant une section il suffit, par
exemple, d'écrire :

\makeatletter
\renewcommand\section{\@startsection{section}{1}{\z@}% 
	{2cm \@plus -1ex \@minus -.2ex}%
	{2.3ex \@plus.2ex}%
	{\reset@font\Large\bfseries}}
\makeatother

dans le préambule du document.


 Comment obtenir un document multicolonnes ?
------------------------------------------

* L'option standard twocolumn permet de présenter le texte sur
deux colonnes verticales.

* Pour agir localement, on peut utiliser la commande :
\twocolumn[texte sur une colonne]{texte sur deux colonnes}

* Le package 'multicol', disponible sur CTAN dans
/macros/latex/packages/tools/, définit l'environnement multicols
qui permet de redéfinir localement le nombre de colonnes
désirées (10 maximum). Lorsqu'une page n'est pas complète, le
texte apparaît réparti sur toutes les colonnes.

Exemple :
\begin{multicols}{3}
   blabla
\end{multicols}

Pour ajouter un titre sur plusieurs colonnes, il faut utiliser 
l'option [\section{Titre.}] juste après \begin{multicols}{3}.

Pour qu'une ligne de séparation apparaîsse entre les colonnes,
il faut utiliser \setlength{\columnseprule}{1pt}. 

Pour redéfinir la largeur de l'espace inter-colonnes, il faut
utiliser \setlength{\columnsep}{30pt}.


 Comment composer une brochure ?
------------------------------------------

* Pour redéfinir un format de page, (par exemple un A4 plié en
trois) il faut utiliser la commande \setlength.  Il suffit de
savoir quelles sont les longueurs à préciser, le meilleur moyen
pour ce faire est de les visualiser avec la commande \layout
(definie par le package 'layout' disponible sur CTAN dans
/macros/latex/packages/tools/).

Exemple (Th. Bouche) :
\ProvidesPackage{a6size}
% rien a voir avec la taille : ajustement du \baselineskip
\renewcommand\normalsize{%
   \@setfontsize\normalsize\@xiipt{13.5}%
   \abovedisplayskip 12\p@ \@plus3\p@ \@minus7\p@
   \abovedisplayshortskip \z@ \@plus3\p@
   \belowdisplayshortskip 6.5\p@ \@plus3.5\p@ \@minus3\p@
   \belowdisplayskip \abovedisplayskip
   \let\@listi\@listI}
\renewcommand\small{%
   \@setfontsize\small\@xipt{12.4}%
   \abovedisplayskip 11\p@ \@plus3\p@ \@minus6\p@
   \abovedisplayshortskip \z@ \@plus3\p@
   \belowdisplayshortskip 6.5\p@ \@plus3.5\p@ \@minus3\p@
   \def\@listi{\leftmargin\leftmargini
   \topsep 9\p@ \@plus3\p@ \@minus5\p@
   \parsep 4.5\p@ \@plus2\p@ \@minus\p@
   \itemsep \parsep}%
   \belowdisplayskip \abovedisplayskip}
\normalsize

\setlength\paperheight {148mm}%
\setlength\paperwidth  {105mm}%
%\voffset-1cm
%\hoffset-2cm
\setlength{\topmargin}{-1.3cm}%
\setlength{\oddsidemargin}{-.5cm}%
\setlength{\evensidemargin}{-1cm}%
\setlength{\marginparsep}{0\p@}%
\setlength{\headsep}{0\p@}%
% calcule la hauteur du texte en fonction du \baselineskip, pour 
% que les lignes soit placees au meme niveau sur toutes les pages
\setlength{\textheight}{\topskip}
\addtolength{\textheight}{22\baselineskip}%
\setlength{\textwidth}{7cm}%
\setlength{\footskip}{23\p@}% (originellement : 48)
%\setlength{\baselineskip}{13\p@}%
%\setlength{\marginparwidth}{0\p@} %
%\addtolength{\baselineskip}{.2\baselineskip}%
\setlength{\parindent}{0\p@}
%\addtolength{\headsep}{\headsep}
\setlength{\push@skip}{.2\textwidth}
\newenvironment{page}{\vspace*{\stretch{1}}}
{\vspace*{\stretch{2.5}}\newpage}
\pagestyle{plain}

Il faut ensuite opérer ce que les imprimeurs appellent une
imposition : imprimer la page tant à tel endroit de la x-ième
feuille de telle sorte qu'il n'y ait plus qu'à plier la liasse
pour obtenir un livre prêt à être relié. Cette étape est facile
à réaliser soit à l'aide de dvidvi (mais qui n'autorise pas les
rotations, ce qui peut en limiter l'intérêt pour des formats
spéciaux) soit avec pstops :
pstops "2:0L@.7(21cm,0)+1L@.7(21cm,14.85cm)" un.ps deux.ps
disponible sur CTAN dans /support/psutils/.

ATTENTION : ce genre de manipulation n'est pas recommandé.

* L'option a5paper de LaTeX2e permet également de faire cela.
Il faut ensuite utiliser dvidvi et dvips en -t landscape. dvips
est disponible entre autres par ftp à ftp.univ-rennes1.fr dans
/pub/GUTenberg/sources/.

* Le package 'a5booklet' est disponible sur CTAN dans
/pub/tex/dviware/a5booklet/. 

* Consulter également le package '2up' pour LaTeX2e.

* Le résultat est possible également en ne travaillant que sur
le postscript : dvips -h twoup -t landscape.

* Sur PC on peut utiliser dvidrv.

* Pour PC twoup fait cela, mais ce n'est pas du domaine public.


 Comment définir un format de document ?
------------------------------------------

ATTENTION : ce genre de manipulation n'est pas recommandé.

* \setlength{\textwidth}{??cm} permet de fixer la largeur du
texte 

* \setlength{\textheight} {??cm} permet de fixer la hauteur du
texte  

* \setlength{\oddsidemargin} {(-)??cm} permet de définir la
marge des pages impaires   

* \setlength{\evensidemargin} {(-)??cm} permet de définir la
marge des pages paires 

* \setlength{\topskip} {??cm} laisse un espace en haut de page

* \setlength{\footskip} {??cm} laisse un espace en bas de page

* \setlength{\headheight} {??cm} fixe la hauteur de l'entête

* \addtolength{\topmargin}{(-)??cm}

* \addtolength{\textheight}{(-)??cm}

* Il existe également les packages 'a4' et 'a4wide' disponibles
sur CTAN respectivement dans
/macros/latex/contrib/supported/ntgclass/ et dans
/macros/latex/contrib/other/misc/ qui permettent de redéfinir
les marges et largeur de texte.

* Le package 'geometry' est plus flexible que les précédents
pour définir entièrement son format de page.


 Comment redéfinir les marges d'un document ?
------------------------------------------

ATTENTION : ce genre de manipulation n'est pas recommandé.

* Le package 'vmargin', de V. Kuhlmann, est disponible sur CTAN
dans /macros/latex/contrib/other/misc/. Il  permet facilement de
redéfinir globalement (pour tout le document) les marges d'un
document par la commande : 
\setmarginsrb{1}{2}{3}{4}{5}{6}{7}{8} 
1 est la marge gauche 
2 est la marge en haut 
3 est la marge droite
4 est la marge en bas
5 fixe la hauteur de l'entête
6 fixe la distance entre l'entête et le texte
7 fixe la hauteur du pied de page 
8 fixe la distance entre le texte et le pied de page

* L'environnement changemargin décrit ci-dessous permet de
modifier localement les marges d'un document. Il prend deux
arguments, la marge gauche et la marge droite (ces arguments
peuvent prendre des valeurs négatives).

\newenvironment{changemargin}[2]{\begin{list}{}{%
\setlength{\topsep}{0pt}%
\setlength{\leftmargin}{0pt}%
\setlength{\rightmargin}{0pt}%
\setlength{\listparindent}{\parindent}%
\setlength{\itemindent}{\parindent}%
\setlength{\parsep}{0pt plus 1pt}%
\addtolength{\leftmargin}{#1}%
\addtolength{\rightmargin}{#2}%
}\item }{\end{list}}

Exemple :
\begin{changemargin}{2cm}{-1cm}
   texte...
\end{changemargin}
permet d'augmenter la marge gauche de 2cm et de dimimuer
celle de droite de 1cm.

* Le package 'geometry' permet de redéfinir les marges d'un
document ou de définir le layout de la page.

Exemple :
\geometry{margin=5pt}
équivalent à 
\geometry{hmargin=5pt, vmargin=5pt}
équivalent à 
\geometry{margin={5pt,5pt}}

Autre exemple :
\documentclass{article}
\usepackage{geometry}
\geometry{scale=1.0, nohead}



 Comment changer l'orientation d'un document ?
------------------------------------------

* Globalement, pour passer en orientation portrait, il suffit de
mettre l'option landscape dans \documentstyle (LaTeX2.09) ou
dans \documentclass (LaTeX2e).

Exemple :
\documentclass[landscape]{report}

* Le package 'lscape' de D. Carlisle (LaTeX2e), disponible sur
CTAN dans /macros/latex/packages/graphics/, permet de changer
localement d'orientation portrait vers paysage et vice versa. Il
définit l'environnement landscape.

* Il existe également le package 'rotating' disponible sur CTAN
dans /macros/latex/contrib/supported/rotating/.

* Il existe aussi le programme docstrip disponible par ftp à
ftp.cdrom.com dans /macros/latex/unpacked/docstrip.tex. 


 Comment justifier verticalement un paragraphe ?
------------------------------------------

L'environnement vcenterpage ci-dessous permet de centrer
verticalement un texte sur une page seule :

\newenvironment{vcenterpage}
{\newpage\vspace*{\fill}}
{\vspace*{\fill}\par\pagebreak}

Exemple :
\begin{vcenterpage}
   texte ...
\end{vcenterpage}


 Comment modifier la commande \caption ?
------------------------------------------

* Le package 'hangcaption', disponible sur CTAN dans
/macros/latex209/contrib/misc/, dans lequel la commande
\isucaption remplace la commande \caption, permet de modifier la
mise en page de cette dernière. En particulier, il permet de
définir \captionwidth.

* Il existe également les packages 'caption' et 'caption2'
disponibles sur CTAN dans
/macros/latex/contrib/supported/caption/ qui permettent de
modifier la commande \caption classique (largeur, style,
fonte, ...).

Note : il est recommandé d'utiliser 'caption2' plutôt que
       'caption'.

* Le package 'topcapt' disponible sur CTAN dans
/macros/latex/contrib/misc/ définit la commande \topcaption qui
permet de placer le texte de la commande caption au dessus du
flottant auquel elle est rattachée.

* Le package 'french' de B. Gaulle propose la commande
\unnumberedcaptions qui permet de supprimer la numérotation des
flottants.


 Comment obtenir des colonnes parallèles ?
------------------------------------------

Le package 'parallel' disponible sur CTAN dans
/macros/latex/contrib/supported/parallel/ permet d'obtenir deux
colonnes dont l'une peut contenir la traduction de l'autre.


 Comment définir l'espace de début de paragraphe ?
------------------------------------------

Il faut valoriser la variable \parindent :
\setlength{\parindent}{1cm}


 Comment supprimer la date sur une page de titre ?
------------------------------------------

Il faut ajouter la commande \date{} dans le préambule du
document.


 Comment mettre en valeur la première lettre d'un
------------------------------------------

chapitre ?
----------
* Il faut utiliser le package 'dropcaps' de F. Lauwers.
Il est disponible sur CTAN dans
/macros/latex209/contrib/dropcaps/. Ce package est utilisable
avec LaTeX2.09 et LaTeX2e.

* Les packages 'initial' et 'initials' pour LaTeX2e sont
disponibles sur CTAN. Le second que l'on peut trouver dans
/fonts/gothic/yinit/ fait appel à des fontes yinit particulières.

* Il existe également un package 'drop' pour LaTeX2.09, mais
compatible LaTeX2e, disponible sur CTAN dans
/macros/latex209/contrib/misc/.

Exemple :
\documentclass[12pt,a4paper]{article}
\usepackage{drop}
\font\largefont=yinitas % fontes yinit
\begin{document}
\drop{D}OES THERE EXIST a field with 4 elements?
\end{document}

* 'bigdrop' accessible sur CTAN dans/CTAN/digests/ttn est une
macro TeX compatible LaTeX.

* Le package 'bigstart' pour LaTeX2.09 et LaTeX2e permet
également de faire cela.

* Il existe également le package 'picinpar' pour LaTeX2.09
disponible sur CTAN dans /macros/latex209/contrib/picinpar/ et
dans /systems/msdos/4alltex/disk04/.

* On peut également definir sa propre macro :
\font\capfont=cmbx12 at 24.87 pt % or yinit, or...?
\newbox\capbox \newcount\capl \def\a{A}
\def\docappar{\medbreak\noindent\setbox\capbox\hbox{%
\capfont\a\hskip0.15em}\hangindent=\wd\capbox%
\capl=\ht\capbox\divide\capl by\baselineskip\advance\capl by1%
\hangafter=-\capl%
\hbox{\vbox to8pt{\hbox to0pt{\hss\box\capbox}\vss}}}
\def\cappar{\afterassignment\docappar\noexpand\let\a }

Et utiliser :
\cappar Once upon a time ...

* Un petit dernier, le package 'dropping' qui étend le
package 'dropcaps' est disponible sur CTAN dans
/macros/latex/contrib/other/dropping/.


 Comment préserver les tabulations en mode verbatim ?
------------------------------------------

* Le package 'moreverb' est disponible sur CTAN dans
/macros/latex/contrib/other/misc/. Il propose notamment un
environnement verbatimtab qui permet de conserver des
tabulations.

* Il existe également le package 'verbasef' (verbatim automatic
segmentation of external files) disponible sur CTAN. Il utilise
l'environnement figure.

* On peut également inclure les lignes suivantes dans le
préambule du document :
\makeatletter
{\catcode`\^^I=\active
\gdef\verbatim{\catcode`\^^I=\active\def^^I{\hspace*{4em}}%
\@verbatim \frenchspacing\@vobeyspaces \@xverbatim}}
\makeatother


 Comment modifier les entêtes de chapitre ?
------------------------------------------

Il faut modifier la macro makechapterhead.

Exemple, ajouter dans le preambule :
\makeatletter
\def\@makechapterhead#1{%
  \vspace*{50\p@}%
  {\parindent \z@ \raggedright \normalfont
    \interlinepenalty\@M
    \ifnum \c@secnumdepth >\m@ne
        \Huge\bfseries \thechapter\quad
    \fi
    \Huge \bfseries #1\par\nobreak
    \vskip 40\p@
  }}

\def\@makeschapterhead#1{%
  \vspace*{50\p@}%
  {\parindent \z@ \raggedright
    \normalfont
    \interlinepenalty\@M
    \Huge \bfseries  #1\par\nobreak
    \vskip 40\p@
  }}
\makeatother


 Comment définir des tabulations ?
------------------------------------------

Il faut utiliser l'environnement tabbing qui permet de placer
des marques d'alignement dans un texte.

Exemple :
\begin{tabbing}
   Voici \= des \= marques \= de tabulation \\
   \> la je m'aligne sur la premiere \\
   \> \> \> la sur la troisieme \\
   \hspace{3cm} \= \hspace \= \kill
   un \> autre \> exemple.
\end{tabbing}


 Comment obtenir des lettres accentuées dans tabbing ?
------------------------------------------

* Pour produire un \'{e} dans un environnement tabbing ou la
commande \' a été redéfinie, il faut utiliser \a'{e} ou \a'e

* Le style suivant de Jean-Pierre Drucbert du CERT/ONERA GPI
permet de remplacer l'environnement standard tabbing par
l'environnement Tabbing dans lequel les commandes \` \' \> \<
\= \+ \- sont remplacées par \TAB` \TAB' \TAB> \TAB< \TAB=
\TAB+ \TAB- ce qui permet de préserver les commandes d'accent
(\' \` ou \=).

\ProvidesPackage{Tabbing}[1996/01/16]
\NeedsTeXFormat{LaTeX2e}[1995/12/01]
\gdef\Tabbing{\lineskip \z@skip
%     \let\>\@rtab
%     \let\<\@ltab
%     \let\=\@settab
%     \let\+\@tabplus
%     \let\-\@tabminus
%     \let\`\@tabrj
%     \let\'\@tablab
\def\TAB##1{\ifx ##1>\@rtab\else
            \ifx ##1<\@ltab\else
            \ifx ##1=\@settab\else
            \ifx ##1+\@tabplus\else
            \ifx ##1-\@tabminus\else
            \ifx ##1`\@tabrj\else
            \ifx ##1'\@tablab\else
                         \PackageError{Tabbing}%
                         {Bad argument ##1 for Tabbing
			 specification} \fi\fi\fi\fi\fi\fi\fi}
     \let\\=\@tabcr
     \global\@hightab\@firsttab
     \global\@nxttabmar\@firsttab
     \dimen\@firsttab\@totalleftmargin
     \global\@tabpush\z@ \global\@rjfieldfalse
     \trivlist \item\relax
     \if@minipage\else\vskip\parskip\fi
     \setbox\@tabfbox\hbox{\rlap{\indent\hskip\@totalleftmargin
       \the\everypar}}\def\@itemfudge{\box\@tabfbox}%
     \@startline\ignorespaces}
\gdef\endTabbing{%
  \@stopline\ifnum\@tabpush >\z@ \@badpoptabs \fi\endtrivlist}
\endinput


 Comment encadrer du texte ?
------------------------------------------

* Une solution consiste à définir un tableau d'une seule
cellule.

* On peut également utiliser :
\fbox{
\minipage{\columnwidth}
	Texte...
\end{minipage}
}

* Il existe également le package 'fancybox', disponible sur CTAN
dans /macros/latex/contrib/others/seminar/inputs/, qui définit
des fonctions telles que \shadowbox, \doublebox, \ovalbox,...

* Le package 'boxedminipage' est un vieux style LaTeX2.09
disponible sur CTAN dans /macros/latex209/contrib/misc/.

* Sous LaTeX2e, on peut utiliser l'environnement lrbox.


 Comment gérer des URL WWW ?
------------------------------------------

* Il existe le package 'url' disponible sur CTAN dans
/macros/latex/contrib/other/misc/ qui permet de gérer les
coupures des URL WWW un peu longues. Il est également capable de
gérer les adresses e-mail, les liens hypertexte, les noms de
directories. En outre, cet outil est paramètrable.

* La macro \discretionary permet également de dire comment
couper une chaîne de caractères :
\discretionary{Avant la coupure}{après}{s'il n'y en a pas} 

Remarque : \- est défini comme \discretionary {-}{}{}


 Comment mettre en page des exercices dont les solutions sont reportées dans un autre paragraphe ?
------------------------------------------

L'objectif ici est de pouvoir saisir dans le fichier source les
textes des exercices suivis de leurs solutions, mais qu'au
niveau de la mise en page du document, les solutions
apparaîssent groupées dans un autre paragraphe/chapitre.

* Le package 'sverb', de M. Wooding (csuov@csv.warwick.ac.uk),
disponible sur CTAN offre l'environnement exercise qui permet de
réaliser ce type de mise en page. Les exercices sont numérotés
suivant les numéros de chapitre ; la présence de la réponse est
facultative, et l'ensemble des réponses peut être appelé par la
commande \theanswers. Il est possible d'utiliser cette commande
plusieurs fois, n'importe ou dans le document (on peut ainsi
regrouper les solutions par chapitre ou globalement dans une
annexe).

Exemple :
\begin{exercise}
   L'intitul\'e de l'exercice...
   \answer
   et la r\'eponse
\end{exercise}

* Le package 'answers', de M. Piff, disponible sur CTAN dans 
macros/latex/contrib/supported/answers/ permet également de
réaliser ce genre d'exercice. Il permet entre autres :
+ d'avoir plusieurs types de solutions (ex: réponse numérique
seule ou détail),
+ d'inclure les solutions (l'une, l'autre ou les deux dans
l'exemple ci-dessus) dans le texte (après une marque spécifique
si désiré),
+ mettre les solutions à la fin,
+ ne pas mettre les solutions.

* On peut également trouver le package 'exams' sur CTAN dans
/macros/latex/contrib/supported/.


 Comment positionner un objet dans une page ?
------------------------------------------

* Pour pouvoir positionner un objet aux coordonnées x,y par
rapport au coin supérieur gauche d'une page, il suffit
d'utiliser le package 'atxy' disponible par ftp à
ftp.univ-orleans.fr dans le module l209misc.zip du répertoire
/pub/tex/PC/AsTeX/Paq_Base/.

Exemple :
\atxy(3cm,2.5cm) {toto adresse toto}

Remarque : petit défaut, si le document ne contient que des
	   commandes \atxy il n'y a pas de dvi généré. Il faut 
	   donc ajouter n'importe quoi au début du document, ~
	   par exemple, pour que ça marche.


 Comment lier le placement des flottants aux sections ?
------------------------------------------

Pour lier la position des éléments flottants aux limites
de sections, D. Arseneau a développé le package 'placeins'
disponible sur CTAN dans
/macros/latex/contrib/other/misc. Ce package définit la
commande \FloatBarrier qui force le placement des flottants
avant son appel.


 Comment griser le fond (background) d'un paragraphe ?
------------------------------------------

* Le package 'psboxit', disponible sur CTAN dans
/macros/latex2.09/contrib/misc/ et en particulier
l'environnement "boxitpara" permet de faire cela.

* Le package 'shadbox' disponible sur CTAN dans
/macros/latex/contrib/other/shadbox permet de griser toute
boîte, texte, figure, ....

* Le package 'shading' disponible sur CTAN dans
/macros/latex209/contrib/shading/ permet de griser un
paragraphe.

* De même, le package 'shade', de P. Schmitt, est disponible sur
CTAN dans /macros/generic/

* On peut également utiliser le package 'color'.

Exemple :
\colorbox[gray]{0.5}{some words}


 Comment modifier l'espace inter-colonnes ?
------------------------------------------

Il faut modifier la variable \columnsep.

Exemple :
\addtolength{\columnsep}{5mm}


 Comment modifier les environnements de liste ?
------------------------------------------

* Les environnements de liste utilisent les paramètres suivants :
\topsep espace vertical supplémentaire (ajoute à \parskip)
	inséré entre le texte précédent la liste et le 1er objet 
	de la liste
	
\partosep espace vertical supplémentaire inséré devant la liste
	si celle-ci est précédée d'une ligne blanche

\itemsep espace vertical supplémentaire (ajouté à \parsep)
	inséré entre les éléments d'une liste.

qui peuvent être ajustés à la main (commande \setlength).

ATTENTION : cette solution est peu recommandée.

* Le petit bout de code ci-dessous, de M. Boyer
(mboyer@robot.ireq.ca), définit les commandes : 
+ \noitemsep pour supprimer tout espacement vertical entre les
items des environnements \itemize, \enumerate et \description. 
+ \doitemsep pour les remettre.
Pour l'utiliser, il suffit de le sauvegarder dans un fichier
.sty et de l'inclure dans son document par une commande
\usepackage.

%% ------------------------------------------

%% Copyright (c) 1993 Hydro-Quebec mboyer@robot.ireq.ca
%% ------------------------------------------


%% Bring items closer together in
list environments % Prevent infinite loops
\let\orig@Itemize =\itemize	    
\let\orig@Enumerate =\enumerate
\let\orig@Description =\description
% Zero the vertical spacing parameters
\def\Nospacing{\itemsep=0pt\topsep=0pt\partopsep=0pt%
\parskip=0pt\parsep=0pt}
% Redefinition de art12.sty pour commencer a la marge de gauche
%\leftmargini 1.2em      % 2.5em

\def\noitemsep{
% Redefine the environments in terms of the original values
\renewenvironment{itemize}{\orig@Itemize\Nospacing}{\endlist}
\renewenvironment{enumerate}{\orig@Enumerate\Nospacing}{\endlist}
\renewenvironment{description}{\orig@Description\Nospacing}%
{\endlist}
}

\def\doitemsep{
% Redefine the environments to the original values
\renewenvironment{itemize}{\orig@Itemize}{\endlist}
\renewenvironment{enumerate}{\orig@Enumerate}{\endlist}
\renewenvironment{description}{\orig@Description}{\endlist}
}

* La macro ci-dessous de T. Murphy permet de remplacer les
numéros de l'environnement enumerate par des caractères grecs :
\makeatletter
\def\greek#1{\expandafter\@greek\csname c@#1\endcsname}
\def\@greek#1{\ifcase#1\or$\alpha$\or$\beta$\fi}% as many as you
% need 
\renewcommand{\theenumi}{\greek{enumi}}
\makeatother

* Les définitions suivantes :
\renewcommand{\labelitemi}{\textbullet}
\renewcommand{\labelitemii}{---}
\renewcommand{\labelitemiii}{votre-label-pour-le-niveau-iii}
\renewcommand{\labelitemiv}{votre-label-pour-le-niveau-iv}
permettent de redéfinir les caractères utilisés par
l'environnement itemize pour ces différents niveaux
d'encapsulation.

De même, utilisez \descriptionlabel pour changer le style des
étiquettes de l'environment description.

Exemple :
\renewcommand\descriptionlabel[1]{\hspace\labelsep\normalfont%
\itshape #1:}
produit des étiquettes en italique, avec deux points.

* Pour réduire globalement l'espace entre les items d'une liste
on peut également utiliser le bout de code suivant de M. Wooding
\makeatletter
\toks@\expandafter{\@listI}
\edef\@listI{\the\toks@\setlength{\parsep}{1pt}}
\makeatother

* Le package 'mdwlist', de M. Wooding, disponible sur CTAN dans
/macros/latex/contrib/supported/mdwtools/ permet de redéfinir
certains paramètres de mise en page des listes qui ne sont pas
facile d'accès sous LaTeX.

* Plus généralement, l'environnement list permet de définir son
propre style de liste. Sa syntaxe est la suivante :
\begin{list}{label}{mep}\end{list}
+ l'argument label permet de définir le symbole qui sera
associé à chaque élément de la liste. 
+ mep permet de définir la mise en page des éléments de la liste

Exemple :
\newenvironment{maliste}%
{ \begin{list}%
	{$\bullet$}%
	{\setlength{\labelwidth}{30pt}%
	 \setlength{\leftmargin}{35pt}%
	 \setlength{\itemsep}{\parsep}}}%    
{ \end{list} }


 Comment souligner plusieurs lignes ?
------------------------------------------

Pour souligner un texte qui comprend des retours à la ligne, il
faut utiliser le package 'ulem' disponible sur CTAN dans
/macros/latex/contrib/other/misc/. Ce package est paramètrable.

Exemple :
\usepackage[normalem]{ulem}

Remarque : \underline ne permet pas de gérer les retours à la
	   ligne du fait qu'il encapsule le texte passé en
	   argument dans une boîte.


 Comment réaliser des onglets ?
------------------------------------------

* Pour insérer un carré noir, décalé vers le bas à chaque
nouveau chapitre, le long de la marge des pages de droite d'un
document, on peut utiliser le package 'fancyhdr' ou
'fancyheadings'. Ces packages sont disponibles sur CTAN
respectivement dans /macros/latex/contrib/supported/fancyhdr/ et
/macros/latex/contrib/other/.

Note : On note toutefois qu'il est peu probable qu'une imprimante
       puisse accéder au ras de la marge. La solution consiste 
       alors à définir un format de document plus petit et
       utiliser le massicot. Attention dans ce cas lors de la
       définition des marges.

* On peut également utiliser le package 'onglet' définit
ci-dessous par B. Bayart. Celui-ci nécessite le package
'everyshi' disponible sur CTAN dans
/macros/latex/contrib/supported/everyshi/.

\ProvidesPackage{onglet}[1996/07/25 B. Bayart]
\RequirePackage{everyshi}

\newcounter{maxchapter}
\newcounter{tmpa}
\newlength{\basehauteur}
\setlength{\basehauteur}{1cm}
\newlength{\ajoutdroite}
\newlength{\htcclv}
\def\concatener{%
  \setlength{\ajoutdroite}{\textheight}
  \divide\ajoutdroite by \basehauteur
  \setcounter{maxchapter}{\number\ajoutdroite}
  \setcounter{tmpa}{\value{chapter}}
  \addtocounter{tmpa}{-1}
  \divide\value{tmpa} by\value{maxchapter}
  \multiply\value{tmpa} by\value{maxchapter}
  \advance\value{tmpa} by -\value{chapter}
  \addtocounter{tmpa}{-1}
  \setlength{\ajoutdroite}{\paperwidth}
  \setlength{\htcclv}{\ht255}
  \addtolength{\ajoutdroite}{-\wd255}
  \addtolength{\ajoutdroite}{-1in}
  \addtolength{\ajoutdroite}{-1.5cm}
  \setbox255=\vbox{\hbox to \wd255{%
    \box255%\relax
    \rlap{\vbox to \htcclv{%
      \vskip-\value{tmpa}\basehauteur
      \hbox{%
        \hskip\ajoutdroite\relax
        \usebox{\laboite}%
      }%
      \vfill
    }}%
  \hfill}}%
}
\newsavebox{\laboite}
\def\faireboite{\sbox{\laboite}%
{\hbox to 1.5cm{\let\protect\relax
\huge\thechapter\hfill\vrule height 1em depth 0pt width 5mm}}}

\AtBeginDocument{\EveryShipout{\faireboite\concatener}}
\endinput


 Comment réaliser des QCM ?
------------------------------------------

Il existe le package 'exams' disponible sur CTAN dans
/macros/latex/contrib/supported/. Ce package permet entre autres
de paramétrer les questions de manière à ce que les propositions
apparaîssent dans un ordre aléatoire.


 Comment modifier l'orientation des flottants ?
------------------------------------------

* Le package 'rotating' disponible sur CTAN dans
/macros/latex/contrib/supported/rotating/ offre deux
environnements sidewaysfigure pour les figures et sidewaystable
pour les tableaux.

* On peut également utiliser l'environnement sideways du
package 'rotating'.

Exemple :
\begin{figure}
  \begin{sideways}
  ...
  \end{sideways}
\end{figure}


 Comment faire référence plusieurs fois à la même note de bas de page ?
------------------------------------------

* Il faut utiliser les commandes \footnotemark[] et
\footnotetext[]{}. \footnotemark permet de gérer le compteur de
notes et \footnotetext permet d'insérer le texte correspondant
en bas de page.

Exemple :
bla blabla\footnotemark[1] bla bla bla blablabla\footnotemark[2]
bla bla bla bla blabla\footnotemark[1] bla.

\footnotetext[1]{double bla}
\footnotetext[2]{triple bla}

* Avec french, on peut utiliser la commande refmark.

Exemple :
le vrai appel\footnote{C'est une note en bas de
page\label{manote}} et le deuxieme appel\refmark{manote}

Note : Si vous n'utilisez pas french, cette macro refmark est
       décrite dans le cahier Gutenberg numero 15, avril 1993, 
       page 52. Ce cahier est accessible a l'URL:
       http://www.univ-rennes1.fr/pub/GUTenberg/publications


 Comment éviter les orphelins en début ou fin de page ?
------------------------------------------

* Il suffit d'ajouter les lignes :
\widowpenalty=10000
\clubpenalty=10000
\raggedbottom
dans le préambule du document.

* Pour agir localement, on peut également utiliser la commande
\enlargethispage.


 Comment définir de nouveaux flottants ?
------------------------------------------

Le package 'float' disponible sur CTAN dans
/macros/latex/contrib/supported/float/ permet de définir de
nouveaux objets flottants. Il définit notamment des
environnements permettant d'encadrer des objets ou de les
séparer du reste du texte par des lignes horizontales.


 Comment utiliser la commande \caption hors d'un environnement flottant ?
------------------------------------------

* Pour pouvoir attacher un titre de style table à un
environnement non flottant (i.e. autre que figure ou table), il
faut définir : \makeatletter
\def\@captype{table}
\makeatother

* Autre possibilité pour obtenir un titre de style table
n'importe où :
\documentclass{article}
\makeatletter
\def\captionof#1#2{{\def\@captype{#1}#2}}
\makeatother
\begin{document}
some text
\captionof{table}{\caption{a caption goes here}\label{foo}}
\end{document}


 Comment encadrer un objet flottant ?
------------------------------------------

* Il faut utiliser le package 'float' disponible sur CTAN dans
/macros/latex/contrib/supported/float/.

* Si le titre accompagnant le flottant peut être à l'extérieur
du cadre désiré, on peut alors utiliser \fbox dans la
déclaration du flottant.

Exemple :
\begin{table}
    \fbox{
    table à encadrer
    }
\end{table} 


 Comment changer l'orientation d'un caption ?
------------------------------------------

La commande \rotcaption fournie avec le package 'rotating'
permet de changer l'orientation d'un caption.

Exemple :
\begin{figure}
   \centering
   \begin{minipage}[c]{1in}
      \includegraphics[angle=90,width=\textwidth]{coco.ps}
   \end{minipage}
   \begin{minipage} 
      \rotcaption{Ma jolie figure coco}
      \label{coco}
   \end{minipage}
\end{figure}


 Comment mettre en page un programme ?
------------------------------------------

Les packages 'program' (disponible sur CTAN dans
/macros/latex/contrib/supported/program/) et 'programs'
permettent de mettre en reliefs des mots clés, d'utiliser des
mathématiques dans des algorithmes, etc.


 Comment obtenir un espace insécable ?
------------------------------------------

Le caractère ~ est interprété par LaTeX et permet d'obtenir un
espace insécable.

Exemple :
Comme le montre la figure~\ref{nom-fig}, ...


 Comment insérer une page blanche ?
------------------------------------------

Pour forcer latex à laisser une page blanche dans un document,
il faut utiliser successivement les trois commandes :
\newpage
\strut
\newpage


 Comment supprimer l'indentation des paragraphes ?
------------------------------------------

* \noindent au début du texte permet de ne pas indenter un
paragraphe.

* Pour systématiquement supprimer l'indentation du premier
paragraphe d'une section par exemple, il faut redéfinir la
commande \section. Il faut que le 4ème paramètre de
\@startsection soit une distance négative, pour qu'il n'y ait
pas de retrait au premier paragraphe du texte qui suit :

\makeatletter
\renewcommand\section{\@startsection {section}{1}{\z@}%
                           {-3.5ex \@plus -1ex \@minus -.2ex}%
%%%%%%                      ^^^^ (4eme parametre)
                           {2.3ex \@plus.2ex}%
                           {\normalfont\Large\bfseries}}
\makeatother


 Comment modifier l'espacement entre caractères ?
------------------------------------------

* Le package 'letterspace' permet de modifier l'espacement entre
mots ou caractères.

* Le package 'trackin' disponible sur CTAN dans
/macros/latex/contrib/other/tracking/ permet de jouer sur
les espacements dans les mots ou les phrases pour les ajuster
dans une longueur spécifiée.


 Comment supprimer certaines coupures de mots ?
------------------------------------------

* Pour agir sur un mot particulier, il faut utiliser la commande
\hyphenation.

Exemple :
Visualisation des coupures :
+\showhyphens{mousehole AlGaAs GaAs GaInP AlInP}

  Underfull \hbox (badness 10000) detected at line 0
  [] \OT1/cmr/m/n/10 mouse-hole Al-GaAs GaAs GaInP Al-InP

Interdiction de certaines coupures :
+\hyphenation{AlGaAs GaAs GaInP AlInP}
+\showhyphens{mousehole AlGaAs GaAs GaInP AlInP}

  Underfull \hbox (badness 10000) detected at line 0
  [] \OT1/cmr/m/n/10 mouse-hole AlGaAs GaAs GaInP AlInP

* Moins propre mais tout aussi efficace, on peut inclure le mot
à ne pas couper dans une hbox.

Exemple :
\hbox{CeMotTresLongNestPasCoupe}

* Pour empêcher LaTeX de couper les mots dans un paragraphe il
suffit de l'encadrer par les commandes \begin{sloppypar} et
\end{sloppypar}.

* Pour empêcher LaTeX de couper les mots dans tout un document,
il faut utiliser la commande \sloppy dans le préambule du
document.

* De manière globale, on peut aussi déclarer :
\DeclareFontFamily{T1}{cmr}{\hyphenchar\font=-1}

* Pour interdire la coupure de tous les mots commençant par une
majuscule, il faut utiliser : \uchyph=0.

* On peut également interdire la coupure des mots d'une langue
particulière dans un document multilingues en utilisant l'astuce
suivante : Il suffit de créer un fichier de motifs de césures
vide, par exemple pour le russe
%%% ruhyph.tex %%%
\patterns{}
\endinput
%%%%%%%%%%%%%%%%%%%
et dans le fichier language.dat ajouter la ligne
russian ruhyph.tex


 Comment mettre en forme des équations chimiques ?
------------------------------------------

* Le package 'pchtex' disponible sur CTAN dans
/cros/context/ppchtex/ permet d'écrire des formules chimiques.

* Sur Mac ou PC, il existe MDL qui est une version freeware
d'ISIS Draw qi permet de créer ces propres structures et de les
sauver sous format eps. Pour plus de détails, consulter
http://www.mdli.com/prod/ioffer.html.

* Il existe le package 'chemsym' disponible sur CTAN dans
/macros/latex/contrib/other/chemsym/.

* Le package 'xymtex' disponible sur CTAN dans
/macros/latex209/contrib/xymtex/ permet de définir des
structures chimiques.


 Comment mettre en page un calendrier ?
------------------------------------------

Le package 'termcal' disponible sur CTAN dans
/macros/latex/contrib/supported/termcal/ permet de mettre en
page un calendrier. Il permet de préciser quels jours doivent
apparaitre et d'insérer du texte soit régulièrement à certaines
dates soit à des dates particulières.


 Comment forcer un caption sur plusieurs lignes ?
------------------------------------------

Pour forcer un retour à la ligne dans un caption, il faut
utiliser \caption[text1]{text2 \\\hspace{\linewidth} text3}.

Note : text1 est le texte qui apparaitra dans la liste des
       figures ou des tableaux.


 Comment générer des barres de modification dans un document ?
------------------------------------------

Il suffit d'utiliser le package 'changebar' disponible sur CTAN
dans /macros/latex/contrib/supported/changebar/.

Exemple :
dans le préambule :
\usepackage[outerbars]{changebar} % permet de positionner les
% barres en marge externe
\setcounter{changebargrey}{20} % permet de fixer le niveau de
% gris des barres

et dans le document :
\begin{changebar}
   texte..
\end{changebar}

ou bla bla bla bla bla bla bla \cbstart bla bla bla bla bla bla
bla bla bla bla bla bla bla bla bla bla \cbend bla bla bla bla
bla bla bla bla bla bla bla bla bla bla bla bla bla bla 


 Comment encadrer du texte verbatim ?
------------------------------------------

* Le package 'moreverb' propose un environnement encadré.

* On peut également se définir son propre environnement. 

Exemple :
\newsavebox{\fmbox}
\newenvironment{fmpage}[1]
     {\begin{lrbox}{\fmbox}\begin{minipage}{#1}}
     {\end{minipage}\end{lrbox}\fbox{\usebox{\fmbox}}}


 Comment écrire du texte en forme de losange ou autre ?
------------------------------------------

Le package 'shapepar' disponible par ftp à ftp.loria.fr dans
/pub/unix/tex/ctan/macros/latex/contrib/other/misc/ définit des
environnements losange, coeur, etc.

Exemple :
\diamondpar{mon paragraphe en forme de losange.}


 Comment isoler les flottants par un trait horizontal ?
------------------------------------------

Pour que les flottants qui sont renvoyés en début (respectivement
en fin) de page soient isolés du texte par un trait horizontal en
dessous (respectivement au dessus), il faut activer les options
suivantes dans le préambule du document : 
\let\topfigrule\hrule
\let\botfigrule\hrule


 Comment insérer un espace vertical dans une page ?
------------------------------------------

Il suffit d'utiliser la commande \vspace et de préciser en
argument la hauteur de l'espace voulu.

Exemple :
bla bla bla bla 

\vspace{2cm}

blabla 

Remarque : la commande \vspace* force l'insertion d'un espace
	   vertical même si ce dernier se situe sur un
	   changement de page.


 Comment insérer un espace horizontal dans un texte ?
------------------------------------------

De la même manière que pour les espaces verticaux, il existe la
commande \hspace pour insérer un espace horizontal dans un
texte.

Exemple :
blabla bla \hspace{3cm} bli bli bli

Remarque : la commande \hspace* force l'insertion d'un espace
	   même en début ou en fin de ligne.


================================================================
[7] GESTION DES TABLEAUX
================================================================


 Comment gérer les tableaux de plus d'une page ?
------------------------------------------

* Le package 'supertabular' permet de gérer automatiquement (ou
presque) les coupures de tableaux. Il est disponible sur CTAN
dans /macros/latex/contrib/supported/supertabular/.
Il calcule la longueur du tableau à chaque \\ et vérifie si la
fin de page est atteinte. Si oui, il coupe le tableau. Les
commandes à utiliser sont : 
+ \tablefirsthead{...} pour définir le contenu de la première
occurence de la tête du tableau. Cette commande est optionnelle.  
+ \tablehead{...} pour définir le contenu de la tête des
tableaux suivants.
+ \tabletail{...} définit le contenu de la ligne qui doit être
ajoutée avant une coupure
+ \tablelasttail{...} définit le contenu de la dernière ligne
du tableau. Cette commande est optionnelle.
+ \topcaption{...}, \bottomcaption{...} et \tablecaption{...}
permettent de nommer un tableau soit au début ou à la fin du
tableau. \tablecaption{...} est l'option par défaut.

Exemple :
\begin{center}
   \tablefirsthead{\hline  \multicolumn{1}{|c}{Number}
   & \multicolumn{1}{c}{Number$^2$}
   & Number$^4$
   & \multicolumn{1}{c|}{Number!} \\ \hline}
   \tablehead{\hline \multicolumn{4}{|l|}{\small\sl continued
   from previous page}\\
   \hline \multicolumn{1}{|c}{ Number}
   & \multicolumn{1}{c}{Number$^2$}
   & Number$^4$
   & \multicolumn{1}{c|}{Number!} \\ \hline}
   \tabletail{\hline\multicolumn{4}{|r|}{\small\sl continued on
   next page}\\\hline} 
   \tablelasttail{\hline}
   \bottomcaption{This table is split across pages}
   \par
   \begin{supertabular}{| r@{\hspace{6.5mm}}|
      r@{\hspace{5.5mm}}| r | r|} 
      1   &     1  &        1  &           1    \\
      2   &     4  &       16  &           2    \\
      3   &     9  &       81  &           6    \\
      4   &    16  &      256  &          24    \\
      5   &    25  &      625  &         120    \\
      6   &    36  &     1296  &         720    \\
      7   &    49  &     2401  &        5040    \\
      8   &    64  &     4096  &       40320    \\
      9   &    81  &     6561  &      362880    \\
      10  &   100  &    10000  &     3628800    \\
      11  &   121  &    14641  &    39916800    \\
      12  &   144  &    20736  &   479001600    \\
      \hline & & & \\
      13  &   169  &    28561  &  6.22702080E+9 \\
      14  &   196  &    38416  &  8.71782912E+10\\
      15  &   225  &    50625  &  1.30767437E+12\\
      16  &   256  &    65536  &  2.09227899E+13\\
      17  &   289  &    83521  &  3.55687428E+14\\
      18  &   324  &   104976  &  6.40237370E+15\\
      19  &   361  &   130321  &  1.21645100E+17\\
      20  &   400  &   160000  &  2.43290200E+18\\
   \end{supertabular}
\end{center}

* Le package 'longtable', de D. Carlisle, disponible sur
CTAN dans macros/latex/packages/tools/, fonctionne de la même
manière mais mieux et permet de définir une taille de tableau
commune sur toutes les pages. Les commandes associées à ce
package sont \endfirsthead, \endhead, \endfoot et \endlastfoot.

\begin{longtable}
   {|p{0.2\linewidth}|p{0.2\linewidth}|p{0.2\linewidth}|}
   \hline 
   Premiere colonne & Deuxieme & Troisieme \endfirsthead
   \hline
   Premiere & Deuxieme & Troisieme \\
   \multicolumn{3}{|p{0.6666\linewidth}|}{Suite ... } \\ 
   \endhead
   \hline 
   \multicolumn{3}{|p{0.6666\linewidth}|}{Suite page suivante}
   \\ \hline \endfoot \hline
   \multicolumn{3}{|p{0.6666\linewidth}|}{C'est fini} \\
   \hline
   \endlastfoot \hline
   1   &     1  &        1  \\
   2   &     4  &       16  \\
   3   &     9  &       81  \\
   1   &     1  &        1  \\
   2   &     4  &       16  \\
   3   &     9  &       81  \\
   1   &     1  &        1  \\
   2   &     4  &       16  \\
   3   &     9  &       81  \\
   1   &     1  &        1  \\
   2   &     4  &       16  \\
   3   &     9  &       81  \\
\end{longtable}

au lieu de faire des calculs de largeurs de colonne, on peut
aussi mettre un \setlongtables dans le préambule et déclarer
ses tableaux comme à l'habitude en \begin{longtable}{|c|c|c|}
et LaTeX se charge du reste. Dans ce cas, l'ajustement des
colonnes peut nécessiter plusieurs (jusqu'à trois) compilations
enchaînées.

* Le package 'ltxtable' de D. Carlisle, disponible sur CTAN dans
/macros/latex/contrib/supported/carlisle/ permet de profiter des
fonctionnalités de tabularx et de longtable.
    

 Comment modifier l'orientation d'un tableau ?
------------------------------------------

* On peut utiliser la commande \rotatebox du package 'graphics'
de D. Carlisle. Ce package est disponible sur CTAN dans
/macros/latex/packages/graphics/.

Exemple : 
\rotatebox{90}{
\begin{tabular}{|c|c|} 
   \hline 
   asg & sasdf \\  
   \hline 
\end{tabular}
} 

* Le style 'lscape' de D. Carlisle marche aussi avec
l'environnement longtable (du même).

* De même l'environnement sidewaystable du package 'rotating'
permet d'inclure des tableaux en mode landscape. Le sens de
rotation peut être changé par l'option counterclockwise.

Exemple :
\usepackage[counterclockwise]{rotating}


 Comment nommer un tableau ?
------------------------------------------

Pour pouvoir associer une légende (\caption) à un tableau, il
suffit de l'encapsuler dans un environnement table.

Exemple :
\begin{table}[htbp] 
   \begin{center} 
      \begin{tabular}{|c|c|} 
	 \hline 
	 & \\ 
	 \hline 
      \end{tabular} 
      \caption{. \label{table-}} 
   \end{center} 
\end{table}


 Comment modifier l'épaisseur des lignes d'un tableau ?
------------------------------------------

* Pour obtenir des lignes horizontales fines ou épaisses dans un
tableau, il faut utiliser les commandes
\setlength{\doublerulesep}{\arrayrulewidth} dans l'entête du
document puis dans le tableau
\\\hline         pour une ligne fine
\\\hline\hline   pour une ligne épaisse

* Autre solution : la macro de A. Kessi (alain.kessi@psi.ch)

\makeatletter
\def\hlinewd#1{%
\noalign{\ifnum0=`}\fi\hrule \@height #1 %
\futurelet\reserved@a\@xhline} 
\makeatother

  \begin{tabular}{|l|r|}      \hline
  first       &   1       \\  \hline
  second      &   2       \\ \hlinewd{5pt}
  total       &   3     \\  \hline
  \end{tabular}

Même chose pour les lignes verticales.
  \begin{tabular}{@{\,\vrule width 5pt\,}c|c|}

* Le package 'easytable' disponible sur CTAN dans
/macros/latex/contrib/supported/easy/ propose différents styles
de lignes de séparation des cellules d'un tableau.
     

 Comment griser des cellules d'un tableau ?
------------------------------------------

* Le package 'colortab' disponible par ftp à princeton.edu dans
/pub/tvz/ ou sur CTAN dans /graphics/pstricks/inputs/ permet de
griser ou de mettre en couleur certaines parties de tableau.

Remarque : ColorTab n'est pas actuellement maintenu par Van 
	   Zandt et l'extension LongTable, avec laquelle il
	   était  compatible, ne fonctionne plus.

* Il est possible également d'utiliser le package 'shade'
disponible sur CTAN dans /macros/generic/.

* D. Carlisle propose également le package 'colortbl' sur CTAN
dans /macros/contrib/supported/carlisle/. Ce package permet
également de gérer de la couleur. Il nécessite les packages
'array' et 'color'.


 Comment changer la fonte d'une colonne ?
------------------------------------------

Pour préciser une fonte spécifique dans une colonne d'un
tableau, on peut utiliser le package 'array'. Ce dernier est
disponible sur CTAN dans /macros/latex/packages/tools/.

Exemple :
\begin{tabular}{<{\itfamily}l <{\slshape}l}
   

 Comment créer des notes de bas de page dans un tableau ?
------------------------------------------

* Il suffit d'encapsuler le tableau dans un environnement
minipage et pour supprimer le trait de séparation des notes de
bas de page de déclarer : \renewcommand{\footnoterule}{}.

Exemple :
\begin{minipage}[t]{5cm}
   \renewcommand{\footnoterule}{} % permet de supprimer le
   				  % trait de séparation
   \begin{tabular}{|c|c|}
      \hline
      sdfg& sdh\footnote{bls}\\ 
      \hline
   \end{tabular} 
\end{minipage}

Remarque : dans ce cas, la note apparaît juste après le tableau,
	   dans l'environnement minipage.

* On peut également utiliser :
\footnote[cptr]{texte.} 
toujours dans un environnement minipage, et gérer soit même ses
compteurs (cptr est un entier alors que les marques qui
apparaîssent dans le tableau sont les lettres de l'alphabet).
 
Remarque : ici aussi la note apparaît en dessous du tableau.

* Il existe également deux commandes spécifiques \footnotemark[]
et \footnotetext[]{}. \footnotemark permet de gérer le compteur
de notes et \footnotetext permet d'insérer le texte
correspondant en bas de page.

Exemple :
\begin{tabular}{|c|c|} 
   \hline 
   donnee1\footnotemark[1] & donnee2\footnotemark[2] \\
   \hline
\end{tabular} 
\footnotetext[1]{Note associ\'ee \`a la donn\'ee 1.} 
\footnotetext[2]{Note associ\'ee \`a la donn\'ee 2.}

Remarque : ici les notes apparaîssent bien dans le bas de page.


 Comment écrire un texte sur plusieurs colonnes ?
------------------------------------------

Il faut utiliser la commande :
\multicolumn{nb_colonnes}{alignement}{Texte}

Exemple :
\begin{tabular}[b]{||l|c||}
\multicolumn{2}{c||}{Texte sur 2 colonnes} \\
donnee1 & donnee2 \\
\end{tabular}

Remarque : dans le cas où le nombre de colonnes à couvrir est
	   égal à 1, cette commande peut être utilisée pour
	   modifier l'alignement (c, l, r, |, etc) d'une
	   cellule.
	   

 Comment passer certaines cellules en reverse vidéo ?
------------------------------------------

Le package 'color' disponible sur CTAN dans
/macros/latex/packages/graphics/ permet entre autres de le
faire. En fait il permet de définir des couleurs de texte et de
fond de page.

Exemples :
\tabcolsep=30pt
\begin{tabular}{|c|c|c|}
\hline
1&&\\
&\colorbox{black}{\strut{\color{white}Coucou}}&\\
&&2\\
\hline
\end{tabular}

ou (LaTeX2e)
\newlength\Coucoulength
\settowidth\Coucoulength{Coucou Coucou Coucou}
\begin{tabular}{|c|c|c|}
\hline
1&Coucou Coucou Coucou &\\
&\colorbox{black}{\makebox[\Coucoulength][c]{\color{white}Coucou}}
&\\
&&2\\
\hline
\end{tabular}


 Comment fixer la largeur d'une colonne ?
------------------------------------------

* Avec le package 'array', disponible sur CTAN dans
/macros/latex/packages/tools/, il faut utiliser l'option
m{largeur}.

Remarque : dans ce cas si l'on veut en plus spécifier un
	   alignement particulier, il faut utiliser une commande
	   du type {\raggedleft}.
	   
	   Exemple :
	   \begin{tabular}{|r|r|>{\raggedleft}m{190pt}|}
	      \hline
	      col1 & col 2 & col3 \\ 
	      \hline
	   \end{tabular}
	   
	   Malheureusement, cette dernière redéfinit souvent la
	   commande \\. 

L'accès à \\ ce fait alors par \tabularnewline ou en incluant :
\newcommand\PreserveBackSlash[1]{\let\temp=\\#1\let\\=\temp}
dans le préambule puis en utilisant :
\begin{tabular}{|r|r|>{\PreserveBackSlash\raggedleft}m{190pt}|}

* Le package 'easytable' disponible sur CTAN dans
/macros/latex/contrib/supported/easy/ permet facilement d'écrire
des tableaux dont les colonnes ont une largeur fixe.


 Comment écrire un texte sur plusieurs lignes ?
------------------------------------------

Il existe le package 'multirow' disponible sur CTAN dans
/macros/latex209/contrib/misc/.

Exemple :
\begin{table}[htbp] 
   \begin{center} 
      \begin{tabular}{|c|c||c|c|c|} 
	 \hline 
	 \multirow{2}{0.5cm}{k} & \multirow{2}{0.5cm}{$p_G$} &
	 \multicolumn{2}{c|}{test} \\
	 \cline{3-4}
	 & & DADWRD & RARWRD \\
	 \hline
	 2 & $1$ & 90 n & 228 n \\
	 3 & $p_d$ & 202 n & 449 n \\
	 4 & $p_d^2$ & 424 n & 891 n \\
	 5 & $p_d^3$ & 866 n  & 1774 n \\
	 \hline 
      \end{tabular} 
      \caption{Nombre d'opération par cellule des tests
      pseudo-aléatoires de fautes de type k-coupling.
      \label{table-compar}} 
   \end{center} 
\end{table}


 Comment diviser une cellule par une diagonale ?
------------------------------------------

Il faut utiliser le package 'slashbox' disponible par ftp à
ftp.tohoku.ac.jp dans
/pub/TeX/latex-styles/bear_collections/style-files/.


 Comment définir une colonne en mode mathématique dans
------------------------------------------

un tableau ?
------------
Avec le package 'array' disponible sur CTAN dans
/macros/latex/packages/tools/, il suffit de déclarer
\begin{tabular}{>{$}c<{$}cc} pour avoir une colonne en mode
mathématique et deux colonnes de texte.


 Comment modifier le nombre de tableaux par page ?
------------------------------------------

cf. question 8.3.


 Comment aligner des données sur le point décimal ?
------------------------------------------

Le package 'dcolumn' disponible sur CTAN dans
/macros/latex/packages/tools/ permet d'aligner les nombres d'un
tableau par rapport à leur point décimal.


 Comment obtenir des lignes partielles dans un tableau ?
------------------------------------------

La commande \cline{ColonneDebut-ColonneFin} est faite pour ça.

Exemple :
\begin{center} 
   \begin{tabular}{|c|c|c||c|c|c|c|c|c|c||c|} 
      \hline \hline 
      a & b & c & d & e & f & g & h & i & j & k \\  
      \cline{1-4} \cline{6-6} \cline{8-9}
      1 & 2 & 3 & 4 & 5 & 6 & 7 & 8 & 9 & 10 & 11 \\
      \hline 
   \end{tabular} 
\end{center}


 Comment éviter que du texte de grande taille atteigne
------------------------------------------

le cadre des cellules ?
-----------------------
Il faut pour cela utiliser la commande \strut qui simule un
objet vertical invisible,  après le changement de fonte.

Exemple :
\begin{tabular}{|l|}
   \hline
   {\large\strut  HELLO} dfg \\
   \hline
\end{tabular}


 Comment fixer la largeur d'un tableau ?
------------------------------------------

* Le package 'tabularx' disponible sut CTAN dans
/macros/latex/packages/tools/ permet de définir une largeur de
tableau.

* Le package 'easytable' disponible sur CTAN dans
/macros/latex/contrib/supported/easy/ permet facilement de fixer
des largeur de colonnes ou de lignes.


 Comment tracer des traits discontinus ?
------------------------------------------

Il faut utiliser les packages 'hvdashln' et 'array'.

Exemple :
dans le préambule du document :
\usepackage{hvdashln,array}
\setlength{\hdashlinewidth}{.5pt}
\setlength{\hdashlinegap}{2pt}

dans le texte :
$$
\left[ \begin{array}{ccc@{}>{\vdashline}c}            
        a_1 & b_1 & c_1 & d_1 \\
        a_2 & b_2 & c_2 & d_2 \\
        a_3 & b_3 & c_3 & d_3 \\
        a_4 & b_4 & c_4 & d_4
\end{array} \right]
$$

================================================================
[8] GESTION DES FIGURES
================================================================


 Comment inclure une figure ?
------------------------------------------

NOTE :	A. K. Goel a écrit un long document concernant les
	problèmes de gestion de figures et d'images dans LaTeX.
	Celui-ci est disponible par ftp anonyme à
	math.uwaterloo.ca dans ./pub/figsInLatex.ps.Z ou sur
	CTAN dans /info/figsinltx.ps
	
	De même K. Reckdahl a écrit "Using EPS Graphics in
	LaTeX2e Documents". Ce document est disponible sur CTAN
	dans /info/epslatex.ps.

* Sous LaTeX2.09, pour pouvoir appeler un fichier postscript,
il suffit de mettre l'option epsf dans le \documentstyle.
La figure peut ensuite être appelée par la commande
\epsfbox{nom-figure.(e)ps}

Une jolie façon d'inclure une figure dans un source LaTeX2.09
est d'utiliser la macro :

\begin{figure}[htbp] 
   \centerline{\epsfxsize=10cm \epsfbox{nom-figure.format}} 
   \caption{.  \label{fig-}}
\end{figure}

Les options htbp permettent de gérer le placement de la figure
dans le texte (cf. paragraphe 8.7).

* Sous LaTeX2e, il faut utiliser l'un des packages :
'graphics' ou 'graphicx' et la commande \includegraphics. Cette
commande accepte des options telles que dvips ou oztex (cf.
documentation pour plus de détails). 
Le package 'graphicx' a pour majeure différence avec 'graphics'
d'en simplifier les commandes. Ces packages sont disponibles sur
CTAN dans /macros/latex/packages/graphics/. Pour plus de
détails, consulter http://www.loria.fr/tex/packages.html.

Exemple :
\begin{figure}
   \begin{center}
      \includegraphics{images/fig1.ps}
   \end{center}
   \caption{\footnotesize blah blah blah}
\end{figure}

La commande \includegraphics du package 'graphicx' peut prendre
comme paramètres, angle, width, height, scale, clip et draft.

Exemple :
\includegraphics[width=\linewidth, draft=true]{figure.eps}

* Il y a aussi le package 'epsfig' disponible sur CTAN
dans /macros/latex/packages/graphics/ (LaTeX2.09 mais utilisable
avec LaTeX2e). 

Exemple :
\begin{figure}[p] 
   \centerline{\epsfig{file=nom_figure.eps, 
   width=largeur, 
   height=hauteur}}
   \caption{Titre.}
   \label{nom_label} 
\end{figure} 

* Une figure au format tex picture, tex picture + epic, tex
picture + eepic, ... peut être appelée directement par une
commande \input. En outre, si vous utilisez un format epic ou
eepic, il faut penser à rajouter 'epic' ou 'eepic' à la ligne
documentstyle ou d'inclure les packages 'epic' ou 'eepic'.


 Comment placer des figures côte à côte ?
------------------------------------------

* LaTeX2.09. Pour mettre des figures côte à côte, il suffit
d'encapsuler leur appel dans des minipages.

Exemple:
+ avec le package 'epsf'
\begin{minipage}[t]{.46\linewidth}
   \center\epsfxsize= 5cm \epsfbox{fig1.eps}
\end{minipage} % ne pas sauter de ligne
\begin{minipage}[t]{.46\linewidth}
   \center\epsfxsize= 5cm \epsfbox{fig2.eps}
\end{minipage}

ou si l'on veut attacher des titres aux figures :

+ avec le package 'epsfig'
\begin{figure}
 \begin{minipage}[b]{.46\linewidth}
  \centering\epsfig{figure=fig1.ps,width=\linewidth}
  \caption{premiere figure \label{fig1}}
 \end{minipage} \hfill
 \begin{minipage}[b]{.46\linewidth}
  \centering\epsfig{figure=fig2.ps,width=\linewidth}
  \caption{deuxieme figure \label{fig2}}
 \end{minipage}
\end{figure}

* Sous LaTeX2e, il faut utiliser le package 'graphics' ou
'graphicx' disponibles sur CTAN dans
/macros/latex/packages/graphics/.

Exemples :
\begin{figure}
   \begin{minipage}[c]{.46\linewidth}
      \includegraphics{figure1.format}
   \end{minipage} \hfill
   \begin{minipage}[c]{.46\linewidth}
      \includegraphics{figure2.format}
   \end{minipage}
\end{figure}

\begin{figure}
\includegraphics[width=5cm]{fig1.eps}\hfill
\includegraphics[width=5cm]{fig2.eps}
\caption{Titre commun}\label{fig:somefiglabel}
\end{figure}

* Une autre solution consiste à mettre chaque figure dans une
case d'un tableau.

* Si l'on veut un seul titre pour plusieurs figures voir le
package 'subfigure' disponible sur CTAN dans
/macros/latex/contrib/supported/subfigure/.

Exemple :
\begin{figure}[ht]
\begin{center}
  \subfigure[I]{\epsfig{figure=st1.ps,width=6.58cm}}\quad
  \subfigure[II]{\epsfig{figure=st2.ps,width=5.0cm}}\\
  \subfigure[III]{\epsfig{figure=st3.ps,width=5.0cm}}
\end{center}
\caption{Impermeable surface treatments}
\label{fig:inf}
\end{figure}

* Une autre possibilité pour obtenir un titre par figure est
d'utiliser le package 'epslatex' disponible sur CTAN dans
/info/.


 Comment modifier le nombre de figures par page ?
------------------------------------------

En fait, on ne peut agir que globalement sur le nombre de
flottants autorisé par page. Il n'y a pas de sélection
figure/tables/....

Il arrive fréquemment que lorsqu'un flottant dépasse 60% d'une 
page, LaTeX préfère changer de page plutôt que d'utiliser les 
40% de l'espace restant.  

La commande \floatpagefraction permet de redéfinir l'espace
minimum que peuvent occuper des flottants. Cela permet de
limiter le "blanc" sur une page contenant des flottants.

Exemple :
\renewcommand{\floatpagefraction}{.8} 
utilisée avec la commande :
\renewcommand{\textfraction}{.1}
permet de dire que le texte peut n'occuper que 10% d'une
page, et donc que des flottants peuvent occuper les 90% restant.

Il y a d'autre paramètres intéressants :
\setcounter{totalnumber}{4} 
qui détermine le nombre de flottants autorisés par page,
\renewcommand{\topfraction}{.8} et
\renewcommand{\bottomfraction}{.8}
qui indiquent la fraction maximum du haut ou du bas de la page
que peuvent occuper des flottants. 

Remarque : Il est recommandé de ne jamais mettre 100% comme
	   paramètre.


 Comment superposer du texte sur des figures ?
------------------------------------------

* PSFrag (disponible sur CTAN dans
/macros/latex/contrib/supported/psfrag) donne cette possibilité.
La solution consiste en fait à : 
1+ faire le graphique .ps AVEC des textes et légendes MAIS
approximatifs  
2+ utiliser PSfrag pour qu'il remplace les textes approximatifs
par des textes << LaTeX >> 
(3-) l'écriture << par dessus >> (le remplacement en fait) est
fait par PSfrag.
Pour plus de détails, consulter le document "Using EPS Graphics
in LaTeX2e Documents" disponible sur CTAN dans /info/epslatex.ps.

* On peut également générer une courbe dans un fichier .eps,
qui peux ensuite être inclu dans un environnement "picture",
dans lequel il est ensuite possible d'ajouter du texte avec des
\put. Cela demande pas mal de mises au point.

Exemple :
\setlength\unitlength{1cm}
\begin{picture}(10,10)
\put(0,0){\includegraphics{mondessin.eps}}
\put(10,10){Mon commentaire latex avec des $maths$}
\end{picture}

Cette technique permet de conserver la puissance de LaTeX et
d'avoir une typographie homogène mais elle a l'inconvenient
d'être assez lourde.

* PSTricks permet également de faire cela. Il nécessite un gros
investissement (il y a une centaine de pages de documentation).
Cependant, si l'on se limite à des commandes simples (comme par
exemple écrire un programme en C qui trace le dessin, avec
simplement des points, des droites, et un peu de texte), la
liste sommaire des commandes suffit (6 pages).

* Xfig offre une autre solution. Après avoir inclu un fichier
postscript généré par un autre programme, on peut rajouter
du texte ou des commandes LaTeX dessus.  xfig -sp 
Sauvegarder comme "combined PS/LaTeX"

* Metapost est un langage graphique très proche de Metafont,
mais qui génère du postscript. Il permet de produire des figures
avec du texte et est bien interfacé avec TeX (Knuth
l'utilise). Metapost est integré dans web2c 7.0. Metapost a
déjà été porté sous MS-DOS et Mac (CMacTeX). Pour plus
d'informations, vous pouvez consulter la page de D. Roegel :
http://www.loria.fr/~roegel/metapost.html.

* pstoedit permet également de visualiser des fichiers
postscript (sans bitmaps) et d'ajouter du texte ou des figures
par dessus.


 Comment réaliser des captures d'écran ?
------------------------------------------

* Un outil très utile pour effectuer des captures d'écran sous
Unix est xv. xv est un éditeur d'images écrit par  J. Bradley
disponible sous unix. xv est capable de gérer différents
formats d'image (encodage PS, GIF, JPEG, TIFF,...). Il
permet de visualiser des images et aussi de réaliser des
captures d'écran, soit partielles définies à la souris, soit
par fenêtre X Window. Il suffit ensuite de sauvegarder la
saisie d'écran de xv en format postscript et de l'appeler sous
LaTeX.

xv est accessible par ftp à ftp.ibp.fr.

* Sur PC il y a pcxdump disponible sur
http://micros.hensa.ac.uk. Le package 'verbtext', disponible sur
CTAN, permet ensuite d'appeler la saisie réalisée.

* De même, le package 'scrdumps' et l'utilitaire scr2tex.exe
sous DOS permettent d'inclure des saisies d'écran dans un
document LaTeX.


 Comment tracer une courbe ?
------------------------------------------

* Xgraphic est un outil de tracé de courbes simple
d'utilisation (les options sont accessibles en interactif) mais
limité aux courbes 2D. Il est disponible avec une doc française
à http://blanche.polytechnique.fr/ et
ftp://barbes.polytechnique.fr/pub/Xgraphic.

* xmgr marche également très bien.

* GNUplot est disponible sous Unix, sous Dos, sous Windows et
sous macOS
(http://www.ee.gatech.edu/users/schooley/gnuplot.html). Il
possède une sortie LaTeX (eepic). Il permet de tracer des
courbes (2D et 3D) à partir de valeurs ou d'une fonction. Les
formats de sortie sont LaTeX ou postscript. 

En outre, sous Unix, il est possible de récupérer des fichiers
GNUplot exportés par 
	set terminal fig
	set output "graph.fig"
puis de les modifier.

La FAQ GNUplot est disponible à
http://www.uni-karlsruhe.de/~ig25/gnuplot-faq/.

* Xy-pic compatible plain TeX, LaTeX2.09 et LaTeX2e permet de
tracer des courbes, de réaliser des diagrammes commutatifs, des
automates, et plein d'autres choses. Pour plus de
renseignements, consulter :
ftp ftp.diku.dk /diku/users/kris/TeX/ ou
ftp ftp.mpce.mq.edu.au /pub/maths/TeX/ ou
http://www.mpce.mq.edu.au/~ross/Xy-pic.html ou
http://www.diku.dk/users/kris/Xy-pic.html.
Le package 'xypic' est disponible sur CTAN dans
/macros/generic/diagrams/xypic.

* Sur PC grapher et surfer permettent également de tracer des
courbes et des surfaces (ils sont indépendants de LaTeX, mais
une saisie d'écran est toujours possible).

* Le package 'curves' disponible sur CTAN dans
/macros/latex/contrib/supported/curves/ permet de définir des
courbes dans l'environnement picture.

* Mathematica est également utilisable pour tracer des courbes
2D et 3D.


 Comment est géré le positionnement des figures ?
------------------------------------------

Comme mentionné précédemment (8.1), il existe différentes
options de placement des figures. Les plus classiques sont
\begin{figure}[htbp] pour laisser à LaTeX la possibilité de
   placer les figures suivant ses critères de beauté. (h) impose
   si possible le placement de la figure à l'appel de la macro
   ci dessus. Sinon la figure sera placée en haut de la page
   suivante (t) ou en bas (b), voire sur une page seule (p).

En revanche, pour forcer (dans la mesure du possible) LaTeX à
placer une figure là ou elle a été appelée, il faut utiliser le
package 'float', de A. Lingnau, (\usepackage{float}) et l'option
H (\begin{figure}[H]). Ce package permet de définir un tel
placement par défaut via la commande \floatplacement{figure}{H}.
Il est disponible sur CTAN dans
/macros/latex/contrib/supported/float/.

De même l'utilisation du caractère ! devant une option de
placement permet de forcer LaTeX2e à effectuer son placement au
plus tôt (suivant l'option choisie).


 Comment placer une légende à côté d'une figure ?
------------------------------------------

* Pour placer une légende à côté d'une figure, il faut utiliser
l'environnement minipage.

Exemple (extrait du cahier GUTenberg 22 pour l'article sur
esperluette) :
+ mettre dans le préambule :

\newlength\jataille  
\newcommand{\figgauche}[3]%
{\jataille=\textwidth\advance\jataille by -#1
\advance\jataille by -.5cm
\begin{minipage}[c]{#1}
   \includegraphics[width=#1]{#2}
\end{minipage}\hfill
\begin{minipage}[c]{\jataille}
   \footnotesize #3 \normalsize
\end{minipage}}

+ puis utiliser :
\figgauche{5cm}{totor.eps}{Titre.}

* On peut également utiliser des parbox.

Exemple :
\begin{figure}
\parbox{7cm}{...figure}\parbox{7cm}{\caption{---}}
\end{figure}


 Comment insérer des figures dans multicol ?
------------------------------------------

Pour forcer une figure à rester sur une seule colonne sous
l'environnement multicols, il faut utiliser :
\begin{figure*}[H] 
mettre sa figure
\end{figure*}

Exemple (avec le package 'graphicx') :
\begin{figure*}[H] 
    \includegraphics[width=3cm]{totor.eps}
\end{figure*}

Remarque : dans ce cas la gestion de \caption semble ne pas être
	   correcte.


 Comment faire apparaître toutes les figures en fin de
------------------------------------------

document ?
----------
Le package 'endfloat', disponible sur CTAN dans
macros/latex/contrib/supported/, permet de reporter toutes les
figures en fin de document.


 Comment insérer des images Mathematica ?
------------------------------------------

* Sous Unix ou sous DOS, il faut, à partir de Mathematica,
demander : Display["machin",truc]
qui sauve l'image truc dans le fichier machin dans un Postscript
embryonnaire, puis
!psfix -epsf machin > machin.eps
et on a un fichier EPS comme il faut.

* Sous Windows, après avoir sélectionné l'image désirée, il
faut, dans le menu "Fichier" de Mathematica, "Exporter" vers un
format qui peut être .EPS (PostScript Encapsulé). L'appel de ce
fichier .eps sous LaTeX se fait alors de manière classique (cf.
paragraphe 8.1).

Il est également possible d'extraire par copier/coller l'image
(.WMF) et de l'envoyer vers Ghostscript pour Windows, et la
dedans de l'enregistrer dans un fichier postscript. L'avantage
de cette solution est qu'elle peut permettre de retravailler
l'image (avant collage dans GS au moyen de CorelDraw, par
exemple). 

A noter que Y&Y TeX system supporte les images WMF (Windows
MetaFile) aussi bien que les TIFF ou EPSF. Mathematica est l'un
des seuls logiciels pour Windows qui utilise un format WMF
plutôt que TIFF.

* Dernière solution (multi-système) : se servir du notebook
intitulé "GnuDisplay.m"(disponible sur MathSource chez Wolfram).
Celui-ci permet d'exporter une image Mathematica en image
GNUplot. Dans GNUplot, on peut alors exporter une image de deux
façons :  
+ vers un fichier ".mf" qui contient l'image sous forme de
fonte. Avantage : on utilise MetaFont pour générer la fonte qui
contiendra l'image et l'insérer dans le texte, ce qui permet sa
prévisualisation immédiate par "dviscr", sans passer par "dvips"
puis GhostScript ou GhostView ;
+ vers un fichier au format LaTeX eepic, que l'on insère
facilement par la suite (commande \special).


 Comment modifier la taille d'une bounding box ?
------------------------------------------

En format eps (encapsulated postscript), la `bounding box`
permet de préciser la taille d'une image. Malheureusement, il
arrive parfois que la taille de la bounding box soit supérieure
à celle de la taille réelle du dessin qu'elle contient (il n'y a
pas de mise à l'échelle). Le package 'boxedepsf', de L.
Siebenmann, offre les commandes \Trim qui permettent de résoudre
le probleme.

A. J. Carr à adapté ce package à LaTeX2e. Son package s'appelle
'boxedeps'. Ce dernier est disponible sur CTAN dans
/macros/generic/boxed et par ftp à matups.math.u-psud.fr dans
/pub/TeX/Graphics.dir/ArtIntegration.dir/boxedeps.dir.

Exemple :
\TrimTop{15pct}\BoxedEPSF{toto}
avec pct = pourcentage de l'hauteur


Comment obtenir une figure avec un titre non numéroté ?
------------------------------------------

Il faut utiliser la commande \unnumberedcaption dont voici la
définition :
\makeatletter    % <=== in a .sty file delete this

\newcommand{\unnumberedcaption}%
	{\@dblarg{\@unnumberedcaption\@captype}}

\newcommand{\@unnumberedcaption}{}% undefined yet
\long\def\@unnumberedcaption#1[#2]#3{\par
  \addcontentsline{\csname ext@#1\endcsname}{#1}{%
    % orig: \protect\numberline{\csname the#1\endcsname}%
    %{\ignorespaces #2}
    \protect\numberline{}{\ignorespaces #2}%
    }%
  \begingroup
    \@parboxrestore
    \normalsize
    % orig: \@makecaption{\csname fnum@#1\endcsname}%
    %{\ignorespaces #3}\par
    \@makeunnumberedcaption{\ignorespaces #3}\par
  \endgroup}

% redefine \@makeunnumberedcaption (like \@makecaption)
% for your own layout
\newcommand{\@makeunnumberedcaption}[1]{%
  \vskip\abovecaptionskip
  \sbox\@tempboxa{#1}%
  \ifdim \wd\@tempboxa >\hsize
    #1\par
  \else
    \global \@minipagefalse
    \hbox to\hsize{\hfil\box\@tempboxa\hfil}%
  \fi
  \vskip\belowcaptionskip}

% for LaTeX 2.09 compatibility, define \above/belowcaptionskip:
\@ifundefined{abovecaptionskip}{%
  \newlength{\abovecaptionskip}%
  \setlength{\abovecaptionskip}{10pt}%
}{}
\@ifundefined{belowcaptionskip}{%
  \newlength{\belowcaptionskip}%
  \setlength{\belowcaptionskip}{0pt}%
}{}

\makeatother    % <=== in a .sty file delete this

Remarque : le package 'french' V3,49 inclut cette macro.


 Comment redéfinir le style de caption ?
------------------------------------------

Par exemple pour changer la fonte de Figure : en gras, il
faut utiliser : 
\makeatletter
\renewcommand{\fnum@figure}{\small\textbf{\figurename~\thefigure}}
\makeatother


 Comment fondre une image dans du texte ?
------------------------------------------

* Le package 'floatfig', de T. Kneser, disponible sur CTAN
dans /macros/latex/contrib/other/floatfig/, permet d'entourer
une figure de texte de manière très efficace grâce à
l'environnement floatingfigure. Ce package a été conçu pour
LaTeX2.09, pour des documents sans colonnes.

Exemple :
\begin{floatingfigure}{width of figure}
   figure contents
\end{floatingfigure}

* Le package 'floatflt', de T. Kneser et M. Dahlgren, disponible
sur CTAN dans /macros/latex/contrib/other/floatflt/, a été écrit
pour LaTeX2e. Il étend les possibilités de floatfig par de
nombreuses options et est utilisable pour les figures et les
tableaux.

Exemple :
\begin{floatingfigure}[options]{width of figure}
   figure contents
\end{floatingfigure}

* Le package 'picinpar' pour LaTeX2.09 est disponible sur CTAN
dans /macros/latex209/contrib/picinpar/ et dans
/systems/msdos/4alltex/disk04/. Il permet de définir un nombre
de lignes avant lequel la figure pourra être insérée dans le
texte. La taille de la figure dépend de son contenu, sa position
est variable, et peut s'étaler sur plusieurs paragraphes. 

ATTENTION : Ce package n'est pas compatible avec amstex.

Exemple :
\begin{window}[#lines before, l|r|c, picture contents, caption}
   ... paragraph text ...
\end{window}

* Mieux que le précédent, le package 'picins' disponible sur
CTAN dans /macros/latex209/contrib/picins/ permet d'inclure des
figures dans des paragraphes.

Exemple :
\parpic(width,height)(x-off,y-off)[Options][Position]{Picture}
    Paragraph text....

* Le package 'wrapfig' permet de définir la hauteur de la
figure, celle-ci ci peut apparaître à droite ou à gauche dans le
texte ou encore dans une marge. Ce package n'est pas compatible
avec les environnements de liste.

Exemple :
\begin{wrapfigure}[height in lines]{l|r}[overhang]{width}
   {figure, caption etc.}
\end{wrapfigure}

* Le package 'flow' met obligatoirement la figure dans une boîte
avec un cadre et ne permet pas de définir de caption.

Exemple :
\flow[L|R]{paragraph text}{figure box}

* Le package 'window' de E. Schaluck, permet également
d'intégrer une figure dans un paragraphe mais il n'est plus
maintenu. Il a été écrit pour LaTeX2.09 mais il est compatible
LaTeX2e.

Exemple :
\windowbox[toplines][inwindow: contents][ratio: l r]
... paragraph text ... \par
	
	
Voici les résultats d'un test comparatif effectué par P. van
Oostrum :
	    A B C D E F G H I J K
picinipar  |+|+|+| |+|+| |+| | | |
wrapfig    |+|+|+|+| | |+|H|-|+| |
flow       |-|-|+| | | | |+| | | |
floatfig   |+|-|-|+| | |+| | |-| |
floatflt   |+|+|+|+| | |+| |-| |+|
window     |-|-|+| |+|+| |+| | | |
picins     |+|-|+|+| | | |+|+| | |

avec :
A: figure captions/counting/list of figures
B: table captions/counting/list of tables
C: Left/right possible (+ = both)
D: Alternating left/right for twosided docs
E: can be placed in the middle of text with twosided flowing
F: possible to start after the beginning of paragraph
G: can (more or less) float in the text
H: auto detection of size of figure (H=height only)
I: works with list environments
J: works with twocolumn
K: works with multicol

Les meilleurs packages semblent être :
picins, floatflt et wrapfig.


 Comment réaliser des diagrammes en bâtons ?
------------------------------------------

Il existe le package 'bar', disponible sur CTAN dans
/macros/latex209/contrib/misc/, qui offre un environnement
barenv.


 Comment faire un organigramme ?
------------------------------------------

Il existe plusieurs contributions (toutes assez anciennes),
disponibles sur CTAN,  dédiées spécifiquement à la réalisation
de différents types d'organigrammes :
* /macros/latex209/contrib/nassflow/
* /macros/latex209/contrib/rail/
* /support/flow/
A priori, le dernier est le plus évolué et est basé sur un
pré-processeur écrit en langage C, ce qui fait qu'il offre une
interface souple et puissante.


 Comment centrer une figure très large ?
------------------------------------------

Le package 'bigcenter' ci-dessous permet de centrer des figures
très larges sans message d'erreur de type overful.

%%% ----------debut de bigcenter.sty--------------

%%% nouvel environnement bigcenter 
%%% pour centrer sur toute la page (sans overfull)

\newskip\@bigflushglue \@bigflushglue = -100pt plus 1fil

\def\bigcenter{\trivlist \bigcentering\item\relax}
\def\bigcentering{\let\\\@centercr\rightskip\@bigflushglue%
\leftskip\@bigflushglue
\parindent\z@\parfillskip\z@skip}
\def\endbigcenter{\endtrivlist}

%%% ----------fin de bigcenter.sty--------------


================================================================
[9] INCLUSION DE FICHIERS
================================================================


 Comment inclure des fichiers en mode verbatim ?
------------------------------------------

* Le package 'verbatim', de R. Schopf, permet via la commande
\verbatiminput, qui prend en argument un nom de fichier,
d'inclure un fichier en mode verbatim. Ce package est disponible
sur CTAN dans /macros/latex/distribs/ ou
/macros/latex/packages/tools/.

* L'environnement alltt du package du même nom (package dû à J.
Braams) permet la même prouesse tout en gardant active
l'interprètation des commandes LaTeX dont le nom commence par le
caractère \ (le "backslash" reste actif). Il est disponible dans
les archives CTAN dans /macros/latex/contrib/misc/ ou dans
/macros/latex/base/.

Exemple :
\begin{alltt}
   Notez la différence subtile entre $f(x)$ et \(f(x)\) grâce à
   l'utilisation du "backslash".
\end{alltt}

* Les commandes \listinginput et \verbatimtabinput du package
'moreverb' (disponible sur CTAN dans
/macros/latex/contrib/other/misc/) permettent d'inclure des
documents en mode verbatim avec ou sans numérotation des lignes
du fichier inclu. 

Exemple : 
{
\small
\listinginput[5]{10}{totor.c}
}
Les paramètres [5] et {10} indiquent que la numérotation des
lignes doit se faire de 5 en 5 en commençant à 10.

* Il existe aussi le package 'verbtext' disponible sur CTAN.

* Le package 'fancyvrb' est également disponible sur CTAN.

* Pour insérer du code lisp dans un document LaTeX, il existe le
package 'lispcode' disponible par ftp à
ki-server.informatik.uni-wuerzburg.de dans /pub/tex/.

* On peut aussi essayer le package 'verbasef' (verbatim
automatic segmentation of external files) disponible sur CTAN.
Il utilise l'environnement figure.

* Encore un autre, le package 'cprog' disponible sur CTAN dans
/macros/latex209/contrib/misc/ permet d'inclure des morceaux de
code dans un document et de les gérer comme des flottants.


 Comment gérer un document par parties ?
------------------------------------------

Pour travailler sur un gros document, il est plus agréable de le
découper en plusieurs fichiers. Il y aura quoiqu'il en soit un
fichier principal (celui qui comprend le préambule et les
commandes \begin{document} et \end{document}).

\input{fichier} permet d'inclure le fichier "fichier" dans le
fichier principal. Cette commande réalise une importation pure
et simple. Elle est plutôt réservée à l'importation de fichiers
de macros.

\include{chapitre} permet d'intégrer le fichier "chapitre.tex"
dans le document principal en commencant une nouvelle page.
Cette commande réinitialise la numérotation des titres. Utilisée
avec la commande \includeonly{chapitre}, cette commande  mise
dans le préambule, permet de ne recompiler le fichier principal
que sur les parties indiquées.

exemple :
\documentclass{report}
\includeonly{chap1, chap3}
\begin{document}
\include{chap1}
\include{chap2}
\include{chap3}
\end{document}

Remarque : avec le package 'french', pour que la numérotation 
	   des chapitres ne soit pas réinitialisée lorsqu'on
	   change de partie, il faut ajouter la commande
	   \noresetatpart  en début de document après le
	   \begin{document}


 Comment isoler une partie d'un fichier ps ou dvi ?
------------------------------------------

Il faut utiliser un outil qui permet de découper en page :
+ un fichier postscript,
* il s'agit des outils PSnup,PStoPS,PSSelect, qui ont été portés
sur Mac, aussi bien pour MPW qu'en "stand-alone" (dans la
distribution CMacTeX)
* on peut également utiliser ghostview (save marked pages)
disponible par ftp à iphthf.physik.uni-mainz.de dans
/pub/gv/gv_2_7_b5.tar.gz

+ un fichier dvi ?
DVIDVI pour MPW (utilitaire qui fait bien d'autres choses que
de découper) doit permettre de faire cela. 


 Comment inclure un fichier PICT ?
------------------------------------------

On peut utiliser la commande \special{pictfile mondessin.pict}
ou \put(0,0){\special{pict=Mondessin}}.


================================================================
[10] HAUTS ET BAS DE PAGES
================================================================


 Comment définir les hauts et bas de page ?
------------------------------------------


Remarque : lorsque le haut ou bas de page définit est trop grand
	   on voit apparaître des messages d'erreur du style 
	   "Overfull \vbox". Il faut alors redimensionner la
	   longueur correspondante.
	   
	   Exemple :
	   \addtolength{\headheight}{1.5pt}

* Par défaut, LaTeX offre la numérotation des pages en bas de
page (style plain). Mais, il propose également 3 autres  styles
de mise en page. Il s'agit des styles :
+ empty (hauts et bas de pages vides), 
+ headings(la numérotation des pages apparaît en haut ainsi que
différentes informations suivant la classe de document), et
+ myheadings (les commandes \markboth et \markright permettent
de définir les informations qui devront apparaître dans le haut
de page. \markboth{entête gauche}{entête droite} s'utilise pour
un document recto-verso alors que \markright{entête}
s'applique à toutes les pages d'un document en simple recto).

L'appel d'un style pour tout le document se fait via la commande
\pagestyle{style}. La commande \thispagestyle{} permet d'appeler
un style sur une page particulière. 

Remarque : malgré une déclaration globale de style de page, il
	   se peut que des déclarations locales de style soient 
	   également nécessaire, puisque certaines commandes
	   LaTeX réinitialisent le style de la page sur laquelle
	   elles apparaîssent.

* Le package 'fancyheadings', de P. van Oostrum, est disponible
sur CTAN dans /macros/latex/contrib/other/. Il est compatible
LaTeX2.09 et LaTeX2e. Il permet de définir des entêtes et des
pieds de page relativement facilement. La définition des pages
spéciales se fait par : \thispagestyle{xxx} ou xxx peut être
fancy (utilise les définitions ci-dessous sur une page en
respectant le style plain pour les autres ), plain (style TeX)
ou fancyplain(permet de redéfinir le style plain et donc de
disposer de deux styles : fancy et plain). L'application d'un
style à toutes les pages d'un document s'obtient par
\pagestyle{nom_style}.

Les macros principales sont :
+ pour les entêtes
\lhead[paire gauche]{impaire gauche}
\rhead[paire droit]{impaire droit}
\chead{centre}
+ pour les pieds de page
\lfoot[paire gauche]{impaire gauche}
\rfoot[paire droit]{impaire droit}
\cfoot{centre}

On distingue les styles fancy et plain par : 
\lhead[\fancyplain{paire gauche plain}{paire gauche fancy}]
{\fancyplain{paire gauche plain}{paire gauche fancy}}
Le style par défaut est alors plain.

Ce package définit quatre nouvelles longueurs :
+ \headrulewidth
+ \footrulewidth
+ \plainheadrulewidth
+ \plainfootrulewidth

Exemple :
\documentclass{article}
\usepackage{fancyheadings}
\pagestyle{fancy}
\usepackage{graphicx}
\renewcommand{\sectionmark}[1]{\markboth{#1}{}}
\renewcommand{\subsectionmark}[1]{\markright{#1}}
\rfoot{\leftmark\\\rightmark}
\lhead{\includegraphics[width=0.5cm]{foobar.ps}}

\begin{document}
Text.
   \section{Premiere section}
\end{document}

Remarque : sur certaines pages où le style de page est
	   réinitialisé par certaines commandes (telles que 
	   \tableofcontents), il faut repréciser le style voulu
	   via la commande \thispagestyle.
	   
	   Exemple :
	   \addtocontents{toc}{\protect\thispagestyle{fancyplain}}
	   
* Le package 'fancyhdr' est le successeur du package
'fancyheadings' pour LaTeX2e. Il est disponible sur CTAN dans
/macros/latex/contrib/supported/fancyhdr/.


 Comment utiliser le mode verbatim dans une note de bas
------------------------------------------

de page ?
---------
Par défaut, le mode verbatim n'est pas accessible dans une note
de bas de page (et de manière générale inutilisable dans un
argument d'une autre commande). 
* On peut alors le forcer par :
\DeclareRobustCommand\espacement{{\fontencoding{OT1}
\selectfont \texttt{\char32}}}
* ou utiliser 
\tt sous LaTeX2.09
\texttt sous LaTeX2e


 Comment mettre les notes de bas de page en fin de
------------------------------------------

document ?
----------
Le package 'endnotes', de J. Lavagnino, disponible sur CTAN
permet de faire cela. 


 Comment réduire les rappels de titres dans un haut ou
------------------------------------------

bas de page ?
-------------
Pour éviter que des titres trop longs n'apparaîssent dans les
entêtes ou les pieds de pages, il suffit de passer un titre
plus court en option des commandes de structuration de
document.

Remarque : dans ce cas, ce sont les titres courts qui
	   apparaîtrons dans la table des matières.

Exemple :
 \section[Un titre résumé.]{Un titre trop long qui surchargerai
mon entête.}


 Comment référencer une note de bas de page ?
------------------------------------------

Il suffit simplement de définir un label à l'intérieur de la
commande \foonote et de la référencer par \ref.

Exemple :
bla bla bla\footnote{Notons que ce bla l\`a est diff\'erent des
pr\'ec\'edents\label{footnote-bla}}...
[...]
..., comme l'indique la note~\ref{footnote-bla}, ...


 Comment supprimer la numérotation des pages ?
------------------------------------------

* Pour supprimer la numérotation des pages, il faut utiliser la
commande \pagestyle{empty}. Si elle ne suffit pas, il faut en
plus utiliser \thispagestyle{empty} sur les pages où la
numérotation susbsite. Ce comportement se justifie par le
fait que certaines commandes comme \tableofcontents ou \chapter
réinitialisent le style de page sur laquelle ils apparaîssent.

* On peut également redéfinir le style plain à empty
\let\ps@plain=\ps@empty.


 Comment numéroter les pages par rapport à la dernière ?
------------------------------------------

Pour pouvoir référencer les pages d'un document par rapport à la
dernière page (e.g. page 54/345), il faut utiliser le package
'lastpage' (disponible sur CTAN dans
/macros/latex/contrib/other/lastpage/).

Exemple (à ajouter dans le préambule) : 
\makeatletter
\renewcommand{\@evenfoot}%
	{\hfil \upshape page {\thepage} of \pageref{LastPage}}
\renewcommand{\@oddfoot}{\@evenfoot}
\makeatother

Si l'on ne dispose pas du package 'lastpage', on peut définir ce
label "à la main" en ajoutant \label{LastPage} juste avant
\end{document} ou encore en utilisant la commande
\AtEndDocument{\label{LastPage}} dans le préambule du document.


 Comment supprimer le trait de séparation des notes de
------------------------------------------

bas de page ?
-------------
Il suffit de mettre dans le préambule :
\renewcommand{\footnoterule}{}


 Comment modifier la numérotation des pages ?
------------------------------------------

* Il faut modifier la commande \thepage.

Exemple (P. van Oostrum) :
\renewcommand{\thepage}{\thechapter-\arabic{page}}
% chapter-page numbering
\@addtoreset{page}{chapter}
% reset page number when chapter is stepped
% The next magic makes the page counter be reset to one rather
% than zero
\def\@stpelt#1{\global\csname c@#1\endcsname
               \expandafter\ifx \csname#1\endcsname \page
                  \@ne
               \else
                  \z@ \fi}

* Il existe également le package ci-dessous de A. Kielhorn.
%% 
%% This is page-per-chapter-package
%% version 2.0
%%
%% Don't use it with refrep!
%% Refrep has these commands already implemented
%%
%% This version forces openright!!
%%
%% Index-commands should work in chapters and appedices,
%% they will not work as expected in the preface when the
%% pagenumbering is not arabic. (MakeIndex can't sort roman-
%% numbers)
%%
%% Bugs:
%%       The index is sortet according to the pagenumber
%%       without looking at the chapternumber.
%%       I don't think MakeIndex could handle that.
%%

\NeedsTeXFormat{LaTeX2e}
\ProvidesPackage{pagepc}[1995/05/13]

\@ifundefined{chapter}
{\PackageError{pagepc}{%
  You can't number your pages per chapter\MessageBreak when you 
  have no chapters
  }{%
  Use ``report'' or ``book'' instead.
  }
}%
{}

%% Reset the pagecounter to 1 at the start of a new chapter
%%
\let\ppchapter=\@chapter
\def\@chapter{\if@pageperchapter\setcounter{page}{1}\fi
                    \ppchapter}

%% Force a pagebreak at the start of the appendix, otherwise
%% the number of the page right before the appendix comes
%% out wrong
%%
\let\ppappendix=\appendix
\def\appendix{\if@pageperchapter\newpage\fi\ppappendix}

\newif\if@pageperchapter \@pageperchapterfalse

%% This command enables Page-per-Chapter, it is *not* on by
%% default to allow roman pagenumbers in the preface
%% (see sample-document)
%%
\newcommand{\pageperchapter}
    {\@pageperchaptertrue
     \@openrighttrue
     % Remember old setting for chapter 0 = preface
     \let\ppthepage=\thepage
     % The new number needs more space
     \renewcommand\@pnumwidth{2.55em}
     % Here it comes :-)
     \renewcommand\thepage{%
       \ifnum \c@chapter = \z@
          \ppthepage
       \else
          \thechapter\ -- \arabic{page}
       \fi
       }
     }

%% This is a hack to make MakeIndex happy :-(
%% You can't use the |-form of an indexentry because
%% it is used to store the chapternumber.
%%
\def\@wrindex#1{%
   \ifnum \c@chapter = \z@
      \protected@write\@indexfile{}%
      {\string\indexentry{#1}{\arabic{page}}}%
      % The above is wrong if pagenumbering!=arabic,
      % but I think this is better than nothing.
   \else
       \protected@write\@indexfile{}%
      {\string\indexentry{#1|ppc{\thechapter}}{\arabic{page}}}%
   \fi
 \endgroup
 \@esphack
}

%% This prints the pagenumber in the index
%%
\def\ppc#1#2{#1 -- #2}

\endinput


 Comment supprimer les entêtes et bas de page de pages
------------------------------------------

vierges ?
---------
Lorsqu'on utilise l'option 'openright' pour faire débuter un
nouveau chapitre sur une page de droite dans un document
recto-verso, pour ne pas afficher les entêtes et bas de page sur
une page de gauche restée vierge, on peut utiliser la commande
suivante (cf. LaTeX companion) :

\newcommand{\clearemptydoublepage}{%
	\newpage{\pagestyle{empty}\cleardoublepage}}

Il est alors nécessaire d'utiliser cette commande avant
la commande \chapter.


 Comment utiliser \footnote dans un titre ?
------------------------------------------

* Il faut utiliser la commande \protect.

Exemple :
\section{foo\protect\footnote{foooo}}

ATTENTION : le problème dans ce cas est que la note apparaîtra
	    aussi bien dans l'en-tête si l'en-tête rappelle le 
	    titre des sections que dans la table des matières.
	    Pour supprimer ces apparitions il faut alors
	    utiliser la même technique que pour les titres
	    résumés :
	    \section[foo]{foo\protect\footnote{foooo}}

* Il existe également le package 'stblftnt' de R. Fairbairns
disponible sur CTAN dans /macros/latex/contrib/other/misc/ qui
gère le problème énoncé ci-dessus sans avoir à réécrire tout
le titre.


 Comment placer les notes de bas de page les unes à
------------------------------------------

côté des autres ?
-----------------
Il faut utiliser le package 'footnote', de R. Fairbairns,
disponible sur CTAN dans
/macros/latex/contrib/supported/footnote/ avec l'option para :
\usepackage[para]{footnote}.


 Comment réinitialiser le compteur de note de bas de
------------------------------------------

page à chaque page ?
--------------------
Il faut utiliser le package 'footnote', de R. Fairbairns,
disponible sur CTAN dans
/macros/latex/contrib/supported/footnote/ avec l'option 
perpage : \usepackage[perpage]{footnote}


================================================================
[11] LE FRANCAIS ET LATEX
================================================================


 Comment françiser un document LaTeX ?
------------------------------------------

* 'babel' version 3.6 disponible sur CTAN dans
/macros/latex/packages/babel/, est un package de J. Braams.
babel permet de composer des documents multi-lingues. Son appel
sous LaTeX2e se fait par \usepackage[langue_1, langue_2, ...,
langue_n]{babel}. 

Parmi les langues actuellement disponibles, on peut citer
english, german, italian, french, français. Le changement de
langue se fait via la commande  \selectlanguage. 

Exemple :
\selectlanguage{spanish}

La langue par défaut est la dernière de la liste passée en
paramètre à babel (langue n dans l'exemple précédent).

Ce package est le plus courant sur le plan international mais il
est souvent contesté en France. On lui préfère alors le package
'french'.

* Le package 'french' de B. Gaulle est disponible par ftp à
ftp.univ-rennes1.fr dans ./pub/GUTenberg/french/ ou sur CTAN
dans /language/french/. La dernière version est la 3,49. Lors de
l'installation, respectez les instructions d'installation
accompagnant french.sty. Le fichier des motifs de césure est
désormais unique (frhyph.tex), que l'on travaille avec le codage
de fontes T1 ou OT1 (avec option MlTeX).

Remarque : Dans un fichier source (.tex), il est conseillé de
	   charger french après tous les autres packages.

french prend notamment en charge la mise en page, la traduction
des balises LaTeX visibles dans le document final (Chapitre,
Table des matières, ...), ....

ATTENTION : ces deux options (french et babel) ne sont pas
	    toujours parfaitement compatibles (les dernières
	    versions le sont french3,49 et babel 3.6).
	    
	    + \usepackage[francais]{babel} et
	    \usepackage[frenchb]{babel} font appel à l'option
	    frenchb maintenue par D.Flipo. 
	    + \usepackage[french]{babel} fait appel au package
	    french de B. Gaulle à condition que french ait été
	    installé (et plus particulièrement french.ldf) 
	    + les versions de tous ces packages sont très
	    importantes  (babel, frenchb, french) pour une bonne
	    coexistence. Une version 3.6 beta de Babel est sur
	    CTAN mais il parait  qu'il y avait un pb avec
	    french. Babel 3.5e et french V3.46 collaborent assez
	    bien dans tous les sens.   
	    + les fichiers de césure sont les mêmes pour babel
	    et pour french.
	    
ATTENTION : la prochaine distribution de french _ne sera plus
	    compatible_ LaTeX 2.09 !	    


 Comment corriger les coupures de mots accentués ?
------------------------------------------

* Une solution consiste à utiliser MlTeX (multilingual TeX).
C'est le moteur TeX de M. Ferguson. Il permet en particulier de
gérer les coupures de mots accentués. Certaines des idées
utilisées dans ce moteur ont d'ailleurs été reprises par la
suite dans TeX V3.

* Suivant la fonte avec laquelle on travaille (i.e. suivant
qu'elle dispose des caractères accentués ou non), il peut
exister des problèmes d'interaction entre les lettres accentuées
et les règles de coupure des mots. En effet, l'utilisation d'une
fonte 7 bits impose que les caractères accentués soient
fabriqués par la macro \accent de TeX qui inhibe toute coupure
pour la suite du mot.

En LaTeX2e l'utilisation de fontes respectant la norme T1
(codage de Cork + 8 bits) permet d'éviter ces problèmes. Il faut
alors faire appel au package 'fontenc' :
\usepackage[T1]{fontenc}
Ce package nécessite que des fontes encodées T1 (suffisamment
récentes) aient été installées (fontes dc par exemple).

Il faut en outre utiliser des modèles de coupure de mot encodées
T1. Il existe pour cela deux fichiers disponibles sur CTAN.  Il
s'agit des fichiers fr8hyph.dc (ou mieux f8hyph, beaucoup plus
récent) pour un codage 8 bits (fonte avec caractères accentués
telle que dc) et fr7hyph (respectivement f7hyph) pour un codage
7 bits (accents TeX). L'association GUTenberg propose également
de tels fichiers (cf. french paragraphe 11.1).

Pour plus de détails, consulter :
http://www710.univ-lyon1.fr/~fouet/gs/pres_page_princip

* Il peut y avoir également un problème de versions,
l'algorithme de coupure des mots à changé entre les versions
2.9 et 3.0. Ainsi si vous utilisez Tex V.3.0 ou plus, il faut
veiller à ce que les fichiers plain.tex et lplain.tex soient
également en version 3.0 ou plus.


 Comment utiliser les lettres accentuées ?
------------------------------------------

* Pour éviter les commandes barbares d'accentuation des
caractères, on peut utiliser une fonte contenant toutes les
lettres accentuées nécessaires (code 8 bits). Si besoin, il faut
préciser un codage d'entrée via l'instruction : 
\usepackage[codage d'entree]{inputenc} 
L'option est rendue nécessaire par le fait que les codes de
caractères au-delà de 127 sont différents sous Dos, MacOS et
Unix... Ainsi, le codage à preciser est latin1 pour un système
Unix ou un PC sous Windows, applemac sous Mac adapté au français
ou encore cp850 pour le code-page 850 sur PC (sous MS-DOS). Il
existe également ansinew, cp438, latin2 selon les systèmes. 

Exemple :
\usepackage[applemac]{inputenc}

* mapcodes de M. Piotrowski autorise également un grand nombre
de codages de caractères (iso8859-1 (latin1), iso8859-2
(latin2), ibm850 ou 852, hproman8, etc).


================================================================
[12] MATHEMATIQUES
================================================================


 Comment passer en mode mathématique ?
------------------------------------------

* Pour les mathématiques en ligne, il faut utiliser :
$ et $ ou
\( et \) ou 
\begin{math} et \end{math}

* Pour les formules isolées, il faut utiliser les modes :
 $$ et $$ ou
 \[ et \] ou
 \begin{displaymath} et  \end{displaymath}.
   
* Avec l'environnement \displaystyle, on peut quand même avoir
une formule de style isolée dans le texte mais cela modifie
localement les interlignes.

Exemple :
$\displaystyle \sum_{i=0}^n u_i $ 

* De même, on peut utiliser $\sum\limits{i=0}^n u_i$.

* Pour écrire des mathématiques sous LaTeX2e, les packages
'amsmath', disponible sur CTAN dans /fonts/ams/amslatex/, et
'amssymb' sont incontournables. En LaTeX2.09, on peut utiliser
'amstex', disponible sur CTAN dans /macros/amstex/, 'amsby' et
'amsopn' mais il faut savoir que amstex est obsolète.


 Qu'est que AMS-LaTeX ?
------------------------------------------

AMS-TeX est un terme utilisé pour désigner un ensemble de
fichiers distribués par l'American Mathematical Society (AMS).
AMS-LaTeX s'appuie sur TeX et LaTeX2e. AMS-LaTeX est
principalement dû a F. Mittelbach et R. Schopf. C'est un outil
complémentaire à LaTeX pour écrire des mathématiques (il devient
d'ailleurs rapidement indispensable). 

Il existe une FAQ disponible sur CTAN dans
/macros/latex/packages/amslatex/math/amslatex.faq

Les packages proposés sont 'amsmath' (complet), 'amstext' (pour
écrire du texte en mode mathématique), 'amsbsy' (pour les
symboles gras), 'amsopn' (pour la déclaration d'opérateurs),
'amsthm' (pour les environnements proof et theorem), 'amsintx'
(pour étendre la syntaxe des sommes et intégrales), 'amscd'
(pour les diagrammes commutatifs), 'amsxtra' et 'upref' (pour
les références croisées).

Ces packages sont disponibles par ftp à e-math.ams.org dans
pub/tex/amslatex/. Il faut également récupérer les fontes
associées dans /pub/tex/amsfonts/, /pub/tex/amsltx11/ et
/pub/tex/amstex/. Sur CTAN, les directories correspondantes sont
/macros/latex/packages/amslatex/, fonts/amsfonts/,
/macros/latex209/contrib/amslatex/ et /macros/amstex/.
Il existe également le site WEB http://www.ams.org/tex.


 Comment écrire les symboles d'ensembles ?
------------------------------------------

Un peu d'histoire (T. Bouche) : au temps jadis, dans les livres,
les symboles d'ensemble étaient imprimés en gras pour qu'on les
voie bien. Problème, comment, lorsque l'on écrit sur un tableau
noir, maintenir ce type de distinction ? En fait, au lieu
d'écraser la craie avec force sur le tableau pour élargir un
trait, on a décidé de le doubler. Ce qui est comique, c'est
qu'en retour, les livres se sont mis à copier l'usage des profs,
et ont distingué le "gras" (bold), du "gras de tableau" qui est
une fonte à part (style "détouré" ou "outline"). 

* Les symboles mathématiques d'ensemble et bien d'autres sont
disponibles par défaut dans les fontes AMS : famille msam
(e.g., msam10 pour 10pt) et msbm. Ces fontes sont disponibles
par ftp anonyme à e-math.ams.org dans /pub/tex/amsfonts ou sur
CTAN dans /fonts/ams/amsfonts/sources/symbols/. Pour avoir accès
aux symboles, il faut alors utiliser les packages 'amsfonts' et
'amssymb'.

Exemple :
soit on définit une commande spéciale :
	\newcommand{\R}{\mathbb{R}}
soit on tape directement :
	$\mathbb{R}$ (LaTeX2e) ou 
	${\Bbb R}$ (LaTeX2.09)

* Si vous ne disposez pas des packages ams vous pouvez au grand
dam de certains puristes utiliser les commandes :
+ pour les naturels :
	\def\N{\mbox{I\hspace{-.15em}N}}
+ pour les entiers :
	\def\Z{\mbox{Z\hspace{-.3em}Z}}
+ pour les reels :
	\def\R\mbox{I\hspace{-.15em}R}}
+ pour les complexes :
	\def\C{\mbox{l\hspace{-.47em}C}}
+ ...

Il faut savoir dans ce cas que leur utilisation est limitée :
pas de mise en indice, exposant, etc.

* C. Fiorio (fiorio@math.tu-berlin.de) propose également un
certain nombre de macros (LaTeX2e) :

%
%%  Les ensembles de nombres
%
\def\nbR{\ensuremath{\mathrm{I\!R}}} % IR 
\def\nbN{\ensuremath{\mathrm{I\!N}}} % IN 
\def\nbF{\ensuremath{\mathrm{I\!F}}} % IF 
\def\nbH{\ensuremath{\mathrm{I\!H}}} % IH
\def\nbK{\ensuremath{\mathrm{I\!K}}} % IK
\def\nbL{\ensuremath{\mathrm{I\!L}}} % IL
\def\nbM{\ensuremath{\mathrm{I\!M}}} % IM
\def\nbP{\ensuremath{\mathrm{I\!P}}} % IP
%
% \nbOne : 1I : symbol one
\def\nbOne{{\mathchoice {\rm 1\mskip-4mu l} {\rm 1\mskip-4mu l}
{\rm 1\mskip-4.5mu l} {\rm 1\mskip-5mu l}}}
%
% \nbC   :  Nombres Complexes
\def\nbC{{\mathchoice {\setbox0=\hbox{$\displaystyle\rm C$}%
\hbox{\hbox to0pt{\kern0.4\wd0\vrule height0.9\ht0\hss}\box0}}
{\setbox0=\hbox{$\textstyle\rm C$}\hbox{\hbox
to0pt{\kern0.4\wd0\vrule height0.9\ht0\hss}\box0}}
{\setbox0=\hbox{$\scriptstyle\rm C$}\hbox{\hbox
to0pt{\kern0.4\wd0\vrule height0.9\ht0\hss}\box0}}
{\setbox0=\hbox{$\scriptscriptstyle\rm C$}\hbox{\hbox
to0pt{\kern0.4\wd0\vrule height0.9\ht0\hss}\box0}}}}
%
% \nbQ   : Nombres Rationnels Q
\def\nbQ{{\mathchoice {\setbox0=\hbox{$\displaystyle\rm
Q$}\hbox{\raise
0.15\ht0\hbox to0pt{\kern0.4\wd0\vrule height0.8\ht0\hss}\box0}}
{\setbox0=\hbox{$\textstyle\rm Q$}\hbox{\raise
0.15\ht0\hbox to0pt{\kern0.4\wd0\vrule height0.8\ht0\hss}\box0}}
{\setbox0=\hbox{$\scriptstyle\rm Q$}\hbox{\raise
0.15\ht0\hbox to0pt{\kern0.4\wd0\vrule height0.7\ht0\hss}\box0}}
{\setbox0=\hbox{$\scriptscriptstyle\rm Q$}\hbox{\raise
0.15\ht0\hbox to0pt{\kern0.4\wd0\vrule height0.7\ht0\hss}\box0}}}}
%
% \nbT   : T
\def\nbT{{\mathchoice {\setbox0=\hbox{$\displaystyle\rm
T$}\hbox{\hbox to0pt{\kern0.3\wd0\vrule height0.9\ht0\hss}\box0}}
{\setbox0=\hbox{$\textstyle\rm T$}\hbox{\hbox
to0pt{\kern0.3\wd0\vrule height0.9\ht0\hss}\box0}}
{\setbox0=\hbox{$\scriptstyle\rm T$}\hbox{\hbox
to0pt{\kern0.3\wd0\vrule height0.9\ht0\hss}\box0}}
{\setbox0=\hbox{$\scriptscriptstyle\rm T$}\hbox{\hbox
to0pt{\kern0.3\wd0\vrule height0.9\ht0\hss}\box0}}}}
%
% \nbS   : S
\def\nbS{{\mathchoice
{\setbox0=\hbox{$\displaystyle     \rm S$}\hbox{\raise0.5\ht0%
\hbox to0pt{\kern0.35\wd0\vrule height0.45\ht0\hss}\hbox
to0pt{\kern0.55\wd0\vrule height0.5\ht0\hss}\box0}}
{\setbox0=\hbox{$\textstyle        \rm S$}\hbox{\raise0.5\ht0%
\hbox to0pt{\kern0.35\wd0\vrule height0.45\ht0\hss}\hbox
to0pt{\kern0.55\wd0\vrule height0.5\ht0\hss}\box0}}
{\setbox0=\hbox{$\scriptstyle      \rm S$}\hbox{\raise0.5\ht0%
\hboxto0pt{\kern0.35\wd0\vrule height0.45\ht0\hss}\raise0.05\ht0%
\hbox to0pt{\kern0.5\wd0\vrule height0.45\ht0\hss}\box0}}
{\setbox0=\hbox{$\scriptscriptstyle\rm S$}\hbox{\raise0.5\ht0%
\hboxto0pt{\kern0.4\wd0\vrule height0.45\ht0\hss}\raise0.05\ht0%
\hbox to0pt{\kern0.55\wd0\vrule height0.45\ht0\hss}\box0}}}}
%
% \nbZ   : Entiers Relatifs Z
\def\nbZ{{\mathchoice {\hbox{$\sf\textstyle Z\kern-0.4em Z$}}
{\hbox{$\sf\textstyle Z\kern-0.4em Z$}}
{\hbox{$\sf\scriptstyle Z\kern-0.3em Z$}}
{\hbox{$\sf\scriptscriptstyle Z\kern-0.2em Z$}}}}

Exemple :
$ \nbN $ pour les entiers naturels

* Autres alternatives :
en metafont:
+ les fontes 'bbold' d'A. Jeffrey (une sorte de Futura vraiment
doublé par endroit, pas détouré -- mais du coup pas du tout du
même style que les autres lettres du mode mathématique)

Exemple :
\font\bbold=bbold12
\newcommand{\R}{\mbox{\bbold R}}

en postscript  (produits commerciaux)
+ dextor outline 
+ Mathematical Pi (une sorte d'helvetica doublé par endroit)


Comment modifier la numérotation des équations ?
------------------------------------------

* Sous LaTeX2e, les options de classe leqno et fleqn permettent
d'indiquer si les numéros d'équations doivent apparaître à
droite ou à gauche des équations.

Exemple, pour que les numéros apparaîssent à gauche :
\documentclass[leqno]{report}

* Pour supprimer la numérotation des équations, il suffit
d'ajouter le caractère * aux noms des environnements d'équation.

Exemple :
\begin{eqnarray*}
      ...
\end{eqnarray*}

* Pour supprimer la numérotation d'une ligne particulière dans
un groupe d'équations, il suffit d'utiliser la commande
\nonumber (avant les \\). Avec amsmath, il faut utiliser \notag.

* Pour réinitialiser le compteur d'équations dans chaque
section, il faut inclure :
\makeatletter
\renewcommand\theequation{\thesection.\arabic{equation}}
\@addtoreset{equation}{section}
\makeatother
dans le préambule.

Remarque : dans ce cas, il ne faut pas mettre d'équation dans un
	   chapitre avant la première section sous peine d'avoir 
	   un numéro du style 3.0.1.

* Pour modifier le style de numérotation, il faut modifier la
commande \theequation.

Exemple :
\renewcommand{\theequation}{\thesection \Alph{equation}}

* Avec le package 'amsmath', la commande \numberwithin permet de
numéroter les équations suivant le paragraphe auquel elles
appartiennent. Il faut alors appeler : 
\numberwithin{equation}{section}
dans le préambule. 

* Les packages 'seceqn' et 'apeqnum' disponibles sur CTAN
permettent pour le premier de numéroter les équations par
section et pour le second de numéroter individuellement les
équations dans les annexes.

* Le package 'deleq' disponible sur CTAN dans
/macros/latex/contrib/supported/ ou dans
/macros/latex/contrib/other/deleq/ permet de définir un label
par groupe d'équations (4) en plus des labels individuels de
type (4a), (4b), etc.

* La commande \tag{xxx} du package 'amsmath' sous LaTeX2e permet
de personnaliser la numérotation des équations. 

Exemple : pour permettre à deux equations d'avoir le même
numéro :
\begin{equation}
  x - y = 0
  \label{equa}
\end{equation}
...
\begin{equation}
  x = y
  \tag{\eqref{equa}}
\end{equation}

Remarque : \eqref remplace \ref pour les équations : la fonte
	   utilisée est toujours la même (c'est plus beau).

* Les environnements subequations (du package 'amsmath') et
subeqnarray (du package 'subeqarray') permettent de référencer
différentes lignes d'un même groupe d'équations par des indices
(3.a), (3.b), etc. 
+ Sous subequations, une référence à une ligne particulière
s'obtient alors par un \label placé sur cette ligne alors qu'une
référence au système d'équation s'obtient par un \label placé
immédiatement après \begin{subequations}. 

Exemple :
\begin{subequations} \label{E:gp}
  \begin{gather}
    x_1 x_2 + x_1^2 x_2^2 + x_3 \label{E:gp1} \\
    x_1 x_3 + x_1^2 x_3^2 + x_2 \label{E:gp2} \\
    x_1 x_2 x_3 \label{E:gp3}
  \end{gather}
\end{subequations}

Remarque : c'est le \\ qui incrémente le compteur des
	   sous-références.

+ Sous subeqnarray une référence à une ligne particulière se
fait alors par \slabel au lieu de \label.


 Comment aligner des équations ?
------------------------------------------

* Pour aligner des équations sur un signe de relation, il suffit
sous LaTeX d'appeler l'environnement eqnarray avec ou sans la
commande \lefteqn ou si l'on dispose du package 'amsmath'
d'utiliser un des environnements split, multline, align ou
flalign.

Exemples :
\begin{eqnarray}
   \lefteqn{} \\
   &  &  
\end{eqnarray}

\begin{align}
   &
\end{align}

\begin{align}
   \label{eq1} 
   \begin{split}
      A &= B + C + D \\
      &\quad + E
   \end{split}
   \\
   \label{eq2}
   F &= G + H
\end{align}
 
 * L'environnement equationarray du package 'eqnarray' associe
 les  avantages des environnements eqnarray et array. Il n'est
 pas  limité en nombre de colonnes.

* Pour produire le système d'équations suivant : 
	 x  = y  + z
	 x1 = y1 + z1 
on peut utiliser le package 'alignat'. Toutefois, il faut noter
que ce package est alors détourné de son utilisation normale.
En effet, ce package est prévu pour aligner des objets
différents sur des colonnes alternativement justifiées à droite
puis à gauche. L'utilisation de colonnes vides permet alors de
choisir la justification voulue. D'autre part
l'utilisation de {} permet d'obtenir des espacements
"normaux".

Exemple :
\begin{alignat}{2}
   x & = y &&+ z\\
   x_1 & = y_1 &&+ z_1  
\end{alignat}

\begin{alignat}{5}
    10a& ={}&  3x&& 3y& +{}& 18z&& 2w&\\
     6a& ={}& 17x&&   & +{}&  5z&& 19w&
\end{alignat}


 Comment générer des vecteurs ?
------------------------------------------

* Il suffit d'utiliser le package 'amsmath' et d'appeler la
commande \overrightarrow 

Exemple : 
$\overrightarrow{u}$

* On peut également définir ses propres flèches notamment pour
régler leur hauteur par rapport aux lettres qu'elles surmontent.
La solution présentée ci-dessous utilise les packages 'amsmath',
'amssymb' et 'xy' :
\newcommand{\xyflecheverladroite}
{\mbox{\xymatrix{*{\hphantom{OM}}\ar[]+L;[]+R}}}

\newcommand{\ra}[1]
{\mathchoice
{\overset{\mbox{\xymatrix{*{\hphantom{\displaystyle #1}}
\ar[]+L;[]+R}}}{\displaystyle #1}}%
{\overset{\mbox{\xymatrix{*{\hphantom{\textstyle #1}}
\ar[]+L;[]+R}}}{\textstyle #1}}%
{\overset{\mbox{\xymatrix{*{\hphantom{\scriptstyle #1}}
\ar[]+L;[]+R}}}{\scriptstyle #1}}%
{\overset{\mbox{\xymatrix{*{\hphantom{\scriptscriptstyle #1}}
\ar[]+L;[]+R}}}{\scriptscriptstyle #1}}% }

%Pour changer la distance de la flèche, on peut procéder ainsi.
%\renewcommand{\ra}[1]
%{\overset{\raisebox{-1pt}{\mbox{\xymatrix{*{\hphantom{#1}}
\ar[]+L;[]+R}}}}{#1}}

Exemples :
$$ \ra{OM} $$
$\ra{OM}$ 
$\ra{OM_i}$ 
$$ \ra{OM} _{\ra{OM}_{\ra{OM}}}$$

* Le package 'vector', de N. Efford, est disponible sur CTAN
dans /macros/latex/contrib/supported/vector/. Il offre notamment
un certain nombre de vecteurs utiles aux physiciens et des
commandes automatiques d'énumération des coordonnées.

Exemple :
(\irvec[k]x}) donne (x1, ..., xk)

* Le package 'easyvector' disponible sur CTAN dans
/macros/latex/contrib/supported/easy/ permet de définir des
vecteurs suivant une syntaxe de type C.


 Comment écrire du texte en mode mathématique ?
------------------------------------------

* La commande \text{xxx} du package 'amsmath' permet d'inclure
du texte dans une formule mathématique sans que les accents et
les espacements mathématiques ne soient nécessaires.

* La commande \textrm{xxx} du package 'amsmath' (ou amstext)
permet en outre de prendre en compte la taille de la fonte
courante.

Exemple : 
$$ f_{\textrm{tracage}}  $$


 Comment ajuster la taille de délimiteurs ?
------------------------------------------

Pour ajuster la taille de délimiteurs tels que les
parenthèses, les accolades, etc, il suffit d'utiliser les
commandes \left et \right.

Remarque : si l'on veut utiliser un seul délimiteur (sans son
	   symétrique) il faut utiliser \right. en fermeture.

Exemples :
\left( \frac{1}{2} \right)
\left \{ et \right \}


 Comment changer de fonte en mode mathématique ?
------------------------------------------

Il faut utiliser les packages 'amsmath' ou 'amssymb' puis :
* pour du gothique
\frak{Texte} en LaTeX2.09 
\mathfrak{Texte} en LaTeX2e

* pour du caligraphique gras :
+ \boldsymbol{\mathcal{Texte}}
+ on peut aussi générer tout l'alphabet correspondant :
\DeclareSymbolFont{boldsymbols}{OMS}{cmsy}{b}{n}
\DeclareSymbolFontAlphabet{\mathbfcal}{boldsymbols}
puis utiliser la commande \mathbfcal

* pour du gras droit :
\mathbf{Texte}

* pour du gras italique :
\DeclareMathAlphabet\mbi{OML}{cmm}{b}{it}
ou
\DeclareSymbolFont{mathbold}{OML}{cmm}{b}{it}
\DeclareMathSymbol{\biGamma}{\mathord}{mathbold}{0}

* pour du gras sans serif :
\mathsf{\boldsymbol{Texte}}

* pour des symboles en gras :
\boldsymbol{\alpha}
ou
\DeclareSymbolFont{mathbold}{OML}{cmm}{b}{it}
\DeclareMathSymbol{\balpha}{\mathord}{mathbold}{11}

Remarque : si cela ne marche pas, c'est que les caractères gras
	   correspondants n'existent pas. Il faut alors soit
	   utiliser la commande \pmb{...} soit utiliser un
	   package supplémentaire tel que 'amsbsy'. Il existe
	   par ailleurs des versions postscript de fontes AMS
	   disponibles sur CTAN. 

* Pour écrire toute une formule en gras, on peut aussi utiliser
la commande \mathversion{xxx} de LaTeX2e. xxx peut valoir bold
ou normal.
Exemple :
\mathversion{bold}
$\sum_{i=0}^n u_i$


 Comment obtenir le L de la transformée de Laplace ?
------------------------------------------

Pour obtenir le L de la transformée de Laplace, il faut
utiliser le package 'mathrsfs' puis la commande
\renewcommand{\L}{\mathscr{L}}.

Remarque : ce package fait appel à des fontes particulières
	   qu'il faut également installer.


 Comment réaliser un tableau en mode mathématique ?
------------------------------------------

Il faut utiliser l'environnement array.

Exemple :
$$
\begin{array}{cc}
   & \\ 
\end{array}
$$


 Comment obtenir d'autres symboles mathématiques ?
------------------------------------------

Des symboles mathématiques supplémentaires (en plus de ceux
offerts par amsmath) sont disponibles dans les packages
'stmaryrd' disponible sur CTAN dans /fonts/stmaryrd/, et
'yhmath' également disponible sur CTAN dans
/macros/latex/contrib/supported/yhmath.


 Comment définir une fonction ?
------------------------------------------

* On peut utiliser la commande \mathop :
\newcommand{\fonction}{\mathop{fonction}}
Si on utilise en plus la commande \nolimits, on peut déterminer
la gestion des indices et exposants de cette fonction. 
Exemple :
\newcommand{\rad}{\mathop{\mathrm{rad}}\nolimits}

* Sous LaTeX2e, le package 'amsopn' est disponible sur CTAN dans
/fonts/ams/amslatex/ et par ftp à e-math.ams.org dans
/pub/tex/amslatex/inputs/. Il permet de déclarer de nouvelles
fonctions qui seront gérées comme une fonction mathématique
(gestion de la fonte, disposition des indices et exposants, etc).
Pour cela il faut déclarer une commande du type : 
\DeclareMathOperator{\fonction}{fonction} dans le préambule du
document. Pour que les indices et exposants de la nouvelle
fonction puissent être géres comme ceux de la fonction \sum , il
faut alors plutot utiliser \DeclareMathOperator*.

* De même, le package 'amstex' (LaTeX2.09) définit les commandes
\operatorname et \operatornamewithlimits.

Exemple :
$\operatorname{rad}$


 Comment définir une matrice ?
------------------------------------------

* Il suffit de construire un tableau sans lignes avec
l'environnement array.

Exemple :
\begin{array}{ccc}
   x_{11} & \cdots & x_{1p} \\
   \vdots & \ddots & \vdots \\
   x_{n1} & \cdots & x_{np} 
\end{array}

Pour encadrer cette matrice avec des délimiteurs, cf. paragraphe
12.8.

* Le package 'amsmath' permet de définir une matrice de manière
plus rapide qu'avec l'environnement array. Les environnements
disponibles sont :
+ matrix,
+ pmatrix pour une matrice encadrée par des parenthèses,
+ bmatrix pour une matrice encadrée par des crochets,
+ vmatrix pour une matrice encadrée par des lignes verticales,
+ Vmatrix pour une matrice encadrée par des doubles lignes
verticales.

Exemple :
\begin{pmatrix} 
         a & b \\
         c & d 
\end{pmatrix}

* Le package 'easybmat' disponible sur CTAN dans
/macros/latex/contrib/supported/easy/ permet facilement d'écrire
des matrices par bloc.

* Le package 'easymat' disponible sur CTAN dans
/macros/latex/contrib/supported/easy/ facilite également
l'écriture de matrices.


 Comment encadrer des formules ?
------------------------------------------

* Le package 'amsmath' offre la commande \boxed.

Exemple : 
$$\boxed{a=b}$$

* Il existe également la commande \fbox.

Remarque : Avec les environnements multi-lignes de AMSTeX (comme 
	   multline ou split), il peut être utile de les 
	   encapsuler dans un math ou dans un displaymath.

Exemple :
\begin{equation}
   \fbox{$
   \begin{array}{rcl}
      ce que vous voulez
   \end{array}
   $}
\end{equation}

* Le package 'fancybox' disponible sur CTAN dans
/macros/latex/contrib/other/seminar/inputs/ peut également être
utile.

Exemple :
\linethickness{3pt}
\Ovalbox{
\begin{Beqnarray}
ds^2&\,=\,& \displaystyle{\frac{\epsilon^{\prime 2}}{L^2}
\frac{12N}{(N+1)(N+2)}}
\end{Beqnarray}
}

* Une dernière solution consiste à mettre la formule dans un
tableau d'une seule cellule.


 Comment ajuster la longueur d'une flèche par rapport à un texte ?
------------------------------------------

Le package 'amsmath' permet de générer des flèches dont la
longueur dépend de la longueur du texte qui est placé au
dessus ou en dessous (ou de la chaîne la plus longue lorsqu'il
y a à la fois un texte au dessus et un autre en dessous).

Exemple : 
\xrightarrow[\text{audessus}]{\text{en dessous}}


 Comment obtenir des indices ou exposants à gauche ?
------------------------------------------

* Les commandes \sideset \overset et \underset du
package 'amsmath' permettent de placer du texte à gauche, en
haut ou en bas.

Exemple, mettre dans le préambule :
\newcommand{\transposee}[1]{{\vphantom{#1}}^{\mathit t}{#1}}

puis, dans le corps du document :
$$ 
\transposee{
\begin{pmatrix} 
   a & b \\
   c & d \\
\end{pmatrix}
}
$$
  
Remarque : on peut également utiliser \sideset qui ne marche
	   que pour les opérateurs :
	   \newcommand{\transposee}[1]{\sideset{^{\mathit{t}}}{}
	   {#1}}

* A défaut, on peut utiliser ${}_{j}H$ ou  ${_jH}$

* Le package 'chemsym' disponible sur CTAN dans
/macros/latex/contrib/other/chemsym/ peut également être utile.
  

 Comment tracer des diagrammes commutatifs ?
------------------------------------------

* Xy-pic est un outil simple et puissant qui permet de réaliser
de tels diagrammes (cf. paragraphe 8.6). 

Exemple :
\documentclass[a4paper,12pt]{article}
\usepackage[all]{xy}
\begin{document}
$$ \xymatrix{
  A \ar[d] \ar[r] \ar@{=}[rd] & B \ar[d] \\
  C \ar[r] & D }
$$
\end{document}

* On peut également utiliser le package 'amscd' ou plus
généralement le package 'amsmath' et l'environnement CD mais 
son offre est plus limitée.

Exemple :
$$
\begin{CD}
   \mathcal{F} @>\otimes>> T\\
   @VdrVlfV @ViVjV\\
   \mathtt{f} @= t
\end{CD}
$$

Remarque : dans la dernière version de amscd, la syntaxe
	   @>Exp1>Exp2> utilisée pour les flèches extensibles a 
	   été abandonnée au profit des commandes \xleftarrow et
	   \xrightarrow. 
	   
	   Pour plus de détails consultez :	   
	   + "\amslatex/ Version 1.2\\User's Guide", dans le
	   fichier "amsldoc.tex" 
	   + "Differences between \amslatex/ version 1.1 and
	   \amslatex/ version 1.2", dans le fichier "diff12.tex".

* Il existe le package 'cd' écrit pour LaTeX209 de D. Hankerson. 


 Comment ajuster la taille de certains opérateurs ?
------------------------------------------

* Pour ajuster la taille de certains opérateurs, il faut
utiliser le package 'exscale' disponible sur CTAN dans
/macros/latex/base/.

* Le package 'amsmath' donne la même possibilité.


 Comment mettre en page des algorithmes ?
------------------------------------------

* Les packages 'algorithms' et 'algorithmic' disponibles sur
CTAN dans /macros/latex/contrib/supported/algorithms/, ont été
spécifiquement conçus pour mettre en forme des algorithmes.

* Il existe aussi le package 'alg' disponible sur CTAN dans
/macros/latex/contrib/other/alg/.

* Le package 'newalg' est également disponible sur CTAN dans
/macros/latex/contrib/supported/newalg/.


 Comment mettre en page des formules longues ?
------------------------------------------

L. Lamport définit certaines conventions dans sa page WEB :
http://www.research.digital.com/SRC/personal/Leslie_Lamport/
latex/latex.html


 Comment créer des unités de mesure ?
------------------------------------------

Le problème du mode mathématique est qu'il change
automatiquement de fonte et qu'il utilise une gestion
particulière des espaces. Le mieux pour écrire des unités de
mesures du type m/s sans avoir à taper systématiquement des
barbarismes du type \rm\,m/s\mit ou \,\textrm{m}/\textrm{s} est
de définir une commande spéciale dans le préambule du document. 

Exemples : 
\newcommand{\units}[2]{#1\textrm{\thinspace #2}}
cette commande nécessite le package 'amsmath'
utilisation : \units{10}{m/s}

ou plus rapide :
\newcommand{\ms}[1]{\units{#1}{m/s}}
utilisation : \ms{10}


 Comment écrire proprement a/b ?
------------------------------------------

Pour qu'à l'impression le numérateur 'a' soit légèrement décalé
vers le haut et vers la gauche et que le dénominateur 'b' soit
légèrement décalé vers le bas et vers la droite, les deux étant
plus petits que la fonte courante, il existe la commande \sfrac
du package tugboat disponible sur CTAN dans
/macros/latex/contrib/supported/tugboat/.


 Comment créer des notes de bas de page dans une formule mathématique ?
------------------------------------------

* On peut utiliser directement la commande \footnote{} ou les
commandes spécifiques \footnotemark[] et \footnotetext[]{}
si l'on veut fixer soit même les compteurs. \footnotemark permet
de gérer le compteur de notes et \footnotetext permet d'insérer
le texte correspondant en bas de page.

Exemple :
$$
Place_{libre}\footnotemark[1] + 
Place_{occup\acute ee}\footnotemark[2] + 
Place_{d\acute efaillante} = 
Capacit\acute e
$$
\footnotetext[1]{La place libre~\ldots} 
\footnotetext[2]{La place occup\' ee~\ldots}


 Comment forcer le style d'un tableau à displaystyle ?
------------------------------------------

Le style par défaut dans un "array" est textstyle. Pour éviter
d'avoir à écrire \displaystyle dans chaque cellule, on peut
déclarer un tableau entièrement en mode displaystyle via la
macro: 
\newenvironment{disarray}%
 {\everymath{\displaystyle\everymath{}}\array}%
 {\endarray}
 

 Comment réduire les espaces gérés par eqnarray ?
------------------------------------------

Il suffit d'inclure la macro suivante dans le préambule de votre
document :

\makeatletter
\newlength{\earraycolsep}
\setlength{\earraycolsep}{2pt}
\def\eqnarray{\stepcounter{equation}\let\@currentlabel%
\theequation
\global\@eqnswtrue\m@th
\global\@eqcnt\z@\tabskip\@centering\let\\\@eqncr
$$\halign to\displaywidth\bgroup\@eqnsel\hskip\@centering
$\displaystyle\tabskip\z@{##}$&\global\@eqcnt\@ne
\hskip 2\earraycolsep \hfil$\displaystyle{##}$\hfil
&\global\@eqcnt\tw@ \hskip 2\earraycolsep
$\displaystyle\tabskip\z@{##}$\hfil
\tabskip\@centering&\llap{##}\tabskip\z@\cr}
\makeatother


 Comment réduire la taille des indices ?
------------------------------------------

Il faut utiliser la commande \DeclareMathSizes.

Exemple :
\DeclareMathSizes{10}{10}{6}{5}
\DeclareMathSizes{11}{11}{7}{5}
\DeclareMathSizes{12}{12}{7}{5}
Ces déclarations permettent de réduire la taille des indices des
fontes 10pt, 11pt et 12pt.
 

 Comment aligner des indices ?
------------------------------------------

* La commande \mathstrut définit une boîte invisible de largeur
nulle mais de hauteur et profondeur fixes.

Exemple :
$$ f_{\mathstrut d}=f_{\mathstrut\widehat{d}} $$

* On peut également utiliser la commande \vphantom dont la
hauteur est celle de son argument.

Exemple :
\vphantom{\alpha}\beta et \vphantom{\beta}\alpha


 Comment mettre en page des théorèmes ?
------------------------------------------

* Il existe pour cela le package 'thmmarks' disponible à
http://www.informatik.uni-freiburg.de/~may/thmmarks.html.

* Le package 'amsmath' définit l'environnement proof. 

* Le package 'newthm' disponible sur CTAN dans
/macros/latex/contrib/other/newthm/ permet de générer une liste
des théorèmes apparaissant dans un document.

================================================================
[13] REFERENCES CROISEES
================================================================


 Quelles sont les commandes de base ?
------------------------------------------

LaTeX permet de gérer automatiquement des références croisées
grâce aux commandes suivantes :
\label{} pour repérer la référence à rappeler
\ref{} pour faire appel à la référence décrite par la commande
\label  
\pageref{} pour faire référence à la page de la référence
reperée par la commande \label 
\cite{} pour faire référence à un document cité en
bibliographie.

ATTENTION : pour les figures et les tables, la commande \label
	    doit se situer juste après un \caption ou en faire 
	    partie pour que les références soient correctes.

ATTENTION : lorsque l'on travaille avec une version inférieure à
	    3.6 du package babel et l'option french, sachant que 
	    cette dernière redéfinit certains caractères de
	    ponctuation tels que :;?! pour gérer automatiquement
	    les espaces, il ne faut pas les utiliser dans des
	    noms de label : \label{section:Introduction} est
	    interdit.

Exemple :
\begin{table}[htbp] 
   \begin{center} 
      \begin{tabular}{|c|c|} 
	 \hline 
	 & \\ 
	 \hline 
      \end{tabular} 
      \caption{. \label{table-}} 
   \end{center} 
\end{table}


 Comment obtenir un renvoi à une page ?
------------------------------------------

* Il suffit d'utiliser la commande \pageref présentée ci-dessus.

Exemple :
voir tableau~\ref{tab} \`a la page~\pageref{tab}

* Le package 'varioref', de F. Mittelbach, disponible sur CTAN
dans /macros/latex/packages/tools/, permet de n'appeler une
référence de page que si l'objet flottant auquel elle se
rattache n'est pas présent sur la page courante. Il faut alors
utiliser \vref au lieu de \ref. Cette commande peut également
introduire des expressions du type "sur la page précédente",
"sur cette même page", etc. Dans ce cas, on précise la langue
voulue en option lors de l'appel du package.

Exemple:
\usepackage[french]{varioref}

Ce package définit la commande \vpageref qui ne fera apparaître
le numéro de la page référencée que si ce n'est pas la page
courante.


 Comment obtenir des références croisées à partir de plusieurs sources ?
------------------------------------------

Il faut utiliser le package 'xr' de D. Carlisle. Il est
disponible sur CTAN dans /macros/latex/packages/tools/. La
commande \externaldocument[KEY]{DOCUMENT} permet même de gérer
les références de même nom.

Exemple : dans le document B, il faut utiliser :
\usepackage{xr}
\externaldocument{A}


 Comment définir des liens hypertexte PDF sous LaTeX ?
------------------------------------------

* Le package 'hyperref' de S. Rahtz disponible sur CTAN dans
/macros/latex/contrib/supported/hyperref/ permet de définir des
liens hypertexte pour PDF. Il définit pour cela des commandes de
type \ref{}.

* La distribution Y&Y TeX System supporte des liens en TeX
debuggable dans le previewer qui seront automatiquement
convertis en liens PDF pour Adobe. 


 Comment faire référence à ses propres compteurs ?
------------------------------------------

Lorsque l'on définit son propre environnement avec ses popres
compteurs, pour que les commandes \label et \ref puissent les
prendre en compte, il suffit d'utiliser \refstepcounter au
lieu de \stepcounter pour incrémenter le compteur auquel on
fait référence.


 Comment faire des références extérieures à un fichier ?
------------------------------------------

Le package 'xr' permet de faire des références d'un document à
l'autre. Le document contenant les labels est appelé dans le
document contenant les références via la commande
\externaldocument.

Exemple :
* fichier essai.tex :
	\documentclass[a4paper,12pt]{book}

	\begin{document}
	   \section{Ma section.\label{ma-sect}}
	\end{document}

* fichier essai-ass.tex :
	\documentclass[11pt]{report}
	\usepackage{xr}
	\externaldocument{essai}
	
	\begin{document}
	\ref{ma-sect}
	\end{document}


================================================================
[14] TABLE DES MATIERES
================================================================


 Comment générer une table des matières par chapitre ?
------------------------------------------

Le package 'minitoc', disponible sur CTAN dans
/macros/latex/contrib/supported/minitoc/, permet de construire
une minitable des matières au début de chaque chapitre sous les
classes 'book' et 'report'. Pour l'utiliser, il faut appeler les
commandes \dominitoc avant la commande \tableofcontents
habituelle. La commande \faketableofcontents permet de ne garder
que les tables des matières locales et remplace alors la
commande \tableofcontents. La commande \minitoc doit être
appelée après chaque commande de début de chapitre \chapter dans
lequel on veut inclure une table locale. A chaque appel de
minitoc correspond un fichier .mtc ou n est le numéro du
chapitre.

Le compteur minitocdepth permet de fixer la profondeur des
tables des matières désirées. 

Exemple : 
\setcounter{minitocdepth}{1} 
pour ne faire apparaître que les titres de sections.

Le bon fonctionnement des numérotations peut nécessiter jusqu'à
trois compilations enchaînées.


 Comment ajouter une entrée dans la table des matières ?
------------------------------------------

Pour ajouter un titre de paragraphe qui n'apparaît pas par
défaut dans la table des matières, il faut utiliser la commande
\addcontentsline{1}{2}{3} où :
1 est l'extension du fichier contenant la table des matières
généralement 1 vaut toc, 
2 est un niveau définit dans la table des matières, ce peut être
chapter, section, ... et,
3 est le texte que l'on veut voir apparaître dans la table.

Exemples : 
+ \addcontentsline{toc}{subsection}{Modeles de fautes.}
+ \addcontentsline{toc}{chapter}{Bibliography}


 Comment changer le titre de la table des matières ?
------------------------------------------

On peut utiliser la commande :
\renewcommand{\contentsname}{Contenu du document.}


 Comment changer la profondeur de la table des matières ?
------------------------------------------

Les sections sont numérotées si leur profondeur est inférieure à
secnumdepth. 

Elles apparaîssent dans la table des matières si leur profondeur
est inférieure à tocdepth.

Il suffit de changer les valeurs de ces deux compteurs. Il peut
alors être nécessaire d'enchaîner deux compilations latex.

Exemple :
\documentclass[a4paper,12pt,openany]{book}

% Profondeur de \subsubsection = 3
\setcounter{tocdepth}{3}     % Dans la table des matieres
\setcounter{secnumdepth}{3}  % Avec un numero.

\begin{document}
\subsubsection{aaa}
\tableofcontents
\end{document}


 Comment gérer des chapitres de préface, d'introduction, et de conclusion non numérotés ?
------------------------------------------

* La classe book définit les commandes \frontmatter, \mainmatter
et \backmatter qui permettent de faire cela.

Exemple :
\documentclass[a4paper,12pt,openany]{book}
\usepackage{french}
 
\def\TEXTE{Bla bla bla...}
 
\begin{document}

\frontmatter

%\begin{abstract}  Bla, bla... \end{abstract}
 
\chapter{Préface} \TEXTE
\chapter{Introduction} \TEXTE
 
\mainmatter
 
\chapter{Début} \TEXTE
\chapter{Suite} \TEXTE
\chapter{Fin}  \TEXTE
 
\appendix
\chapter{Résultats élémentaires}
\chapter{Résultats compliqués}
 
\backmatter
 
\chapter{Conclusion}  \TEXTE
 
\tableofcontents
\end{document}

* L'autre solution consiste à définir ses propres commandes
\introduction, \preface, etc, à partir des définitions
de \chapter sans prendre en compte la numérotation.

* On peut également utiliser les commandes étoilées et gérer
soit même la table des matières et éventuellement les en-têtes
et bas de page.

Exemple :
\chapter*{Intro}
\addcontentsline{toc}{chapter}{Intro...}


 Comment enlever le numérotation des pages de tdm ?
------------------------------------------

Il suffit d'inclure les lignes suivantes dans le préambule du
document :
\makeatletter
\def\addcontentsline@toc#1#2#3{%
   \addtocontents{#1}{\protect\thispagestyle{empty}}%
   \addtocontents{#1}{\protect\contentsline{#2}{#3}{\thepage}}}
\def\addcontentsline#1#2#3{%
  \expandafter\@ifundefined{addcontentsline@#1}%
  {\addtocontents{#1}{\protect\contentsline{#2}{#3}{\thepage}}}
  {\csname addcontentsline@#1\endcsname{#1}{#2}{#3}}}
\makeatother


================================================================
[15] BIBLIOGRAPHIE SOUS LATEX
================================================================


 Comment construire une bibliographie ?
------------------------------------------

Il existe deux techniques pour construire une bibliographie :
+ la première consiste à l'intégrer dans le corps même du
document (fichier.tex), 
+ la seconde consiste à la définir dans un fichier particulier
(fichier.bib) puis à l'appeler dans le corps du document.

* Technique on-line :
Il suffit d'appeler l'environnement thebibliography et de
définir des bibitem.

Exemple :
\begin{thebibliography}{nombre de references}
   \bibitem[label]{cle} Auteur, TITRE, editeur, annee
   \bibitem[LAM94]{lam1} L. LAMPORT, {\it LATEX : A Document
   preparation system, Addison-Wesley, 1994}
   ...
\end{thebibliography}

[label] définit le label qui apparaîtra dans la bibliographie
et dans le texte après appel de la commande \cite{cle}.

* BibTeX permet de construire simplement une bibliographie. La
technique consiste alors à écrire un ou plusieurs fichiers.bib
qui seront appelés dans le document source par la commande
\bibliography{fichier1, fichier2,...}. BibTeX est disponible
sur CTAN dans /biblio/bibtex.

Le style de la bibliographie peut être défini par
\bibliographystyle{xxx} avec xxx valant généralement plain ou
alpha, mais il existe aussi d'autres styles. La commande
\bibliographystyle doit être appelée après \begin{document}.

Le contenu d'un fichier.bib est un ensemble de déclarations du
style :
@ARTICLE{cle, 
author = "liste-noms-auteur", 
title = "titre-article", 
journal = "nom-journal", 
year = "annee-parution",
}  
% OPTIONAL FIELDS 
% volume = "", 
% number = "", 
% pages =  "",  
% month =  "", 
% note =  "",

La syntaxe des différentes déclarations possible peut se
trouver dans toute bonne documentation LaTeX ou dans les menus
des éditeurs tels que emacs ou Alpha.

Losqu'on utilise BibTeX, il faut alors enchaîner plusieurs
compilations pour que toutes les références soient exactes.
Généralement, il faut exécuter une passe LaTeX, une
passe Bibtex et encore deux passes LaTeX.

* Pour pouvoir inclure dans la bibliographie des références à
des documents qui ne sont pas cités explicitement par une
commande \cite, il suffit d'inclure la commande
\nocite{la-ref-non-citee} avant la commande
\bibliography{fichier_biblio}. Pour faire apparaître toutes les
références non citées sans toutes les énumérer, on peut utiliser
\nocite{*}.


 Comment gérer plusieurs bibliographies ?
------------------------------------------

* Le package 'chapterbib' permet d'inclure une bibliographie
par chapitre. Il est disponible sur CTAN dans
/macros/latex/contrib/supported/cite/. Les bibliographies
réparties peuvent alors apparaître comme section de ce chapitre
grâce à la commande : \usepackage[sectionbib]{chapterbib}.

* Le package 'bibunits', de A. Fernandez, permet de partager une
bibliographie en plusieurs unités chacune attenante à une
section, ou à un chapitre d'un article ou d'un livre donné.

Exemple :
\documentclass[a4paper]{article}

\usepackage{bibunits}
\newcommand{\BIBLIO}[1]
{\nocite{*}%
  \bibliographystyle{amsalpha}%
  %\bibliographystyle{apalike}%
  \putbib[#1]}

\begin{document}
 \chapter{toto} ...  \BIBLIO{alg}
 \chapter{tutu} ...  \BIBLIO{geom}
\end{document}

* Le package 'bibtopics', de P. Basso, permet de contruire une
bibliographie générale comportant des rubriques diverses
(livres, articles, thèses, ...) ou des sujets divers (physique,
mathématiques, ...). Il est disponible par ftp à 
ftp.univ-rennes1.fr dans /pub/GUTenberg/contrib/.

* Voici un petit fichier 'multibib.sty' de V. Henn, créé en
recopiant les définitions latex des commandes relatives à la
gestion des bibliographies, et en les rendant paramétrables.

Exemple :
Les commandes qui ont été définies sont :
\newbibliographyGENE{biblio-x} pour déclarer une bibliographie
	biblio-x. 
\citeGENE{biblio-x}{clé} pour appeler une référence.
\bibliographystyleGENE{biblio-x}{alpha} pour définir le style de
	biblio-x.
\bibliographyGENE{biblio-x}{fichier-biblio1} pour faire
	référence au fichier fichier-biblio1.bib qui contient 
	les références.

%% ---------  debut de multibib.sty  -------
%%%%%% biblio generique %%%%%%%%

\newcommand\newbibliographyGENE[1]{%
\expandafter\expandafter\expandafter\newwrite\@nameuse{@#1aux}
\immediate\expandafter\expandafter\openout\@nameuse{@#1aux}=#1.aux
%\expandafter\expandafter\expandafter\show\@nameuse{@#1aux}
}

\def\citeGENE#1{\@ifnextchar [{\@tempswatrue\@citexGENE{#1}}%
{\@tempswafalse\@citexGENE{#1}[]}}
 
\def\@citexGENE#1[#2]#3{\if@filesw\immediate\expandafter%
\expandafter\expandafter\write\@nameuse{@#1aux}{\string%
\citation{#3}}\fi
 \let\@citea\@empty
 \@cite{\@for\@citeb:=#3\do
  {\@citea\def\@citea{,\penalty\@m\ }%
   \def\@tempa##1##2\@nil{\edef\@citeb{\if##1\space##2%
   \else##1##2\fi}}%
   \expandafter\@tempa\@citeb\@nil
   \@ifundefined{b@\@citeb}{{\reset@font\bf ?}\@warning
    {Citation `\@citeb' on page \thepage \space undefined}}%
   \hbox{\csname b@\@citeb\endcsname}}}{#2}}
 
\def\bibliographyGENE#1#2{\if@filesw\immediate\expandafter%
\expandafter\expandafter\write\@nameuse{@#1aux}{\string%
\bibdata{#2}}\fi
  \@input{#1.bbl}}
 
\def\bibliographystyleGENE#1#2{\if@filesw\immediate\expandafter%
\expandafter\expandafter\write\@nameuse{@#1aux}
    {\string\bibstyle{#2}}\fi}
 
\def\nociteGENE#1#2{\@bsphack
  \if@filesw\immediate\write\@nameuse{@#1aux}{\string%
  \citation{#2}}\fi
  \@esphack}
 
%% --------- fin de multibib.sty  -------

Exemple d'utilisation :
Les fichiers gene.bib et speci.bib contiennent respectivement
une référence bibliographie de clé cle-gene et cle-speci.

\documentclass{report}
\usepackage{multibib}

\begin{document}

\newbibliographyGENE{Toto}  
\newbibliographyGENE{Titi}  

\chapter{Introduction.}
 blabla bla \citeGENE{Toto}{cle-gene} et
\citeGENE{Titi}{cle-speci} rebla rebla rebla

\bibliographystyleGENE{Toto}{plain}
\bibliographyGENE{Toto}{gene}

\bibliographystyleGENE{Titi}{alpha}
\bibliographyGENE{Titi}{speci}

\end{document}

il faut alors compiler de la manière suivante :
latex fichier-source
bibtex Toto
bibtex Titi
latex fichier-source
latex fichier-source


 Comment changer de langue dans une bibliographie ?
------------------------------------------

* Il existe des versions françisées des styles bibliographiques
(fplain, falpha, fralpha, frealfullname...) qui ne sont pas
parfaits mais qui peuvent être modifiés facilement. Voir la
distribution GUTenberg.

Exemple :
\documentclass[12pt]{report}
\usepackage{../tex/inputs/frbib,french}
\bibliographystyle{../tex/inputs/fralpha}
\begin{document}
bla bla
\nocite{*}
\bibliography{essai}
\end{document}

@ARTICLE{c,  
author = "Jean-Paul Truc and John P. Machin", 
title = "titre", 
journal = "jiji", 
year = "1990",
}  

* Sachant que les items d'une bibliographie sont des
paragraphes, ils peuvent commencer par la spécification d'une
langue. Si on utilise le package 'french', on dispose des
commandes \french et \english et si l'on utilise le package
'babel', il faut utiliser la commande \selectlanguage{} (cf.
paragraphe 11.1).

* Voir également le package 'frbib' pour les bibliographies
françaises.


 Comment couper une chaîne de caractères ?
------------------------------------------

Pour forcer la coupure d'une chaîne de caractères, il faut
utiliser la commande \- à l'endroit ou la chaîne doit être
coupée.

Exemple :
cou\-per


 Comment renvoyer une référence en note de bas de page ?
------------------------------------------

* Pour remplacer une référence insérée dans le texte par la
commande \cite, par un appel à une note de bas de page
contenant une référence succinte à un document (la référence
détaillée apparaîssant toujours dans la bibliographie), il faut
redéfinir la commande \cite dans le préambule du document par : 
\makeatletter
\def\@cite#1#2{%
  \footnote{#1\if@tempswa, #2\fi}}
\makeatother

Les informations reportées dans la note de bas de page
dépendent du style de bibliographie choisi.

* Le package 'overcite', de D. Arseneau, est disponible sur CTAN
dans macros/latex/contrib/supported/cite/. Il permet de faire
référence à des documents par un exposant. 

Le problème qui se pose alors est de distinguer les références,
des notes de bas de page. Si le document contient moins de 10
notes de bas de page, on peut utiliser la commande
\renewcommand{\thefootnote}{\fnsymbol{footnote}}. Sinon pour
disposer de 10 notes de bas de page par page, il existe le
package 'footnote' de R. Fairbairns disponible sur CTAN dans
/macros/latex/contrib/supported/footnote.

Exemple :
\usepackage[perpage,symbol]{footnote}

* Le package 'camel' disponible sur CTAN dans
/macros/latex/contrib/supported/camel/, propose différents
styles de bibliographies renvoyant à des notes de bas de page.

* E. Domenjoud a également écrit un package qui permet de
faire cela. Eric.Domenjoud@loria.fr.


 Comment faire référence à un document ?
------------------------------------------

Il suffit d'utiliser la commande \cite avec en paramètre la clé
du document (cf. paragraphe 15.1). La commande \cite peut
prendre un texte en option.

Exemple :
\cite[chapitre 1]{cle}
pour obtenir [cle, chapitre 1]


 Comment grouper des références multiples ?
------------------------------------------

* Le package 'cite' de D. Arseneau est disponible sur CTAN
dans /macros/latex/contrib/supported/cite/. Il permet de trier
et de grouper les références multiples. Par exemple, il permet
automatiquement de générer [1-6] au lieu de lister [1, 2, 3, 4,
5, 6].

* Il existe également le package 'mcite' disponible sur CTAN
dans /macros/latex/contrib/supported/mcite/.


 Comment changer le titre de la bibliographie ?
------------------------------------------

On peut utiliser dans le préambule, la commande : 
\renewcommand{\bibname}{nouveau titre}


 Comment changer le style de la bibliographie ?
------------------------------------------

* Il existe un certain nombre de style par défaut (alpha,
plain, unsrt, abbrrv) que l'on peut appeler par la commande
\bibliographystyle{xxx} placée après la commande
\begin{document}.

Remarque : Certains styles comme plain, acm ou apalike trient
	   alphabetiquement les références d'une bibliographie 
	   suivant le premier auteur de chaque référence.
	   D'autres comme alpha utilisent la première lettre du
	   nom de chacun des auteurs multiples.

* Il existe également le package 'custom-bib' disponible sur
CTAN dans /macros/latex/supported/custom-bib/ qui permet de
définir ses propres styles sans avoir à se plonger dans le
language BibTeX. Le programme est interactif.

* De même le package 'natbib' est disponible par ftp à
ftp.loria.fr dans /macros/latex/contrib/supported/natbib. Il
permet entre autres de classer la bibliographie de plusieurs
façons différentes, notament par ordre alphabétique pour les
auteurs, par l'année de parution, etc.

* Pour remplacer la numérotation [1] par 1., on peut utiliser la
commande :
\makeatletter
\renewcommand{\@biblabel}[1]{\quad #1.}
\makeatother

* Pour supprimer toute numérotation il faut utiliser de la même
manière :
\makeatletter
\renewcommand{\@biblabel}[1]{}
\makeatother


 Comment construire une bibliographie à partir de plusieurs fichiers .bib ?
------------------------------------------

Pour cela il existe le package 'biblist' de J. chrod, disponible
sur CTAN dans /macros/latex209/contrib/biblist/.

Exemple :
\documentclass[12pt]{article}
\usepackage{biblist}
\begin{document}
\nocite{*}
\bibliographystyle{plain}
\bibliography{bibfile1,bibfile2,...}
\end{document}


 Comment utiliser la commande \cite dans un \caption ?
------------------------------------------

On peut mettre la commande \cite dans un \caption à condition de
la protéger : \protect\cite. 

Remarque : Ceci n'est plus nécessaire pour les nouvelles
	   versions de LaTeX.


 Comment référencer une thèse française ou un mémoire ?
------------------------------------------

Il faut définir de nouvelles références. Pour cela, copier le
fichier .bst dans un autre fichier .bst avec un nom différent
et rechercher PhD thesis puis remplacer par Thèse de doctorat.

Exemple :
FUNCTION {thesedoc}
{ output.bibitem
format.authors "author" output.check
new.block
format.btitle "title" output.check
new.block
"Th\`ese de Doctorat" format.thesis.type output.nonnull
school "school" output.check
address output
format.date "year" output.check
new.block
note output
fin.entry
}

Idem pour les mémoires de DEA :

FUNCTION {memdea}
{ output.bibitem
  format.authors "author" output.check
  new.block
  format.title "title" output.check
  new.block
  "M\'emoire de D.E.A." format.thesis.type output.nonnull
  school "school" output.check
  address output
  format.date "year" output.check
  new.block
  note output
  fin.entry
}


 Comment supprimer la virgule supplémentaire dans une liste d'auteurs ?
------------------------------------------

Par défaut, dans une liste d'auteurs le "et" qui introduit le
dernier auteur est précédé d'une virgule. Pour la supprimer, il
faut modifier la fonction format.names dans un nouveau fichier
.bst de la manière suivante :

FUNCTION {format.names}
{ 's :=3D
  #1 'nameptr :=3D
  s num.names$ 'numnames :=3D
  numnames 'namesleft :=3D
    { namesleft #0 > }
    { s nameptr "{ff~}{vv~}{ll}{, jj}" format.name$ 't :=3D
      nameptr #1 >
	{ namesleft #1 >
	    { ", " * t * }
	    { namesleft #2 >
		{ "," * }
		'skip$
	      if$
	      t "others" =3D
		{ " et~al." * }
		{ " and " * t * }
	      if$
	    }
	  if$
	}
	't
      if$
      nameptr #1 + 'nameptr :=3D
      namesleft #1 - 'namesleft :=3D
    }
  while$
}


 Comment configurer la commande \cite ?
------------------------------------------

Le package 'cite' disponible sur CTAN dans
/macros/latex/contrib/supported/cite/ offre certaines
possibilités.

Exemple :
\usepackage{cite}
\renewcommand\citepunct{;\penalty999\ }


 Comment construire une liste d'auteurs ?
------------------------------------------

Pour séparer les éléments d'une liste d'auteurs il faut
utiliser "and". 

Exemple :
Dewitt, D.J. and Naughton, J.


 Comment spécifier un tri dans une bibliographie ?
------------------------------------------

Il existe l'outil BibTool disponible sur CTAN dans
/biblio/bibtex/utils/bibtool/ ou sur
http://www.uni-koblenz.de/~gerd/ftp/BibTool. BibTool est un
outil de manipulation de bases de données BibTeX et il permet en
particulier de spécifier différents ordres de tri pour une
bibliographie.

Exemples :
* bibtool -s file.bib -o the_new_bibtex_file.bib
pour trier suivant les auteurs et les titres
* bibtool -s -- 'sort.format={%d(year)}' file.bib -o
the_new_bibtex_file.bib 
pour trier par ordre croissant des années de parution
* bibtool -s -- 'sort.format={%s($type)%N(author)}' gn-publ.bib
pour trier par classe de document et par auteur.

L'appel du fichier ainsi trié se fait alors par exemple par :
\documentclass{article}
\begin{document}
\nocite{*}
\bibliographystyle{unsrt}
\bibliography{the_new_bibtex_file}
\end{document}


 Comment référencer les pages contenant des citations ?
------------------------------------------

Il faut utiliser le package 'backref' disponible sur CTAN.


 Où trouver des styles de bibliographie ?
------------------------------------------

* Le package 'biblist' de J. Schrod offre différents styles de
bibliographie prédéfinis. Il est disponible sur CTAN dans
/macros/latex209/contrib/biblist/.


 Comment faire des références croisées ?
------------------------------------------

Il faut utiliser le champ crossref de bibtex.

Exemple :
@InProceedings{contejean96rta,
  author =      "Evelyne Contejean and Claude March\'e",
  title =       "{CiME: Completion Modulo $E$}",
  crossref =    "rta96",
  pages =       "416--419",
  year =        1996,
  note =        "System Description",
  ftp =         "ftp://ftp.lri.fr/LRI/art/march/cime-rta96.ps.gz",
  abstract =    "http://www.lri.fr/~marche/cime-rta96.html"
}

@Proceedings{rta96,
  title         = "7th International Conference on ...",
  booktitle     = "7th International Conference on ...",
  editor        = "Harald Ganzinger",
  publisher     = SV,
  year          = 1996,
  month         = jul,
  address       = "New Brunswick, NJ, USA",
  series        = LNCS,
  volume        = 1103,
}

ATTENTION : l'ordre de déclaration @InProceedings/@Proceedings
	    dans le fichier .bib est important.


 Comment citer une URL ?
------------------------------------------

* Le style harvard propose un champ URL. Il est disponible à :
http://www.arch.su.edu.au/~peterw/latex/harvard/ ou sur CTAN
dans /macros/latex/contrib/supported/harvard/.

* Le style bibliographique 'utphys', de J. Distler, disponible à
http://xxx.lanl.gov/hypertex/bibstyles ajoute le champ eprint à
toutes les entrées bibtex classiques.

* On peut également utiliser la macro @MISC et mettre la
référence URL dans le champ note.


 Comment définir des initiales à deux lettres ?
------------------------------------------

En français, les prénoms commencant par Ch, Ph, Th... ont pour
initiales leurs deux premières lettres (Ex : Philippe --> Ph.).
Pour forcer BibTeX à considérer ces groupes de lettres, il faut
utiliser : {\relax Ph}ilippe. BibTeX prend alors tout le groupe
{..} pour une lettre. La commande \relax est considérée comme
une commande d'accent qui est supprimée lors du tri
alphabetique. Elle est nécessaire car lorsqu'un groupe apparaît
dans un nom, BibTeX s'attend à trouver une commande d'accent
juste après l'accolade ouvrante.


 Comment conserver les majuscules dans les titres ?
------------------------------------------

Il suffit de mettre les majuscules entres accolades.


 Comment changer l'espace entre les item ?
------------------------------------------

Il faut copier la définition de l'environnement thebibliography
qui se trouve dans le fichier de style de la classe utilisée
(par exemple article.cls pour un article) dans un fichier de
style (qui sera appelé via une commande \usepackage) ou dans le
préambule de votre document (entre \makeatletter, \makeatother).
Il faut ensuite remplacer \newenvironment par \renewenvironment
et modifier la définition en ajoutant \setlength\itemsep{0pt}
après \list.

================================================================
[16] INDEX
================================================================


 Quelles sont les commandes de base ?
------------------------------------------

Pour permettre à LaTeX (via un générateur d'index) de construire
un index, il faut référencer dans le texte tous les mots que
l'on souhaite y voir apparaître. La commande à utiliser pour
cela est \index. Elle permet la construction d'un fichier .idx
dans lequel sont répertoriées toutes les références indiquées
dans le texte. 

Les entrées de ce fichier sont ensuite triées dans un fichier
nom_fichier.ind qui sera inclut dans le source .tex par la
commande \printindex ou \input{nom_fichier.ind}.

Pour indiquer à LaTeX qu'il doit construire un index, il faut
également ajouter la commande \makeindex dans le préambule du
document.

Remarque : compte tenu du temps pris pour générer un index, il
	   est conseillé de ne faire apparaître la commande 
	   \makeindex que lors des dernières compilations.

Si les commandes de génération d'index ne sont pas définies par
défaut, ou dans un package autre que makeidx (tel que french de
B. Gaulle par exemple), il faut utiliser le package 'makeidx'
(cas notamment de la commande \printindex).

Note : Pour éviter de répéter la commande \index dans un
       texte où elle devrait apparaître souvent, on peut
       utiliser les commandes \index{mot_a_indexe|(} au début
       du texte et \index{mot_a_indexe|)} à la fin.

Pour plus de détails, consulter 
ftp://ftp.ibp.fr/pub/TeX/CTAN/indexing/makeindex/doc/makeindex.ps


 Comment construire un index hiérarchique ?
------------------------------------------

Pour construire un index à plusieurs niveaux d'entrée, il faut
utiliser les commandes suivantes :
\index{niveau1} comme précédemment puis, pour faire apparaître
un sous-thème de ce niveau, on appellera :
\index{niveau1!niveau1.1}.

Exemple :
\index{Sport}
\index{Sport!Equitation}
\index{Sport!Equitation!Dressage}
\index{Sport!Equitation!Complet}
\index{Sport!Escalade}
% et pour L. Fournigault, un dernier exemple :
\index{Sport!Rugby}

Ces fonctions sont disponibles dans le package 'index' ou
(exclusif) 'french'.


 Quels sont les générateurs d'index ?
------------------------------------------

* makeindex, pour LaTeX sous Unix, Macintosh, MS-DOS ou OS/2 est
disponible sur CTAN dans respectivement /indexing/makeindex,
/systems/mac/macmakeindex.sit et dans les ditributions emTeX et
gTeX.

* texindex, fondé sur sed pour LaTeX sous Unix, est disponible
sur CTAN dans /support/texindex/.

* idxtex, pour LaTeX sous VMS, est disponible sur CTAN dans
/indexing/glo+idxtex/.

* texix pour TeX sur CMS et Macintosh. 


 Comment changer le style de certains mots indexés ?
------------------------------------------

Il faut utiliser la commande :
\index{mot_a_indexe@mot_reporte}

Exemple :
\index{Texte@\sc{Texte}} en LaTeX2.09
\index{Mot@\textsc{Mot}} en LaTeX2e


 Comment changer le style des pages de référence ?
------------------------------------------

Pour mettre en relief certains numéros de référence par rapport
à d'autres, on peut utiliser les macros suivantes : 
+ pour appeler un numéro de page en gras (LaTeX2e) :
	\newcommand{\idb}[1]{\textbf{#1}}
+ pour appeler un numéro de page en italique (LaTeX2.09) :
	\newcommand{\idi}[1]{\it{#1}}

La référence dans le texte se fait alors par :
\index{mot_a_indexe|idb}


 Comment rappeler certains mots dans un haut de page ?
------------------------------------------

Le package 'fancyhdr' (LaTeX2e) disponible sur CTAN dans
/macros/latex/contrib/supported/fancyhdr/ permet entre autres de
faire apparaître le premier mot de la page en haut à gauche de
la page et le dernier en haut à droite.


 Comment générer plusieurs index ?
------------------------------------------

* Il faut utiliser le package 'index' (et, si nécessaire,
charger le package 'french' après).

* Il existe également le package 'multind'.

Remarque : french (3.45) fonctionne bien avec "multind", par
	   contre  "multind" n'implémente pas toutes les
	   commandes d'index (notamment, le séparateur "!" ne
	   fonctionne pas).


 Qu'est ce que IdXTeX ?
------------------------------------------

IdXTeX programme écrit en C par R. Aurbach facilite la
génération d'index. Il est disponible sur CTAN dans
/indexing/glo+idxtex/.


 Qu'est ce que xindy ?
------------------------------------------

xindy est un système d'indexation. Il est disponible sur CTAN
dans /support/xindy/. Il est capable de gérer différentes
langues suivant différentes lettres (notamment les lettres
accentuées) et règles de tri. Il est paramètrable en fonction de
repères de localisation dans le document qui ne correspondent
pas forcément à un numéro de page ou de paragraphe prédéfini.

Pour plus de détails, consulter :
http://www.iti.informatik.th-darmstadt.de/xindy.


================================================================
[17] GLOSSAIRE
================================================================


 Quelles sont les commandes de base ?
------------------------------------------

La création d'un glossaire s'apparente à la création d'un index.
La commande d'appel du glossaire est \glossary, la commande de
construction du fichier .glo est \makeglossary. Les mots à
prendre en compte dans le glossaire sont repérés par la commande
\glossaryentry. 


 Qu'est ce que GloTeX ?
------------------------------------------

Le programme "GloTeX" de R. Aurbach, utilise une base de données
pour créer un glossaire. Il est donc possible d'utiliser la même
BD pour plusieurs documents, et ainsi d'avoir des glossaires
cohérents (les mêmes termes auront la même définition). GloTeX
est disponible sur CTAN dans /indexing/glo+idxtex/.


================================================================
[18] STYLES PREDEFINIS
================================================================


 Où trouver un style de thèse ?
------------------------------------------

* Le serveur loria présente quelques exemples de styles de
thèses : http://www.loria.fr/tex/. 

* Une doc sur la classe 'TheseCRIN' est disponible sur
http://www.loria.fr/tex/guide.html mais la classe elle même
n'est disponible qu'auprès de D. B. Roegel par mail
(roegel@loria.fr).

* Un autre exemple est disponible à l'université de Californie
ou sur CTAN dans /macros/latex209/contrib/ucthesis.

* utthesis est le package de l'université du Texas. Il est
disponible sur CTAN dans
/macros/latex/contrib/supported/utthesis/.

* Il existe également le package 'uwthesis' disponible sur CTAN
dans /macros/latex/contrib/supported/uwthesis/ de l'université
de Washington.


 Comment faire son CV en LaTeX ?
------------------------------------------

* Il existe deux packages : 'resume' et 'vita' disponibles sur
CTAN. 'vita' se trouve dans /macros/latex/contrib/other/vita/.

* Le package 'ESIEEcv' écrit par B. Bayart est disponible à
http://www.esiee.fr/~tex/Doc/ESIEEcv.sty. Il offre un certain
nombre d'environnements qui facilitent la mise en page type d'un
CV. 

Exemple :
\begin{rubrique}{Titre de la rubrique.}
   \begin{sousrubrique}
      \Date{deb-fin}
      \Duree{longue}
      \Lieu{Ca c'est passe ici}
      \Titre{Ce que j'ai fait}
      \Descr{Quelques details}
      \Apport{Ce que cela m'a apporte}
      \Apport{et ca aussi}
   \end{sousrubrique}
   \begin{sousrubrique}
      \Competence{Parlote}
      \Descr{Bonne maitrise}
   \end{sousrubrique}
\end{rubrique}

Remarque : la largeur de la première colonne peut être modifiée
	   par : \setlength{\largeurcolonne}{2.5cm}


 Où trouver un format de publication ?
------------------------------------------

Des classes et packages de format de publication dans les
revues scientifiques telles que IEEE, IFAC, SIAM, SIGGRAPH, etc,
sont disponibles sur CTAN.

Pour plus de renseignements sur les documents IEEE, consulter la
page http://www.ieee.org/pubs/authors.html qui donne des
références de style pour LaTeX2.09 et LaTeX2e.


 Où trouver un style de manuel de référence ?
------------------------------------------

Il existe la classe 'refman' disponible sur CTAN dans 
/macros/latex/contrib/supported/refman/. 


 Où trouver un style de poster ?
------------------------------------------

Il existe le package 'poster' disponible sur CTAN dans
/macros/generic/poster/. Il permet entre autres d'imprimer un
texte s'étalant sur plusieurs pages avec des parties communes
de manière à pouvoir recoller les morceaux après.


 Comment créer son propre style ?
------------------------------------------

Lorsque l'on veut modifier un style prédéfini pour l'adapter à
ses besoins, il est conseillé de créer son propre fichier .cls
dans lequel seront réunies les nouvelles commandes de mise en
page.

Ci-dessous un exemple de V. Henn pour définir son propre style
d'article :

%%% ----------- Debut de MonArticle.cls -----------
%%%
%%%  Format de style adapté de article.cls 
%%%
%%%  V.H. le 13 avril 1995
%%%

\NeedsTeXFormat{LaTeX2e}
\ProvidesClass{MonArticle}[1995/06/20 Article personnel]

%%%  Chargement de la classe article, avec transfert d'options
\PassOptionsToClass{a4paper}{article} % format a4paper par défaut
\DeclareOption*{\PassOptionsToClass{\CurrentOption}{article}}
\ProcessOptions

\LoadClass{article}

%%%  Chargement des Packages les plus courants
\RequirePackage{frbib}
\RequirePackage{general}
\RequirePackage{fuzzy}
\RequirePackage{bigcenter}
\RequirePackage{traffic}
\RequirePackage[dvips]{epsfig}
\RequirePackage{epic}
\RequirePackage{french}

%%%  Destination de l'article (proposé pour une conf, une revue...)

\def\Destination#1{\ifx{#1}{}\gdef\@Destination{}%
\else\gdef\@Destination{#1}\fi}

%%%   Destination vide par défaut

\Destination{}                            

%%%   Auteur par défaut, pour eviter de recopier a chaque fois

\author{Vincent \fsc{Henn}\thanks{Laboratoire d'ingénierie
circulation--transport, (Unité mixte \lsc{INRETS}--\lsc{ENTPE}),
109, Avenue Salvador Allende, Case 24, 
F--69675 \fsc{Bron} Cedex, {\tt henn@inrets.fr}.}}

%%% -----------fin de MonArticle.cls-----------

Utilisation :
\documentclass[a4paper,11pt]{MonArticle}

\title{Exemple d'article}
\Destination{IEEE}
\begin{document}
\maketitle

\chapter{Introduction}

Bla bla

\end{document}


================================================================
[19] CREATION DE TRANSPARENTS
================================================================


 Quels sont les packages et styles existants ?
------------------------------------------

* La classe 'seminar', de T. van Zandt, offre de nombreuses
possibilités et peu de modifications sont nécessaires pour
pouvoir l'utiliser avec des documents initialement écrits pour
"slide". seminar est disponible sur CTAN dans
/macros/latex/contrib/other/seminar/inputs/.

Note : A l'origine, seminar.sty était un package ; aujourd'hui,
       il a évolué en classe.

La lecture du cahier GUTenberg no 16 disponible en ligne à
l'URL :
http://www.univ-rennes1.fr/pub/gut/publications/publis.html
peut être utile. Voir également la page
http://tug.cs.umb.edu/applications/ qui présente notamment une
FAQ.

* Le package 'cours' permet également de réaliser des
transparents. Il permet, à partir d'un fichier source de créer
des transparents et de gérer un syllabus. Cela fonctionne très
bien sous  LaTeX2.09 et LaTeX2e (disponible sur les serveurs
CTAN).

* La classe 'foiltex', de J. Hafner, est pas mal du tout.
Elle permet de définir des entêtes et des pieds de page. Il a
été adapte à LaTeX2e et est disponible sur CTAN dans
/macros/latex/contrib/supported/foiltex/.

* La classe 'slides' est la classe de document fournie avec
LaTeX en remplacement de SliTeX, la solution proposée avec
LaTeX2.09 et disparue avec lui.


 Comment définir un contour pour des transparents ?
------------------------------------------
-----------------
* Avec le package 'seminar', disponible sur CTAN dans
/macros/latex/contrib/other/seminar/.

Exemple :
\slideframe[]{oval}

* Il existe également le package 'fancybox', disponible sur CTAN
dans /macros/latex/contrib/others/seminar/inputs/, qui définit
des fonctions telles que \shadowbox, \doublebox, \ovalbox,...

* On peut également utiliser une commande \special (cf.
paragraphe 29.8).

Exemple :
Cet exemple nécessite dvips et travaille sur un format a4. Il
permet d'obtenir un fond jaune clair avec un cadre noir à 1 cm
du bord de la feuille et de définir deux zones de texte en
vert.
\special{
 !userdict begin /bop-hook {gsave
 1 1 0.5 setrgbcolor clippath fill
 0 setlinecap 0 setlinejoin 2 setlinewidth 0 setgray
 /Times-Bold findfont 10 scalefont setfont newpath
 (Maurizio Loreti, 1996-02-25)
 dup stringwidth pop neg 510 add 25 moveto
 currentpoint 3 -1 roll 0 1 0 setrgbcolor show 0 setgray
 moveto -6 3 rmoveto 28 28 28 814 15 arcto
 28 814 567 814 15 arcto 79 814 lineto 6 -3 rmoveto
 0 1 0 setrgbcolor
 (CMS week at CERN)
 show 0 setgray 6 3 rmoveto 567 814 567 28 15 arcto
 567 28 516 28 15 arcto 516 28 lineto stroke grestore}
 def end
}


 Comment inclure des commentaires dans les transparents ?
------------------------------------------

Dans l'environnement slide, l'environnement note permet de
taper du texte qui sera imprimé sur une page séparée sans être
intégré dans un transparent. Cela permet à l'auteur de taper
ses notes personnelles sans qu'elles apparaîssent dans un
transparent.


 Comment modifier l'interligne sous seminar ?
------------------------------------------

Pour modifier l'espacement entre lignes à l'intérieur des 
paragraphes (sous seminar), il faut utiliser le paramètre
\slidestretch (cf. page 9 de la documentation de Seminar).

Exemple :
\documentclass[portrait]{seminar}
\begin{document}
\begin{slide*}
   Ceci est le texte de la d\'eclaration que j'ai \`a vous
   communiquer. Ceci est le texte de la d\'eclaration que j'ai
   \`a vous communiquer. Ceci est le texte de la d\'eclaration
   que j'ai \`a vous communiquer.
\end{slide*}

\def\slidestretch{0.5}
\begin{slide*}
   Ceci est le texte de la d\'eclaration que j'ai \`a vous
   communiquer. Ceci est le texte de la d\'eclaration que j'ai
   \`a vous communiquer. Ceci est le texte de la d\'eclaration
   que j'ai \`a vous communiquer.
\end{slide*}
\end{document}


 Comment définir des en-têtes et pieds de pages ?
------------------------------------------

Le package 'seminar' permet de définir des entêtes et des pieds
de pages.

Exemple :
\newpagestyle{filets}%
  {\hrulefill}% en haut
  {\hrulefill\tiny\theslide}% en bas
\slidepagestyle{filets}


 Comment modifier la taille du cadre d'un transparent ?
------------------------------------------

Sous seminar, pour un changement local, il suffit de préciser
les dimensions voules dans le \begin{slide} ou \begin{slide*}.

Pour un changement global, il faut modifier les variables
\slidewidth et \slideheight.

Exemple :
\documentclass[a4,portrait]{seminar}
\usepackage{semcolor}
\input seminar.bug

\begin{document}
% Cadre de la taille par defaut (en A4)
\begin{slide*}
  Texte ...
\end{slide*}

% Changement global de la taille du cadre
\addtolength{\slidewidth}{1cm}
\addtolength{\slideheight}{1cm}
\begin{slide*}
  Texte...
\end{slide*}

% Changement local de la taille du cadre
% [hauteur,largeur] en mode portrait
% [largeur,hauteur] en mode paysage
\begin{slide*}[15cm,10cm]
  Texte
\end{slide*}

% On revient a la taille précédente
\begin{slide*}
  Texte...
\end{slide*}
\end{document}


 Comment empêcher les figures de flotter ?
------------------------------------------

On peut forcer l'option par défaut à H :
\usepackage{float}
\newfloat{figure}{H}{lof} %Don't let them float by default
\newfloat{table}{H}{lot}
\floatname{figure}{\figurename}
\floatname{table}{\tablename}


 Comment gérer la couleur avec seminar ?
------------------------------------------

* L'exemple suivant montre comment paramétrer la couleur du
fond.

\documentclass[a4,portrait]{seminar}
\usepackage{semcolor}
\input seminar.bug

\newcommand{\SlideColours}[1]{%
\slideframe[\psset{fillcolor=#1,fillstyle=solid}]{scplain}}

\begin{document}
\SlideColours{blue}
	\begin{slide*}
	   Mon transparent avec un fond bleu.
	\end{slide*}
\end{document}

* Pour utiliser d'autres couleurs que celles prédéfinies (black,
blue, cyan, darkgray, gray, green, lightgray, magenta, red,
white, yellow), il faut le faire via l'une des commandes de
PSTricks définissant les couleurs (dans Seminar, par défaut la
couleur est en effet gérée via PSTricks).

Néanmoins, aujourd'hui il est conseillé d'utiliser plutôt
l'extension standard 'color' de LaTeX pour ce faire. Voici un
exemple :

\documentclass[a4,portrait]{seminar}

% Pour utiliser l'extension standard "color" avec Seminar
\usepackage[dvips]{pstcol}
\usepackage{semcolor}

\newcommand{\SlideColours}[1]{%
\slideframe[\psset{fillcolor=#1,fillstyle=solid}]{scplain}}

\definecolor{Bleu}{rgb}{0.,0.,1.}
\definecolor{Rose}{rgb}{1.,0.75,0.8}

\begin{document}
\SlideColours{Bleu}
	\begin{slide*}
	   Mon transparent avec un fond bleu.
	\end{slide*}

\SlideColours{Rose}
	\begin{slide*}
	   Mon transparent avec un fond rose.
	\end{slide*}
\end{document}

* On peut également composer un fond dégradé :
\documentclass[a4,portrait]{seminar}

% To use the standard "color" package with Seminar
\usepackage[dvips]{pstcol}
\usepackage{semcolor}

\usepackage{gradient}
\definecolor{Gold}{rgb}{1.,0.84,0.}
\slideframe[\psset{fillstyle=gradient,gradbegin=Gold,
	gradend=yellow, gradmidpoint=0.5}]{scplain}
\begin{document}
	\begin{slide*}
	    My slide with a nice gradient background.
	\end{slide*}
\end{document}


 Comment imprimer des transparents en miroir ?
------------------------------------------

Pour imprimer en miroir un document (pour faire de la
photocomposition), il existe un prologue PostScript pour DVIPS
que voici :

%%------ debut du fichier miroir.pro ------

% Pour imprimer des films transparents en miroir
% Pascal PERICHON - Distribution PC-TeXMF 2.6 - (c) 1997
userdict begin
/bop-hook{
   210 2.834 mul
   0 translate
   -1 1 scale
}def end

%%------- fin du fichier miroir.pro -------

 Il s'utilise de la façon suivante :
       dvips -hmiroir.pro

Remarque : N'oubliez pas de mettre ce fichier dans la path
	   HEADERS de dvips (ou dans un chemin spécifié dans 
	   config.ps pour l'inclusion des en-tête).


================================================================
[20] LETTRES ET MAILING
================================================================


 Comment structurer une lettre ?
------------------------------------------

* Voici un exemple simple :
\documentclass[11pt]{letter}

\name{expediteur}
\address{adresse de l'expediteur}
\signature{signature de l'expediteur}

\begin{document}
\begin{letter}{le destinataire}   
   \opening{Cher destinataire,}
   Texte...
   \closing{Formule de politesse}
   \ps{PS : j'ai oublie...} 
   \cc{Monsieur Truc.}
   \encl{pieces jointes}
\end{letter}
\end{document}

ATTENTION : si l'on ajoute le package 'french' la disposition
	    des champs utilisés change puisque les lettres
	    anglo-saxones ne respectent pas la même typographie
	    que les françaises.

* La classe 'lettre' de D. Megevand semble être un outil très
bien fait et adaptable à toutes les situations. Elle est
disponible par ftp à obsftp.unige.ch dans /pub/tex/macros/ ou
bien sur html://obswww.unige.ch/pub/tex/macros/.

* La classe 'fribrief' disponible sur CTAN dans
/macros/latex/contrib/supported/fribrief/ permet également de
mettre en page des lettres.


 Comment préparer un mailing ?
------------------------------------------

* Le package 'mailing', disponible sur CTAN dans
/macros/latex/contrib/supported/mailing/ permet de créer
plusieurs documents similaires avec des valeurs définies dans
des bases externes (fichiers ou autres). Il ne fonctionne
qu'avec la classe 'letter'. 

* Il existe également le package 'envlab', de B. Veytsman
(boris@plmsc.psu.edu),  disponible par ftp à ftp.dante.de ou sur
CTAN dans /macros/latex/contrib/supoorted/envlab/. Il est
aujourd'hui aux standards américains mais doit être étendu à
d'autres pays.

* Le package 'labels' est également disponible sur CTAN dans
/macros/latex/contrib/supported/labels/.

Exemple :
% LaTeX2e source file
\documentclass[12pt]{book}
\usepackage{labels}
\LabelCols=3
\LabelRows=11
\LeftBorder=8mm
\RightBorder=4mm
\TopBorder=2mm
\BottomBorder=4mm
%\LabelInfotrue

\begin{document}

%\footnotesize\sf
\numberoflabels=3

\addresslabel[\small\sf]
{Me, Myself \& I\\
SomePlace\\
SomeCompany\\
SomeStreet\\
SomeTown, SomeZip}

\end{document}



 Comment faire des références dans une lettre ?
------------------------------------------

* Dans la classe 'lettre' de D. Megevand, il existe les
commandes \nref, pour les références de l'expéditeur et \vref
pour celles du destinataire.

Exemple :
\documentstyle{lettre}
\signature{signature}

\begin{document}
\nref{text}
\vref{another text}
\begin{letter}
   \opening{Dear Paul,}
   Text....
   \closing{Sincerely yours}
   \encl{included document}
   \cc{copy to Totot}
\end{letter}
\end{document}

* Le package 'french' offre également les commandes \yourref et
\ourref.

Exemple :
\documentclass[a4]{letter}
\usepackage{french}
\signature{ma signature}

\begin{document}
\yourref{mon texte}
\ourref{mon autre texte}
\begin{letter}{le destinataire}   
   \opening{Cher Monsieur,}
   
   Texte...
   \closing{Salutations}
   \encl{pieces jointes}
   \cc{copie a M. Totot}
\end{letter}
\end{document}


================================================================
[21] SYMBOLES
================================================================


 Où trouver des symboles électroniques ?
------------------------------------------

* Le package 'epic' disponible sur CTAN permet d'accéder aux
symboles électroniques.

* Il y a également le package 'circ' disponible sur CTAN dans
/macros/generic/diagrams/circ/ qui fait ça.


 Comment dessiner des circuits électroniques ?
------------------------------------------

* Certains packages sont disponibles sur CTAN dans
/graphics/lcircuit/, /graphics/circuit_macros/ et
/macros/generic/diagrams/circ/.

* Il existe également des librairies xfig disponibles par ftp à
ftp.x.org dans /contrib/applications/drawing_tools/xfig/.


 Quelles sont les polices de symboles sous LaTeX ?
------------------------------------------

Il y en a plusieurs que l'on peut trouver sur CTAN dans /fonts/.

dancers	     Des petits bonshommes stylisés (dessins d'enfants) 
	     qui se trémoussent dans tous les sens... 
dingbat	     Symboles
goblin	     On dirait des personnages (étranges) stylisés...
hands	     Des grosses mains pointant vers la gauche ou la
	     droite 
iching	     Yi-King. Avec des symboles chinois bizarres...
karta	     Symboles bizarres (voir aussi niceframes.sty)
knot	     noeuds 
wasy2	     Symboles


 Comment obtenir les symboles mâle et femelle ?
------------------------------------------

* Les polices "astrosym", "cmastro", "wasy" ou "wasy2"
définissent ces caractères.

* Voir également le package 'genealogy' disponible sur CTAN dans
/fonts/genealogy/.


 Comment obtenir le symbole degré ?
------------------------------------------

* En fait le symbole "degré" ne devrait pas être confondu avec
un << petit o en exposant >>. C'est un caractère définit :
+ en codage OT1, par \char23
+ en codage  T1, par \char6

En LaTeX2e,
\DeclareTextSymbol{\degre}{T1}{6}
\DeclareTextSymbol{\degre}{OT1}{23}
définiront la commande \degre, que vous travailliez en codage
OT1 (défaut) ou T1 sélectionné par \usepackage[T1]{fontenc}

* Le package 'french' définit de cette manière la commande
\degre.

* Le package 'babel' définit lui \degre et \degres.

* Le package 'textcomp', de S. Rahtz, disponible sur CTAN dans
/fonts/psfonts/ts1/ basé sur les fontes DC 1.2 ou plus récentes,
disponibles sur CTAN dans /fonts/dc/ définit les commandes
\textdegree et \textcentigrade.

* Le package 'inputenc' disponible sur CTAN dans
/macros/latex/base/ fournit les commandes \textdegree et
\mathdegree.

* La solution qui consiste à mettre dans le préambule (LaTeX2e) :
\newcommand{\deg}{\ensuremath{^\circ}} ou
\newcommand{\deg}{\ensuremath{^\circ}\xspace} en utilisant le
package 'xspace' ou encore
\newcommand{\deg}{\(\mathsurround=0pt{}^\circ\)}
puis utiliser \deg en mode texte ou mathématique,
est donc à éviter.


 Où trouver des symboles astronomiques ?
------------------------------------------

Il existe le package 'astro' disponible sur CTAN dans
/fonts/astro/.


 Où trouver une police de symboles phonétiques ?
------------------------------------------

* Il existe le package 'phonetic' disponible sur CTAN dans
/fonts/phonetic/.

* La fonte ipa est disponible sur CTAN dans
/macros/latex/contrib/other/ipa/.

* La fonte tipa est également disponible dans /fonts/tipa/.


 Où trouver des opérateurs de logique floue ?
------------------------------------------

* Il existe le package 'logic' disponible sur CTAN dans
/fonts/logic/.

* V. Henn (henn@inrets.fr) propose ses propres définitions
(LaTeX2e). Sauvegarder les commandes suivantes dans un fichier
'fuzzy.sty'. Ce package nécessite les packages 'amsfonts' et
'xspace'.

ATTENTION : Le fichier ci-dessous est perso, ce qui signifie :
	    1. pas d'exhaustivité 
	    2. pas de garantie de solution optimale (en terme de
	    'pureté teX') 
	    3. pas de garantie de non bug 
	    4. ces opérateurs ne correspondent pas à une norme
	    pour la notation : ce sont des choix perso (qui
	    correspondent plus ou moins à l'usage)
%%%
%%%  Format de style permettant d'utiliser directement un certain
%%%  nombre  d'operateurs propres à la logique floue. Ces
%%%  opérateurs sont généralement surlignés d'un tilde... 
%%%
%%%    V.H., le 12 avril 1995
%%%

\RequirePackage{amsfonts}
\RequirePackage{xspace}

\message{Opérateurs flous}

\newcommand{\fmin}{\mathop{\flou{\min }}}
\newcommand{\fmax}{\mathop{\flou{\max }}}
\newcommand{\V}{\mathop{\mathrm V\kern 0pt}}
\newcommand{\ET}{\mathrel{\mathrm{ET}}}
\newcommand{\OU}{\mathrel{\mathrm{OU}}}
\newcommand{\Sim}{\mathop{\mathrm S \kern 0pt}}
\newcommand{\hauteur}{\mathop{\mathrm h\kern 0pt}}
\newcommand{\card}[1]{\| #1 \|}
\newcommand{\flou}[1]{\ensuremath{\widetilde{#1}}}
\newcommand{\R}{\ensuremath{\Bbb{R}}}
\newcommand{\cpp}{\ensuremath{\hbox{C}^{++}}\xspace}

\newcommand{\Poss}{\mathop{\Pi}}
%\newcommand{\Nec}{\mathop{{\cal{N}}}}
\newcommand{\Nec}{\mathop{\mathrm N\kern 0pt}}

\newcommand{\poss}{\operatoname{Poss}}
\newcommand{\nec}{\operatoname{Néc}}

\newcommand{\serie}[3]{% 
%  #1 -> le nom de la variable
%  #2 -> l'indice de début
%  #3 -> l'indice de fin
\ensuremath{{#1}_{#2},\ldots,{#1}_{#3}}}

\newcommand{\DP}{\fsc{Dubois} et \fsc{Prade}\xspace}


 Comment obtenir le symbole de paragraphe ?
------------------------------------------

Il suffit de taper \S.


 Comment obtenir le caractère 'registered' ?
------------------------------------------

* La commande \textregistered est disponible lorsqu'on utilise
une fonte qui le permet (TS1 ou 8r, p. ex. DC).

* Le package 'textcomp', de S. Rahtz, disponible sur CTAN dans
/fonts/psfonts/ts1/ définit également ce caractère.

* Voici d'autres définitions :
\def\Registered{\raisebox{1ex}{\kern-.1em\setbox\@tempboxa\hbox{%
\footnotesize$\bigcirc$}\hbox
to 0pt{\hbox
to\wd\@tempboxa{\hss\tiny\textrm{R}\hss}\hss}\box\@tempboxa\kern-.1
em}}

%% The following is from Ulick Stafford (ulick.stafford@nd.edu)

\def\registered{{\ooalignther information read 'usrguide.tex'
and {\hfil\raise .05ex\hbox{\scriptsize
R}\hfil\crcr\mathhexbox20D}}}

%% The following is from Maurice Dohmen (m.dohmen@cs.tudelft.nl)

\def\REgistered{{\ooaligne @ in macro names
{\hfil\raise.09ex\hbox{\tiny \sf R}\hfil\crcr\mathhexbox20D}}}

%% Alternatively, use the amssymb package, which provides a ready-
%% made circled R (J. Knappen, KNAPPEN@VKPMZD.kph.Uni-Mainz.DE)  
%% might work only in 2e.

\documentclass{article}
\usepackage{amssymb}
\begin{document}
\circledR
\end{document}

From: Boris A. Veytsman 

\newcommand{\reg}{\textsuperscript{\textcircled{\textsc r}}}


 Où trouver le symbole trade-mark ?
------------------------------------------

* Il suffit d'utiliser le package "pifont" puis de faire
\Pisymbol{psy}{212} ou encore \Pisymbol{psy}{228}.


 Comment obtenir un underscore ?
------------------------------------------

Il suffit de taper \_.


 Où trouver le symbole radioactif ?
------------------------------------------

Il est disponible dans le package 'karta' disponible sur CTAN.


 Comment obtenir le logo LaTeX ?
------------------------------------------

Il suffit d'appeler la commande \LaTeX.


 Comment obtenir le logo LateX2e ?
------------------------------------------

Le nouveau logo s'obtient par \LaTeXe.


 Où trouver des chiffres entourés ?
------------------------------------------

* Le package 'go' disponible sur CTAN dans /fonts/go propose
des chiffres blancs sur fond noir.

* Le package 'pifont' propose également des chiffres entourés.

* A défaut, on peut construire ses propres symboles à l'aide du
package  'overlay' ci-dessous.

%  Dans TTN  Vol3 Num 2 1994, Jeremy Gibbons propose une macro
% \overlay qui permet de créer des symboles en superposant
% d'autres symboles. Le symbole ainsi construit peut changer de 
% taille en fonction du style (display, script...) 
%   
%%% overlay.sty to overlay two symbols, respecting styles

\def\loverlay#1#2{\mathpalette\@overlay{{#1}{#2}{}{\hfil}}}
\def\overlay#1#2{\mathpalette\@overlay{{#1}{#2}{\hfil}{\hfil}}}
\def\roverlay#1#2{\mathpalette\@overlay{{#1}{#2}{\hfil}{}}}
	%  calls to \@overlay look like
	%	\overlay\textstyle{{x}{y}{\hfil}{\hfil}}
\def\@overlay#1#2{\@@overlay#1#2}
	% strip brackets from 2nd arg, to get
	%	\@@overlay\textstyle{x}{y}{\hfil}{\hfil}
\def\@@overlay#1#2#3#4#5{{%
	\def\overlaystyle{#1}%
	\setbox0=\hbox{\m@th$\overlaystyle#2$}%
	\setbox1=\hbox{\m@th$\overlaystyle#3$}%
	\ifdim \wd0<\wd1 \setbox2=\box1 \setbox1=\box0%
	\setbox0=\box2\fi % \box0 is now the wider box
	\rlap{\hbox to \wd0{#4\box1\relax#5}}\box0}}

%%% fin du style overlay

Utilisation : $\overlay{symbole1}{symbole2}$

Exemple (V. Henn) :
$ \overlay{\lower.6ex\hbox{$\overlaystyle\smile$}}
	{\raise.6ex\hbox{$\overlaystyle
	{\roverlay{\circ}{\cdot}}{\roverlay{\circ}{\cdot}}$}}
$


================================================================
[22] MUSIQUE
================================================================


 Comment écrire de la musique sous LaTeX ?
------------------------------------------

* Le package 'MuTeX', écrit par A. Steinbach et A. Schofer,
permet d'écrire de la musique. Il est disponible par ftp
anonyme à ftp.cs.ruu.nl dans pub/TEX/MuTeX.tar.Z.  

* Un autre package encore plus performant est 'MusiXTeX' de
D. Taupin (taupin@frups51.bitnet ou taupin@lps.u-psud.fr), R.
Mitchell et A. Egler. Il permet d'écrire des partitions pour
orchestres ou de la musique polyphonique. Il est disponible par
ftp anonyme à rsovax.ups.circe.fr dans .musictex ou à
hprib.lps.u-psud.fr dans /pub/musixtex/. ou encore  sur CTAN
dans /macros/musixtex/taupin/ ou /macros/musixtex/egler/. La
lecture du cahier GUTenberg 21 apporte beaucoup d'informations à
ce sujet. Voir http://www.ens.fr/gut/cahiers.html#cahiers21 ou
ftp://ftp.univ-rennes1.fr/pub/GUTenberg/publications/node7.html.
A voir également http://www.gmd.de/Misc/Music/ le site de W.
Icking avec une mine d'informations, en particulier des
pointeurs vers des partitions saisies en musixtex, etc.,

Il existe une FAQ disponible sur CTAN dans /macros/musictex/FAQ.

Remarque : le programme abc2mtex d'aide à l'utilisation de
	   MusiXteX peut également être utile, voir
	   http://www.gre.ac.uk/~c.walshaw/abc/


 Comment convertir du midifile en MusicTeX  ?
------------------------------------------

* Music TeX, de M. Beigbeder et J.J. Girardot permet la
conversion de midifiles en Music TeX.

* midi2tex permet également de traduire des fichiers de données
midi. Il est disponible sur CTAN dans /support/midi2tex/.


 Existe-t'il une liste de discussion de musique ?
------------------------------------------

Il existe une liste de discussion consacrée à l'écriture de
musique en TeX. Pour s'y inscrire, il faut envoyer un mail
"subscribe" à mutex-request@stolaf.edu.

Consulter également : http://www.gmd.de/Misc/Music/ ou
ftp://ftp.gmd.de/music/.


 Comment éditer un livre de chants ?
------------------------------------------

Il existe le package 'songbook' disponible sur CTAN dans
/macros/latex/contrib/supported/songbook. Voir la page
http://www.cyberus.ca/~crath/Misc/Songbook/index.html pour
quelques exemples.


================================================================
[23] CONVERSIONS DE FICHIERS
================================================================


 Comment générer un fichier .ps à partir d'un .dvi ?
------------------------------------------

* dvips, de T. Rokicki a l'avantage de savoir gérer les
"virtual fonts". Il est disponible par ftp anonyme à : 

+ pour unix au labrea.stanford.edu dans /pub/ ou sur CTAN dans
/dviware/dvips/. 

+ pour VMS sur CTAN dans /systems/vms/Alpha/ ou /systems/vms/VAX
suivant l'architecture utilisée. 

+ pour MSDOS au monu1.cc.monash.edu.au dans /pub/dvips54.zip ou
au shape.mps.ohio-state.edu dans /pub/msdos/dvips/dvips54.zip ou
sur CTAN dans /systems/msdos/drivers/dvips/. 

+ pour OS2 sur CTAN dans
/pub/tex/systems/msdos/dviware/dvips/dvips558.pc.

* dvitops, pour unix, msdos, vms et primos, de J. Clark, est
disponible sur CTAN dans /dviware/. dvitops ne gère pas les
fontes virtuelles.

* dvipsk, de K. Berry, est disponible sur CTAN dans /dviware/.


 Comment transformer du source C/C++ en LaTeX ?
------------------------------------------

* La distribution GUTenberg contient un package du nom de
'lgrind' (executable+lgrind.sty) qui, entre autres (C, C++,
Pascal, BASIC, Modula-2, Fortran, RATFOR, Yacc, PostScript,
Prolog, MLisp, Icon, LaTeX, Perl, CSH, Bourne Shell, assembler,
68000 assembler, asm68, VMS assembler, ISP, LDL, Linda, MODEL,
MatLab, Russell), formatte du code C++ en LaTeX. Ce programme
disponible sur CTAN permet à partir de code source C de générer
du code LaTeX respectant l'indentation. Ce package est également
disponible sur CTAN dans support/lgrind/.

* wflman disponible par ftp à ftp.keck.hawaii.edu
dans /pub/wlupton/wflman-2.2.2.tar.gz peut aussi être utile.

* Il existe également DOC++ qui peut créer une documentation
HTML ou LaTeX à partir des infos ajoutées dans les commentaires
du code C++. Pour plus de détails, voir :
http://www.ZIB-Berlin.DE/VisPar/doc++/doc++.html

* Le package 'tgrind', qui vient avec un .sty et une moulinette
permet de transformer un .c en .tex

* Le package 'c++2latex' sous license GNU est capable de
convertir des fichiers C, C++ et JAVA en LaTeX2e. Les lignes
peuvent être numérotées.

* Le package 'cprog' disponible sur CTAN permet de formater des
programmes C en TeX.

* Le package 'c2ltx', de M. Plugge
(plugge@biv7.sr.fh-mannheim.de), appartient à la famille de
convertisseurs cvt2ltx. Il numérote les lignes, traduit != en
$\neq$ , gère les commentaires, les en-tête de procédures, etc.
Il supporte plusieurs fichiers d'entrée et gère automatiquement
les changements de section et la génération d'index. Une
documentation est disponible par ftp à axp3.sr.fh-mannheim.de
dans /cvt2latex/cvt2ltx.zip. Ce package sera bientôt disponible
sur CTAN.


 Qu'est ce que le "Literate Programming" ?
------------------------------------------

Le "Literate Programming" est une approche de la programmation
qui met en valeur le fait qu'un programme doit pouvoir être lu
aussi bien par une personne que par un compilateur. Les outils
associés (disponibles dans les archives CTAN)  vont ainsi
permettre de générer des documents combinant du code source et
du texte (souvent au format TeX). Ensuite, la documentation et
le code source seront automatiquement extraits de ce document
unique. Pour plus d'information sur ces techniques, consulter :
http://www.ius.cs.cmu.edu/help/Programming/literate.html ou 
http://www.lifl.fr/~raczy/LitterateProgramming.html. Il existe
également une FAQ comp.programming.literate disponible sur CTAN
dans : help/comp.programming.literate_FAQ.

Remarque : À l'origine, D.E. Knuth a basé TeX  sur un système
	   de "programmation littéraire" nommé Web. Le code 
	   source en question était du  Pascal.  Aujourd'hui où 
	   l'on ne trouve quasiment plus  de compilateurs
	   Pascal,  il a fallu  écrire des outils  de  passage 
	   au  C.  D'où l'outil Web2C actuellement utilisé.  La 
	   plupart des distributions  Unix utilisent Web2c 
	   maintenu actuellement par Karl Berry.

* WEB, est le premier outil/langage de literate programming. Il
a été créé par D.E. Knuth, qui l'a d'ailleurs utilisé pour
écrire TeX et metafont. WEB est une surcouche du language
pascal. Il offre un indexage et des références croisées
automatiques pour les identificateurs et les procédures ainsi
qu'un pretty printing du code. Il est disponible sur CTAN dans
/web/tweb/.

* CWEB, de D.E. Knuth et S.Levy, est une réécriture de WEB en C.
Cette application est simple d'accès et marche très bien. Un
clône de ce produit est CWEBx qui est un peu plus riche (donc
plus compliqué mais sans excès). Il est disponible sur CTAN dans
/web/c_cpp/cweb/.

* FunnelWEB se présente en tant que surcouche de n'importe quel
langage, mais pour cette raison n'offre pas de "pretty-printing"
du code. Il en existe une variante (FunnelWEB-AC) pouvant
générer de la doc HTML, en plus de la doc TeX.

Il présente l'originalité par rapport aux autres d'avoir été
réécrit de toutes pièces, et non d'être une branche de
dévelopement séparée de CWEB.

Il semble par ailleurs que FunnelWEB soit le seul à pouvoir
donner à l'utilisateur le contrôle total à l'octet près de son
fichier de sortie (utile pour les Makefiles, par exemple), mais
peut paraître assez rigide à un utilisateur habitué à la
puissance de fweb.

Funnel WEB, est disponible sur CTAN dans /web/funnelweb.

* fweb (1.53), de J. Krommes, dérivé de CWEB, met à disposition
une FAQ http://www.arsc.sunyit.edu/fwebdocs/fweb.html et une
DOC  http://w3.pppl.gov/~krommes/fweb_toc.html 
Cette application est très complète, mais un peu difficile
d'accès. Elle permet notamment de travailler en fortran (77 ou
90), RATFOR (77 ou 90), C ou C++. Elle intègre entre autres un
système de gestion de macros très complet, "à la m4" (boucles,
arguments variables, etc.), ainsi qu'un traducteur de RATFOR en
Fortran, pour ceux qui n'auraient pas le compilateur adéquat...

Elle est l'une des rares à utiliser LaTeX plutôt que TeX en
standard, ce qui permet d'utiliser toute la puissance de
celui-ci... 

fweb est disponible sur CTAN dans /web/fweb/.

* noweb et nuweb sont plus légers, plus fiables et très
utilisés. Ils mettent à disposition des mécanismes pour générer
des documents LaTeX ou du code source. Ils sont disponibles sur
CTAN dans /web/noweb/ et /web/nuweb/. Pour plus de détails,
consulter : http://www.cs.purdue.edu/homes/nr/noweb.

* SpiderWEB, de N. Ramsey, dérivé de CWEB, est disponible sur
CTAN dans /web/spiderweb/. SpiderWEB permet de gérer la plupart
des langages, tant que les blancs et les tabs ne sont pas
critiques, tout en assurant le pretty-printing. Quelques heures
suffisent pour ajouter un nouveau langage, sans avoir à tout
recompiler. Un certain nombre de langages sont supportés dans la
distribution (C, ADA, awk, etc.)

* DOC++ est disponible à :
http://www.ZIB-Berlin.DE/VisPar/doc++/doc++.html
Il se base sur les infos dans les commentaires. Il structure
bien les classes, mais est un peu rigide au niveau des
possibilités. Génère du LaTeX ou du HTML

* wflman est disponible à :
ftp://ftp.keck.hawaii.edu/pub/wlupton/wflman-2.1.1.tar.Z
générateur automatique de doc au choix LaTeX, RTF, man et HTML


 Comment convertir du LaTeX en word ?
------------------------------------------

TeXport, de K-Talk permet de convertir vos fichier TeX
et LaTeX en documents WordPerfect ou Microsoft Word. Il traite
les styles de fontes, les notes de bas de page, les caractères
grecs, les mathématiques, les tableaux simples, les accents
européens, ainsi que les commandes \def et \renewcommand.

Ce logiciel est payant. Pour plus de détails, voir :
http://www.ktalk.com/.


 Comment convertir du word en LaTeX ?
------------------------------------------

* Il y a l'outil "Publishing Companion" de K-Talk (voir
http://www.ktalk.com).

* Wd2latex, pour MS-DOS, est disponible sur CTAN dans
/dviware/wd2latex/. Cet utilitaire paraît somme toutes assez
rudimentaire.


 Comment convertir du scribe en LaTeX ?
------------------------------------------

s2latex, de V. Jacobson, fait partie de la ditribution GUTenberg
pour Unix. Celle-ci est disponible par ftp à ftp.univ-rennes1.fr
dans /pub/GUTenberg/distribunix/.


 Comment convertir du WordPerfect en LaTeX ?
------------------------------------------

* wp2latex, de R.C. Houtepen, est disponible sur
wuarchive.wustl.edu dans ./mirrors/msdos/tex et sur CTAN dans
./support/wp2latex. wp2latex a été écrit pour PC en Turbo
Pascal. Il travaille à partir d'un document WordPerfect5.0. On
note quelques limitations concernant les indices, la table des
matières, les marges et les graphiques.

* wp2latex, de G. Geers (glenn@qed.physics.su.oz.au), a été
écrit en C. Il devrait être étendu à WordPerfect 5.1. La version
la plus récente est disponible par ftp à suphys.physics.su.oz.au
dans ./wp2latex ou sur CTAN dans /support/wp2latex/glenn/.


 Comment convertir du LaTeX en RTF ?
------------------------------------------

Le format RTF (Rich Text Format) correspond à un format ASCII
contenant des informations de formattage lisibles (entre autres)
par Word pour DOS, Word pour Windows et Word pour Macintosh.

* TeX2RTF, est disponible sur CTAN dans /support/tex2rtf/ ou
par ftp à ftp.aiai.ed.ac.uk dans /pub/packages/tex2rtf/.  Il
tourne sous SPARC Open Look, SPARC Motif et Windows3.1. 
TeX2RTF ne gère pas les formules mathématiques ni les tableaux
compliqués.

* LaTeX2rtf, de F. Dorner et A. Granzer, est disponible sur CTAN
dans /support/latex2rtf.


 Comment convertir du RTF en (La)TeX ?
------------------------------------------

* rtf2TeX (Unix), de R. Lupton (rhl@astro.princeton.edu), est
disponible sur CTAN dans /support/rtf2tex/. 

* rtf2latex (Unix), de E. Wechtl, est disponible sur CTAN dans
/support/rtf2latex/.

* w2latex, de J. Garmendia et J.L. Maltret, est disponible sur
CTAN dans /support/w2latex/.

* RTFLATEX, de D. Taupin, est capable de transcrire un fichier
RTF en LaTeX et de lui associer des fichiers de macros (.sty).
Toutefois, il ne traite pas encore les formules ni les tables.
RTFLATEX est disponible sur CTAN dans /support/rtflatex/ ou par
ftp à hprib.lps.u-psud.fr dans /pub/rtflatex/.


 Comment convertir du Excel en LaTeX ?
------------------------------------------

* Le package 'Tabular', de A. Gjestland, est disponible par ftp
à ftp.ssh.no dans pub/tabular. Il existe deux versions
Tabular4.xla pour Excel4 et Tabular5.xla pour Excel5. Une
documentation est également disponible : docu-tab.ps.Z. Vous
pouvez aussi consulter la page WEB
http://www.ssh.no/~ag/tabular/docu-tab.html.  Pour tous
renseignements, contacter l'auteur par e-mail : ag@hsh.no.

* Il existe égaleemtn l'utilitaire Excel2LaTeX disponible sur le
WEB à
http://www.informatik.uni-trier.de/CIP/marder/xl2latex.zip.


 Comment convertir du HTML en LaTeX ?
------------------------------------------

* http://www.w3.org/hypertext/WWW/Tools/html2things.html
propose un certain nombre d'outils de conversion à partir d'un
document html.

* html2latex, de N. Torkington, est disponible à
http://www.w3.org/hypertext/WWW/Tools/html2latex.html ou sur
CTAN dans /support/html2latex. Cet outil semble assez limité. Il
est basé sur le parser HTML de Mosaic (NCSA).

* h21, de J. Korpela, est disponible à
http://www.hut.fi/jkorpela/h2l/. Il est un peu plus complet que
le précédent mais doit encore être complété. Une documentation
est également prévue.

Note : HTML étant à la base du SGML, une autre solution (non
       testée) serait d'utiliser un traducteur SGML->LaTeX. 
       Le système 'linuxdoc-sgml' devrait pouvoir faire ça si on
       lui donne la DTD de la version d'HTML concernée et les
       règles de traduction. Pour plus de détails consulter le
       document
       /pub/linux/sunsite/utils/text/linuxdoc-sgml-*.tar.gz
       disponible par ftp à ftp.loria.fr

* techexplorer disponible sur
http://www.ics.raleigh.ibm.com/ics/techexp.h est un navigateur
Netscape pour Windows 95 et NT utilisable pour visualiser des
documents scientifiques et des expressions mathématiques. Il est
compatible avec les navigateurs Netscape 2 et 3 et avec Internet
Explorer. Pour plus de détails voir la mailing liste
techexplorer@LISTSERV.NODAK.EDU.

* Le package 'typehtml' disponible sur CTAN dans
/macros/latex/contrib/supported/carlisle/ permet de lire des
fichiers HTML2 et les mathématiques dans HTML3.0.


 Comment convertir du LaTeX en HTML ?
------------------------------------------

 Remarque : IDVI est un outil qui permet de présenter des
	    documents WWW à partir de sources LaTeX. IDVI 
	    est une implémentation Java d'un viewer dvi qui
	    permet d'afficher des pages LaTeX sur le WEB et non
	    plus un document HTML. Pour plus de détails,
	    consulter : http://www.geom.umn.edu/java/idvi/

* LaTeX2HTML pour Windows 95, NT et UNIX, de N. Drakos (Perl),
est disponible sur CTAN dans /support/latex2html/. Pour plus de
détails, consulter :
http://cbl.leeds.ac.uk/nikos/tex2html/doc/latex2html/.

Il existe également une liste de discussion latex2html à
laquelle on peut s'abonner en envoyant un mail
à majordomo@mcs.anl.gov

La nouvelle version de LaTeX2HTML est la 96.1. Une documentation
est disponible en ligne à l'irisa dans 
/usr/local/doc/LaTeX/latex2html.ps. Pour plus de détails,
consulter :
http://www-interne.irisa.fr/info/latex2html/node36.html

* Il est possible également d'utiliser hyperlatex, qui produit
des documents HTML plus jolis pour peu que votre source LaTeX
suive certaines conventions. Pour plus de détails, consulter :
http://graphics.postech.ac.kr/otfried/html/hyperlatex.html ou
http://www.cs.ruu.nl/pub/SGI/IPE/Hyperlatex-1.4pl2.tar.gz ou
ftp://ftp.cs.ruu.nl/pub/SGI/IPE/Hyperlatex-1.4pl2.tar.gz ou 
encore sur CTAN dans /support/hyperlatex/.

ATTENTION : cet utilitaire nécessite l'utilisation de latex2e et
	    de emacs.

* A voir également : OmniMark  sur http://www.omnimark.com.

* Il existe également ltx2x disponible sur CTAN dans /support/.


 Comment convertir un fichier dvi en ascii ?
------------------------------------------

* dvi2tty est disponible sur CTAN dans /dviware/dvi2tty/.

* dvispell de la distribution EmTeX permet également de
convertir un fichier dvi en fichier texte.


 Comment convertir du WEB en LaTeX ?
------------------------------------------

SchemeWEB, de J. Ramsdell, est disponible sur CTAN dans
/web/schemeweb/.


 Comment convertir du TeX en Framemaker ?
------------------------------------------

Framemaker dispose d'un filtre permettant d'importer des
fichiers TeX.


 Comment enlever les balises LaTeX d'un document ?
------------------------------------------

detex, disponible sur CTAN dans /support/detex/, permet
d'enlever toutes les commandes et séquences de contrôle LaTeX
d'un document. Il existe sous Unix, Doc et Mac.

ATTENTION : detex n'enlève que les macros, et pas leurs
	    arguments, ce qui donne des résultats pas
	    toujours très propres avec des documents LaTeX. 


 Comment convertir du SGML en (La)TeX ?
------------------------------------------

* sgmlspm, de D. Megginson, (Perl) est disponible à
http://www.uottawa.ca/dmeggins/SGMLSpm/sgmlspm.html.

* stil, de J. Schrod et C. Detig, (Common Lisp) est disponible
par ftp à ftp.th-darmstadt.de dans /pub/text/sgml/stil.

Ces deux outils s'appuient sur le parser nsgmls de J. Clark qui
traduit du SGML en ESIS.

* linuxdoc-sgml s'appuie aussi sur sgmls de J.Clark.

* sgml2tex, de P. Flynn, sur PC est disponible sur CTAN dans
/support/sgml2tex.


 Comment convertir du WinWord en LaTeX ?
------------------------------------------

* WINW2LTX, de A. Cottrell, est disponible sur CTAN dans
/support/winw2ltx/english/ww2 pour WinWord2, et
/support/winw2ltx/english/ww6 pour WinWord6. Il ne traite ni les
équations, ni les graphiques.


 Comment convertir un fichier 8 bits en fichier 7 bits ?
------------------------------------------

* B. Raichle a fourni un tel convertisseur parfaitement portable
qui se présente sous forme d'un programme ".tex" appelé
convert.tex. Celui-ci est disponible sur CTAN 
dans /language/typingtex/.

Il existe également une version modifiée par L. Siebenmann dans
/Convert-RaichleHacked.dir/ (interface modifiée, ajout de
quelques tables de transformation dont mac2dek.tbl ou pc2dek
pour la conversion du Macintosh ou du PC vers la syntaxe
classique de Knuth \'e, \c{c} etc.).

* L'outil Tower of Babel (avec option TeX activee), permet
également de réaliser de telles conversions. Il est disponible
par ftp à ftp.ibp.fr
dans /pub/mac/info-mac/text/tower-of-babel-13.hqx.

* recode est un programme unix (GNU) qui convertit tous types 
de fichiers texte.

Exemple :
recode latin1:applemac 
pour une conversion 7 bits vers 8 bits

* dans emacs, le package iso-cvt.el, peut convertir
automatiquement un fichier codé 7 bits en fichier codé 8 bits
(et réciproquement) lors de la lecture et de l'écriture. Pour
l'utiliser, ajouter (require 'iso-cvt) dans .emacs. Le fichier
sera alors en 7bits sur le disque.

* De même, il existe le package x-symbol pour (X)emacs. Pour
plus de détails, voir
http://brahms.fmi.uni-passau.de/~wedler/x-symbol.html.

* Il existe également les utilitaires kb7to8/8to7 distribués
avec 'french'.


 Comment convertir un fichier ChiWriter en TeX ?
------------------------------------------

chi2tex est disponible sur CTAN dans /support/chi2tex/. Il y a
plusieurs versions du convertisseur, selon la version de
ChiWriter à convertir.  A partir des versions 2 ou 3 cela ne
marchait que pour des fichiers très simples (pas plus d'un
niveau d'exposant et d'indice, etc). Il est probable que le
convertisseur de la version 4 marche beaucoup mieux : il fait
une analyse syntaxique des formules, alors que les versions 2 et
3  étaient purement graphiques.


 Où trouver une FAQ de convertisseurs (La)TeX/Traitement de texte PC ?
------------------------------------------

Une FAQ non officielle est disponible sur
http://www.kfa-juelich.de/isr/1/texconv.html. Elle est également
postée toutes les semaines dans comp.text.tex.


 Comment convertir une image en police metafont ?
------------------------------------------

C'est le programme bm2font qui permet de faire cela. Il est
disponible sur CTAN. La documentation est aussi sur le LaTeX
Navigator (http://www.loria.fr/tex).

Pour plus de détails sur metafont il existe la liste
metafont@ens.fr.


 Comment convertir un fichier postscript en ascii ?
------------------------------------------

pstotext est un outil Unix freeware qui permet d'extraire du
texte ascii d'un fichier postscript. Il utilise Ghostscript.
pstotext est disponible à
http://www.research.digital.com/SRC/virtualpaper/pstotext.html.

Sous Windows et OS/2 pstotext est accessible via l'outil GSview
de R Lang (version 2.0 minimum). Voir
http://www.cs.wisc.edu/~ghost/gsview/.

Pour DOS, pstotext est disponible à
http://www.cs.ruu.nl/~piet/software.html.


 Comment convertir un fichier pdf en ascii ?
------------------------------------------

L'outil pstotext présenté à la question précédente permet
également d'extraire du texte ascii d'un fichier pdf.


 Comment convertir du LaTeX en PDF ?
------------------------------------------

Remarque : il vaut nettement mieux demander à LaTeX de travailler
	   avec des polices PostScript, si l'on compte 
	   transformer le document en PDF par la suite. Si vous
	   utilisez  des polices à résolution fixe, le texte
	   aura une apparence  on ne peut plus laide sur votre
	   écran.

* L'utilitaire ps2pdf de ghostscript (version 4.01 au moins)
permet de convertir des fichiers postscript au format PDF
d'Adobe. Ghostscript version 4.02 est disponible par ftp à
ftp.cs.wisc.edu dans /pub/ghost/aladdin/. 
Sur mac, ps2pdf livré avec cmactex, nécessite l'installation de
psview disponible par ftp à ftp.ibp.fr dans
/pub/mac/local/tex/cmactex/.

* il existe une version préliminaire de pdftex (disponible pour
faire des tests). pdftex est un moteur tex qui permet de
produire directement un fichier PDF sans passer par DVI. Il
dispose donc en natif d'un support pour les objets graphiques et
les liens hypertextes. pdftex est disponible pour WinNT/Win95
par ftp à ftp.ese-metz.fr dans
/pub/TeX/private/pdftex.

T. Bouche a créé une page w3 de présentation de pdftex avec
trois fichiers pdf créés de cette façon :
http://www-fourier.ujf-grenoble.fr/~bouche/PDFtex/

ATTENTION : ceci est loin d'une version d'exploitation stable

* Adobe Acrobat Distiller permet avec des fontes simples de
générer des fichiers PDF à partir de postscripts. Dans ce cas,
il vaut mieux utiliser des fontes de type1 car les bitmaps de
tex sont très très mal rendues pas acrobat reader. Pour plus de
détails consulter www.adobe.com.


 Comment définir son propre format de sortie ?
------------------------------------------

ltx2x  disponible sur CTAN dans /support/ltx2x/ permet de
remplacer des commandes LaTeX par des commandes définies par
l'utilisateur.


================================================================
[24] PREVIEWERS ET VIEWERS
================================================================

Les viewers et previewers présentés ci-dessous peuvent permettre
entre autres de visualiser des documents .dvi et .ps.
Toutefois, ils ne sont pas attachés à LaTeX.


 Où trouver un previewer ?
------------------------------------------

* dvipage pour SunView est disponible par ftp anonyme à :
archive.cis.ohio-state.edu ou sur CTAN dans /dviware/dvipage/.

* xtex pour X Window System disponible par ftp anonyme à
aftp.cs.colorado.edu dans
./pub/cs/misc/SeeTeX/SeeTeX/SeeTeX-*.tar.Z ou sur CTAN dans
/dviware/seetex/.

* dviapollo pour stations Apollo disponible par ftp anonyme à
alabrea.stanford.edu dans ./pub/dviapollo.tar.Z ou sur CTAN dans
/dviware/dviapollo/.

* dvidis pour VAXstation sous VWS disponible par ftp anonyme à
src.doc.ic.ac.uk dans /packages/tex/dviware/dvidis ou sur CTAN
dans /dviware/dvidis/.

* xdvi pour X Window System disponible par ftp anonyme à
ftp.x.org dans ./contrib/xdvi.tar.Z ou sur CTAN dans
/dviware/xdvi.

* dviwin sous Windows est disponible sur CTAN.

* xdvik de K. Berry est disponible sur CTAN dans
/dviware/xdvik.

* dvitovdu pour Tektronix 4010 et autres terminaux sous Unix 
est disponible sur CTAN dans /dviware/dvitovdu.

* texsgi pour SGI sous Irix disponible par ftp anonyme à
ftp.brl.mil dans ./info-iris/tex ou sur CTAN dans 
/dviware/texsgi.


 Où trouver un viewer ?
------------------------------------------

* Ghostscript, Ghostview et GSview pour PC, Mac et station de
travail sont disponibles sur
http://www.cs.wisc.edu/~ghost/index.html.

* gview pour PC sous windows est disponible sur
http://cosimo.inf.unifi.it/Latex/gtex.html.

* viewps de T. Kiffe est inclu dans la distribution CMacTeX
disponible sur CTAN.

* TrueTeX est un viewer pour PC sous windows.


================================================================
[25] LOGICIELS DE DESSINS
================================================================


 Où trouver un logiciel de dessin ?
------------------------------------------

* Xfig est un logiciel de dessin disponible sur station de
travail Unix sous X Window System. Il permet via des menus de
manipuler des objets de façon interactive dans une fenêtre X
Window. Il nécessite une souris à trois boutons. Il est
disponible par ftp anonyme à ftp.x.org dans
./contrib/applications/drawing_tools/xfig ou sur CTAN dans
/graphics/xfig/. Son gros avantage est de permettre
l'utilisation de commande LaTeX (formules mathématiques). Il est
capable d'exporter une figure directement en format TeX ou de
générer de l'encapsulated postscript (si l'on veut des zones
grisées ou des choses de ce type, il faut passer par un fichier
postscript).

* PSfrag permet également de combiner du dessin et des textes
LaTeX. Il nécessite LaTeX2e et le package 'graphics'. Il est
disponible sur CTAN dans /macros/latex/contrib/supported/psfrag.

* TeXcad, permet de créer des fichiers de dessin. Le principe
est intéressant puisque cet outil permet de dessiner à l'aide de
la souris et de menus et lui le transforme en commandes LaTeX.
Il permet l'utilisation de la commande \special (cf. paragraphe 
29.8). TeXcad est disponible sur PC (il fait partie de la
distribution emTeX) et sous Unix et Linux (on peut le trouver
dans /systems/unix/linux/).

* Xy-pic permet de réaliser des diagrammes commutatifs, des
automates, et plein d'autres choses (cf. paragraphe 8.6).

* xgraphic offre une très bonne qualité de dessin. Il est
disponible à
http://blanche.polytechnique.fr/XGRAPHICS/xgraphics.html.

* tgif permet également de dessiner des figures. Il est
disponible à http://bourbon.cs.columbia.edu:8001/tgif/. Il peut
faire appel à différentes librairies (cf.
http://bourbon.cs.columbia.edu:8001/tgif/current.html) dont une
LaTeX qui permet entre autres de gérer des équations
mathématiques. 

* LaTeXcad sous Windows3.1 permet de convertir des figures en
commandes LaTeX. Il est disponible sur CTAN dans
CTAN : /systems/msdos/latexcad/.

* MetaPost que l'on peut trouver dans la distribution CMacTeX
peut produire des figures PostScript. Les commandes MetaPost
permettent entre autres d'obtenir directement des cercles, des
segments et des courbes de Bezier ; en outre, on peut nommer des
points en précisant leurs coordonnées ou en indiquant les
chemins à l'intersection desquels ils se trouvent. MetaPost
offre d'autres possibilités, bien décrites dans le manuel qui
l'accompagne.  

Pour plus de détails,
consulter http://www.loria.fr/~roegel/metapost.html.

* Pagedraw sous Windows 3.1 permet de générer des fichiers
postscript. Il est disponible sur CTAN dans :
/graphics/pagedraw/.

* Qfig est un logiciel de dessin sous DOS. Il génère des
fichiers PicTeX et EPIC. Il est disponible sur CTAN dans
/support/qfig/.


 Comment inclure des formules Latex dans Xfig ?
------------------------------------------

* Il suffit de mettre le flag special au texte, de taper son
texte LaTeX en incluant des commandes, d'exporter son fichier
(toto.fig par exemple) en deux parties : Combined PS/LaTeX (PS
part) Combined PS/LaTeX (LaTeX part) et de faire
\input{toto.pstex_t}, en ayant bien sûr chargé au préalable 
le package 'epsfig' par la commande \usepackage{epsfig} placée
dans le préambule. 

* On peut utiliser également tgif qui permet d'insérer du LaTex
dans les figures. Son fonctionnement est différent : le source
est compilé dans tgif même, et le résultat dvi est converti en
Postscript et apparait dans la figure. Le positionnement dans la
figure de  l'élément compilé est donc plus aisé, mais il est
assez contraignant de lancer LaTeX pour compiler chaque élément.


 Comment gérer différents formats de figures ?
------------------------------------------

* Sous Unix, l'outil ImageMagick permet de convertir de
nombreux formats d'image vers un autre. Il permet l'édition et
la manipulation d'images sous X window system. Pour plus de
renseignements, consulter
http://www.wizards.dupont.com/cristy/ImageMagick.html.
ImageMagick est disponible par ftp à ftp.wizards.dupont.com
dans /pub/ImageMagick/.

ImageMagick est également disponible pour VMS par ftp à
ftp.wizards.dupont.com dans /pub/ImageMagick/vms/ ou à
ftp.x.org dans contrib/applications/vms/.

* Transfig permet entre autres de convertir un fichier au format
fig en format postscript. Il est disponible sur CTAN dans
/graphics/transfig/.

* GraphicConverter sur mac (en anglais, français ou allemand)
est capable de lire pratiquement tous types de fichiers
graphiques de toutes origines et de le sauvegarder dans un autre
format dont EPSF. Le programme est shareware et est disponible
sur info-mac.

* tiff2ps (sous Unix), de S. Leffler, permet de convertir des
images au format tiff en format ps.

* GWSWIN11 ou Graphic Workshop pour Windows est disponible par
ftp à hprib.lps.u-psud.fr dans /pub/pc/utils/. Cet outil
(shareware à $40) permet de convertir du PCX, TIFF, JPEG, GIF,
BMP, etc. Il sait convertir des fichiers à niveaux de gris en
fichier tramés ("dithered" en anglais) utilisable pour
l'impression sur des imprimantes noir-et-blanc comme nos
imprimantes laser. Pour le tramage, il propose 7 algorithmes et
la doc de 60 pages donne des conseils. Son tramage paraît
meilleur que celui d'ImageIn ou des scanners usuels.

* Paint Shop Pro, outil pour PC, permet d'exporter des fichiers
en postscript qui pourront être appelés directement sous LaTeX.
Il est disponible à http://www.jasc.com/index.html en shareware
ou par ftp à ftp.ibp.fr.pub dans /pc/win95/desktop/psp311.zip ou
dans /pc/simtelnet/win3/Graphics/psp311.zip. Pour plus de
renseignements, consulter http://ns1.ionetwork.com/twr/psp.html. 

* L'utilitaire gif2eps disponible par ftp à qiclab.scn.rain.com
dans /pub/graphics/.


================================================================
[26] CORRECTEUR 
================================================================


 Où trouver un correcteur orthographique ?
------------------------------------------

* ispell est disponible sur CTAN dans /support/ispell/ ou à
ftp.inria.fr dans/gnu/. Il peut être associé au dictionnaire
français de M. Boyer disponible par ftp à ftp.inria.fr dans
/gnu/. ispell peut être associé à emacs (xemacs), NeXT. 
Pour plus de renseignements, consulter
http://ficus-www.cs.ucla.edu/ficus-members/geoff/ispell.html.

* Pour MS-DOS :
+ il existe amspell disponible sur CTAN dans
/support/amspell/ ou jspell dans /support/jspell/.
+ Micropell (commercial) peut être appelé depuis PCTeX pour
windows.

* Pour Macintosh, Excalibur est disponible sur CTAN dans
/systems/mac/support/excalibur/ avec plusieurs dictionnaires.

* Pour VMS, il existe vmspell  disponible sur CTAN dans
/support/vmspell/.

* On note également le mode ispell-minor-mode de emacs, qui est
capable de vérifier l'orthographe en ligne sans prendre en
compte les commandes LaTeX.

* La distribution Full VTeX [DOS/Win] inclut un correcteur
orthographique (américain, anglais, français, allemand,
hollandais et italien). Pour plus de détails, consulter :
http://www.micropress-inc.com.

* L'éditeur textpad sous Win3.1x et Win95 est capable de
supporter des textes LaTeX et comprend de nombreux
dictionnaires.

* Le dictionnaire de R. Cougnenc peut s'utiliser en mode shell
sous DOS comme sous Linux. Il contient 95 000 mots et 39 000
codes postaux, et permet  de vérifier très rapidement une
orthographe. Ce dictionnaire est disponible sur la page web de
D. Trystram :
http://www.starnet.fr/Homepages/dtrystram/index.html.


 Où trouver un vérificateur de syntaxe LaTeX ?
------------------------------------------

* TeX est un outil puissant disponible par ftp à ftp.tex.ac.uk.

* lacheck, disponible avec la distribution AUC-TeX (mode (La)TeX
pour emacs) est capable de vérifier la syntaxe LaTeX et de
détecter les erreurs les plus fréquentes.

* Le package 'syntonly' permet également de vérifier la syntaxe
LaTeX.

Utilisation :
 \documentclass[syntonly]{...} 
 
 * ChkTeX, de J. T. Berger Thielemann (jensthi@ifi.uio.no), dont
 la version v1.5 est disponible sur CTAN dans /support/chktex/.
 Il est capable de détecter des erreurs typographiques dans
 LaTeX. Cet outil permet de s'affranchir de certaines
 constructions LaTeX non intuitives. Les binaires pour UNIX,
 Amiga, MSDOS et OS/2 sont disponibles.

Quelques exemples de services :
+ pas d'espace ou assimilé après/avant une parenthèse
+ indique les espaces multiples qui ne sont pas équivalents à un
seul
+ gère la ponctuation en mode mathématique
+ indique les espaces avant une note de bas de page
+ gère les fichiers inclus 
+ détecte les blancs avant une référence au lieu de ~
+ vérifie les couples de parenthèses
+ gère l'espace après un passage en italique
+ ...

Cet outil est configurable. Il supporte LaTeX209 et LaTeX2e. 


================================================================
[27] ASSOCIATIONS
================================================================


 Qu'est ce que l'association GUTenberg ?
------------------------------------------

L'association GUTenberg (loi 1901) a pour objectifs de
regrouper les utilisateurs francophones de TeX, de favoriser
les échanges techniques permettant d'augmenter les possibilités
d'impression et de publications scientifiques et d'offrir à ses
adhérents un certain nombre de services dont des ditributions
TeX et LaTeX françisées. Le serveur d'archives de GUTenberg est
herbergé sur le serveur ftp anonyme du CRI à Rennes :
ftp.univ-rennes1.fr dans ./pub/GUTenberg/ ou sur le WEB à
http://www.univ-rennes1.fr/pub/GUTenberg.

GUTenberg publie la Lettre GUTenberg ainsi que les cahiers
GUTenberg.

Remarque : Le cahier 23 de GUTenberg est également une FAQ
	   LaTeX.

ATTENTION : la FAQ que vous êtes en train de lire est
	    indépendante de l'association GUTenberg !


 Qu'est ce que l'association TUG ?
------------------------------------------

Note : TUG (TeX Users Group) est une organisation internationale
dont un tiers des membre est européen. TUGboat est la lettre de
ce groupe. Pour plus de renseignements, contacter tug@tug.org
par mail ou consulter le site WEB http://www.tug.org/.


 Qu'est ce que l'association AsTeX ?
------------------------------------------

L'association AsTeX a comme objectif principal d'essayer de
faire du travail utile au plus grand nombre, dans le domaine des
logiciels scientifiques, et d'essayer de faire ce travail aussi
bien que les éditeurs privés, mais dans un esprit de service
public.

Cela passe par l'écriture d'utilitaires d'installation et de
configuration automatisés (pour que le débutant en TeX/LaTeX
n'ait pas à lire 1000 pages de docs disparates, en anglais de
surcroit, avant de pouvoir imprimer "Bonjour"), par l'écriture 
de docs raisonnablement bien rédigées et agréables à consulter,
par des distributions sur disquettes bien présentées. Cela passe
également par la traduction de docs originales dans un français
correct, etc.


================================================================
[28] FONTES
================================================================


 Que signifient les sigles T1, mf, fd etc. ?
------------------------------------------

Contribution de P. Pichaureau (ppichaur@grannus.u-strasbg.fr) :

Voici une mise au point rapide, histoire que vous compreniez 
de quoi ça parle.

En 1990, lors d'une reunion d'utilisateurs de TeX, à Cork, il a
été décidé de développer une table d'encodage particulière  pour
les fontes de TeX. Cette table contient des signes  diacritiques
et un certains nombre de symboles qui permettent de composer des
textes dans un bon nombre de langues européennes.

Ce codage s'appelle T1 (parfois Cork encoding), et l'ancien
codage de TeX s'appelle OT1 (Old T1). Les autres codages (U,
etc.) concernent des polices particulières et/ou des polices qui
ne  respectent ni T1 ni OT1.

Les fontes « standard » de TeX qui respectent ce codage
s'appellent fontes EC (pour european coding, il me semble). Les
fontes DC étaient une pré-version des fontes EC. La phase de
mise au point des fontes EC ayant duré un certain nombre
d'années, on s'y perd un peu.

Les fontes TC (text companion) contiennent un certain nombre de
caractères textuels utilisés en mode mathématique. À l'origine,
Knuth prenait ces caractères dans les polices textes normales,
mais cela pose des problèmes si vous essayez d'utiliser d'autres
polices de texte. C'est pour cela qu'on préfere maintenant les
mettre à part.

Ça c'est pour les pb de codage.

Pour les pk, mf, etc. je refuse de rentrer dans les détails,
mais voici un rapide aperçu de la question :

fichiers mf -> sources metafont. À partir de la, metafont génère
les fichiers tfm et pk.

fichiers tfm -> métriques des fontes. Contient la taille des
caractères, les corrections d'espacement éventuelles, etc. TeX a
imperativement  besoin de ces fichiers pour compiler un
document.

fichiers pk -> polices au format bitmap. C'est ce qui est
utilisé pour la prévisualisation et l'impression (meme en
postscript, et ne me demandez pas pourquoi !).

fichiers vf -> vf pour Virtual Font. Les fontes virtuelles ont
été mises au point par Knuth pour permettre vous permettre
d'utiliser des fontes de provenance diverses. Les fichiers vf
sont utiles lorsque vous essayez d'utiliser des fontes
postscripts.

fichiers fd -> description des fontes. Ça, c'est en rapport avec
NFSS. Bon, là, je suis obligé de m'étendre.

NFSS, c'est la manière dont latex2e sélectionne une fonte. LaTeX
ne le faisait pas assez proprement, alors on a fait le ménage.
Un fichier .fd dit a LaTeX2e quels sont les fichiers .mf à
utiliser pour telle police, dans telle taille, dans telle
famille, avec telle variation. C'est avec ce fichier que vous
dites a latex2e de prendre la version sans serif dans tel
fichier, la taille 9pt dans tel autre, et le gras dans ce
troisième fichier.

En tout état de cause, tfm et fd sont indispensables à la bonne
marche de latex2e. Les mf sont indispensables à la creation des
tfm et despk, et les pk sont indispensables à la visualisation
et à l'impression.


 Quels sont les attributs d'une fonte ?
------------------------------------------

Une fonte possède :
+ une famille (family) qui par défaut vaut cmr
	Autre valeurs : cmtt, cmss, cmdh, cmfib.

	La famille correspond à l'allure générale de la
	fonte. cmtt pour les fontes « machines à écrire »
	cmss pour les sans serifs, cmdh et cmfib pour
	respectivement les polices dunhill et fibonacci.

+ un codage (encoding) qui par défaut vaut OT1 (cf. question
28.1)
	Autres valeurs : T1, OML, OMS, ...

+ une série (series) de valeur m par défaut
	Les autres valeurs sont obtenues par une combinaison 
	de deux attributs : 
	- Un poids : ul, el, l, sl, sb, b, eb,ub,  
	- Une largeur : uc, ec, c, sc, m, sx, x, ex,ux

	Le poid correspond à la graisse de la fonte à savoir
	si elle sera plus ou moins grasse (de ul
	pour ultra-light à ub pour ultra-bold en passant
	par b pour bold).
	
	Exemple : bx
	
+ une forme (shape) dont la valeur par défaut est n
	Autres valeurs : n, it, sl, sc, ui, ol

	La forme correspond aux différents variations 
	de la fonte : n pour normal, it pour italique,
	sl pour penché (slanted), sc pour les petites
	capitales (small capital).

+ une taille (size) qui vaut par défaut 10pt. Lorsque deux
valeurs sont précisées, la première correspond effectivement à
la taille de la fonte et la seconde généralement supérieure
correspond aa la taille de l'interligne.


 Comment utiliser une fonte ?
------------------------------------------

* Pour déclarer une fonte par défaut dans un document, il
existe les commandes \familydefault, \encodingdefault,
\seriesdefault et \shapedefault.

Exemple :
\renewcommand{\familydefault}{cmtt}
dans le préambule d'un docuement permet de sélectionner la fonte
computer moderne de type machine à écrire pour tout le corps du
document.

* En LaTeX2e, un certain nombre de packages permettent de faire
appel à une fonte particulière.

Exemple :
\usepackage{times}
ou 
\usepackage{yinit}

* A un niveau plus bas, dans la création d'un style par
exemple, la sélection d'une fonte peut se faire de la manière
suivante :
\fontfamily{ccr}\fontencoding{T1}\fontseries{c}\fontshape{sl}% 
\fontsize{9}{11pt}\selectfont

ou encore :

\fontsize{14}{16pt}\usefont{OT1}{cmdh}{bc}{it}

* Pour définir une commande de changement de fonte, on peut
utiliser \DeclareFixedFont.

Exemple :
\DeclareFixedFont{\petitefonte}{\familydefault}%
{\encodingdefault}{\seriesdefault}{\shapedefault}{6pt}
\newcommand{\petit}{\petitefonte}


 Comment changer la forme d'une fonte ?
------------------------------------------

* En LaTeX2e, pour un changement ponctuel de fonte, un certain
nombre de commandes sont disponibles par défaut : 
+ \textrm pour romain 
+ \textsf pour sans sérif
+ \texttt pour du type machine à écrire
+ \textmd pour une série moyenne
+ \textbf pour du gras
+ \textup pour des lettres droites
+ \textit pour de l'italique
+ \textsl pour du penché
+ \textsc pour des petites capitales

Exemple :
Un \textbf{bel} arbre.
Un \textbf{\textit{très bel}} arbre.

* Pour changer la fonte de tout un paragraphe, on utilisera
plutôt les commandes \rmfamily, \sffamily, \bfseries, \itshape,
\slshape et \scshape.

Exemple :
{\sffamily blabla }
ou
\begin{itshape}
	blabla
\end{itshape}


 Comment changer la taille d'une fonte ?
------------------------------------------

* Il existe les commandes \tiny, \scriptsize, \footnotesize,
\small, \normalsize, \large, \Large, \LARGE, \huge, \Huge,
classées dans l'ordre croissant de taille. Ces commandes sont
prédéfinies en fonction de la classe de votre document.

* Plus globalement, vous pouvez utilisez la commande \fontsize
(cf. 28.3).


 Comment modifier la fonte des numéros de paragraphe ?
------------------------------------------

Pour modifier la fonte des numéros de paragraphe, il faut
redéfinir seccntformat.

Exemple (LaTeX2e) :
\makeatletter
\renewcommand\@seccntformat[1]{\texttt{\@nameuse{the#1}\quad}}
\makeatother


 Comment modifier la fonte du mode verbatim ?
------------------------------------------

* Le package 'verbatim' disponible sur CTAN dans
/macros/latex/packages/tools/ permet de faire cela. Il est alors
conseillé d'utiliser des fontes aux normes T1.

* L'environnement verbatimcmd du package 'moreverb', disponible
sur CTAN dans /macros/latex/contrib/other/misc/, permet de
garder les caractères backslash (\) et les accolades ({, et })
actifs. On peut donc entre autres opérer des changements de
fonte.

* Pour changer la taille de la fonte du mode verbatim, il faut
l'encapsuler dans des commandes de modification de taille.

Exemple :
\begin{small}
\begin{verbatim}
   Texte...
\end{verbatim}
\end{small}


 Comment réaliser des changements de fontes relatifs ?
------------------------------------------

Le package 'relsize' disponible sur CTAN dans
/macros/latex/contrib/other/misc/ permet de faire ce genre de
chose. Les commandes offertes sont du type :

+ \relsize{n} permet d'augmenter (n positif) ou de diminuer (n
négatif) la taille de la fonte par rapport à la taille courante.

Exemple :
n = -3 fait passer de \large à \footnotesize

+ \smaller == \relsize{-1}

+ \larger  == \relsize{1}

+ avec un argument, on trouve :
\textsmaller{text}, \textlarger{text}, \mathsmaller{A},
\mathlarger{B}


 Comment mettre en évidence une portion de texte ?
------------------------------------------

Utiliser la commande \emph. Cette commande est définie dans la
classe de votre document, et sert spécifiquement à mettre en
évidence un mot, une expression ou toute une  phrase. La plupart
du temps, elle se contente de mettre en italique votre texte.


================================================================
[29] DIVERS
================================================================


 A quoi sert la commande \par ?
------------------------------------------

Elle permet de marquer explicitement la fin d'un paragraphe pour
permettre à LaTeX de le mettre en page. L'utilité de cette
commande se justifie par le fait que LaTeX met en page un texte
par paragraphe. Elle est, le plus souvent, utilisée dans la
définition de macros.


 Comment commenter une partie d'un source LaTeX ?
------------------------------------------

* Sous LaTeX2.09, il suffit d'utiliser l'environnement
comment du package 'version'. 

Exemple :
\begin{comment}
   texte à ne pas faire apparaître
\end{comment}

* Un environnement comment est également défini dans le package
'verbatim' compatible LaTeX2.09 et LaTeX2e.

* Il est également possible de mettre des parties d'un document
source en commentaire grâce à l'environnement comment défini
dans le package 'comment' de V. Eijkhout. Ce package est 
disponible sur CTAN dans /macros/latex209/contrib/misc/. 

* Sous emacs, il y a une commande `comment-region' qui fait ça
très bien. Avec un argument, elle décommente.


 Comment utiliser LaTeX sur des petites machines ?
------------------------------------------

La capacité de TeX est limitée de manière interne. Cela signifie
que ce sont des variables de TeX qui fixent le nombre de
variables (et la taille mémoire) maximal que TeX s'autorisera à
utiliser. Si vous utilisez plus de variables que prévu, TeX
s'arrètera sur le message d'erreur :
   
   sorry, TeX capacity exceeded.

Tout ceci est destiné à << préserver >> votre machine, c'est à
dire à empêcher TeX d'utiliser la totalité de la mémoire de
la machine, ce qui conduit la plupart du temps à un plantage.

Sur une station, aucun problème, mais sur un micro-ordinateur,
les configurations peuvent-être nombreuses. C'est pourquoi le
TeX fournit en << standard >> sur les macs, par exemple, a ces
variables fixées assez bas (on suppose que le Mac a une toute
petite mémoire comparée à une station). Mais il existe
également un BigTeX, dont la seule différence avec le TeX
normal est qu'il est autorisé à utiliser plus de mémoire (les
variables en questions ont des valeurs plus importantes).

Du coup, si votre micro a une configuration plutôt musclée, vous
pouvez compiler des documents plus gros, plus complexes, etc...

Heureusement Kiffe a changé de stratégie et dans les futures
versions des outils les paramètres internes seront
configurables (en passant par ResEdit) et donc on décidera
soi-même la << bigness >> de son TeX (cf. distributions emTeX
et AsTeX). Pour changer les paramètres de compilation (pool size,
stack, font mem, main mem etc.) cf. fichier
emtex/doc/english/tex.doc, chapitre 6.

La  version 7.0 de web2c est configurable par le fichier
texmf.cnf.

Remarque : pour un gros document, il peut être utile de le
	   découper en chapitres à inclure par la commande 
	   \include (cf. paragraphe 9.2 ).


 Comment visualiser des paramètres ?
------------------------------------------

Pour visualiser des paramètres spécifiques, il existe :
* \showt\baselinestretch  (n'importe où ou presque, donc
affichage des éventuelles modifs, locales ou globales). Ou
pour avoir le résultat à la volée :
\message{Valeur de /baselinestretch = \the\baselinestretch}

* Il existe aussi la commande \showthe.

Exemple :
\showthe\baselineskip

* Il existe également le package 'showkeys' disponible sur CTAN
dans macros/latex/package/tools/. Il modifie les commandes
\label, \ref, \pageref, \cite et \bibitem de manière à
visualiser les paramètres internes utilisés.

* Pour visualiser les paramètres d'une page, il faut utiliser la
commande \layout du package 'layout'. Ce dernier est disponible
sur CTAN dans /macros/latex/packages/tools/.

* Pour visualiser des labels, il existe le package 'showlabels'
disponible sur CTAN dans
/macros/latex/contrib/supported/showlabels/. Les noms des labels
utilisés par \label ou par la numérotation automatique des
équations apparaîtront dans la marge du document. Ce package ne
définit pas de commande particulière, il suffit d'appeler
\usepackage{showlabels} dans le préambule du document. Dans le
cas où des packages AMS sont utilisés, showlabels doit être
appelé après. Les options [inner] ou [outer] ou [twocolumn]
permettent de définir dans quelle marge vont apparaître les
labels. 

Remarque : ce package n'est pas compatible avec les styles
	   multicolonnes ni avec l'option leqno.


 Comment visualiser des compteurs ?
------------------------------------------

Pour visualiser la valeur d'un compteur, il suffit de demander
\immediate\write{16}{\the\nomducompteur}
ou
\message{\the\nomducompteur}


 A quoi servent \makeatletter et \makeatother ?
------------------------------------------

Le caractère "@" est spécial pour LaTeX, et ne peut pas être
utilisé directement comme partie d'un nom de macro dans un
document LaTeX. Pour utiliser certaines macros internes de LaTeX
(celles qui contiennent un "@"), directement dans un document
(au lieu d'utiliser un fichier .sty), il faut les encapsuler
dans une macro dont le nom ne contient pas de "@". En outre,
cette macro doit impérativement être déclarée dans le
préambule du document, et être encadrée par les commandes
\makeatletter et \makeatother.

En fait ces commandes permettent d'indiquer à LaTeX que l'on
veut effectivement travailler sur ses commandes internes  en le
forçant à considérer le "@" (at) comme une lettre
(makeatletter).

ATTENTION : les commandes internes de LaTeX sont à manipuler
	    avec précaution.


 Comment numéroter les lignes d'un document ?
------------------------------------------

* Le package 'lineno' disponible sur CTAN dans
/macros/latex/contrib/supported/lineno/ permet de numéroter les
lignes d'un document.

* Il existe également le package 'numline' de M. Jaegermann et
J. Fortune, pour du texte essentiellement. Ce package  est
disponible sur CTAN dans
/macros/latex/contrib/supported/numline/.

* Pour numéroter les lignes d'un document inclu en mode
verbatim, il faut utiliser la commande \listinginput du package
'moreverb' disponible sur CTAN dans
/macros/latex/contrib/supported/moreverb/.


 A quoi sert la commande \special ?
------------------------------------------

La commande \special permet à TeX d'envoyer des instructions
particulières (non TeX) à un driver sans les interpréter. Les
instructions ainsi passées sont généralement dépendantes du
driver qui lui, saura les interprèter. L'utilisateur n'a en
général pas à se soucier de cette commande (\special), elle est
gérée par des packages tels que 'psfig' pour LaTeX2.09 ou
'graphics' et 'graphicx' pour LaTeX2e.

Remarque : \includegraphics est standard dans latex2e et
	   indépendant de la plateforme alors que le format de 
	   \special est dramatiquement dépendant de la
	   plateforme de travail. Seul, le \includegraphics
	   préserve l'aspect universel d'un document source et
	   donc l'esprit universel de Tex.


 Comment réaliser des calculs avec les variables LaTeX ?
------------------------------------------

* Pour pouvoir utiliser des expressions du type \truc * \machin
dans une macro, il faut charger le package 'calc' disponible sur
CTAN dans /macros/latex/contrib/supported/calc/.

* Une autre solution consiste à utiliser les commandes
\addtolength\dim\dim, \divide \dim by \dim, etc.

* Plus difficile à utiliser, il y a aussi le package 'trig'.


 Où trouver une fonte 9pt ?
------------------------------------------

* La classe de document 'amsart' disponible sur CTAN offre
l'option 9pt.

* De même les classes 'extarticle' et 'extreport' sont
disponibles à
http://www.informatik.uni-freiburg.de/~may/extsizes.html. 

* size9.clo de H. Steffani est disponible à 
http://www.tu-chemnitz.de/~hfst/size9.clo.


 Comment automatiser les compilations LaTeX ?
------------------------------------------

* latexmk est un script perl, disponible sur CTAN dans
/support/latexmk/, qui automatise la compilation d'un document
LaTeX. "latexmk fichier" lance latex/makeindex/bibtex autant de
fois qu'il le faut pour que le dvi soit à jour.

* latexn disponible sur CTAN dans support/latexn/ est un script
csh qui permet également d'automatiser les compilations latex.

* Le package AUC-TeX sous emacs conseille l'utilisateur sur la
prochaine opération à effectuer (latex, bibtex, makeindex, xdvi,
etc.), en fonction des dépendances entre les différents
fichiers.


 Comment obtenir des cadres gris ?
------------------------------------------

* Le package 'shade' disponible sur CTAN dans /macro/generic/
permet de tracer des cadres gris.

* Le package 'psboxit', disponible sur CTAN dans
/macros/latex2.09/contrib/misc/, permet d'obtenir du gris ou de
la couleur.


 Comment obtenir certaines abbréviations ?
------------------------------------------

* Pour obtenir des abbréviations dont certaines lettres doivent
être rehaussées et de taille réduite, on peut utiliser les
indices mathématiques.

Exemple :
M$^{\mathrm{lle}}$

* Le package 'babel' propose la commande \ieme pour les
quantièmes.

* Le package 'french' définit la commande \fup pour écrire
une chaîne de caractères en exposant, ainsi que les commandes
\ier \iers \iere \iere \ieme \iemes pour les quantièmes.

* On peut également définir une nouvelle commande \abbr dans le
préambule du document :
\newcommand{\abbr}[1]{\raisebox{1ex}{\footnotesize #1}}
Cette solution a l'avantage d'éviter d'utiliser le mode
mathématique improprement.

* Il existe aussi la commande \textsuperscript qui permet
d'obtenir un exposant sans passer en mode mathématique.


 Comment gérer les espaces après une macro ?
------------------------------------------

* Pour forcer LaTeX à introduire un espace après une macro il
suffit de faire suivre son appel d'un backslash (\).

Exemple :
texte \macro\ texte \macro, texte

* Pour que LaTeX gère lui même les espaces, il suffit d'utiliser
systématiquement les caractères {}.

Exemple :
texte \macro{} texte\macro{}, texte

* On peut également définir une macro TeX \keepspace :
\def\keepspace{\ifnum\catcode`\ =10
  \let\next\keepspacebis \else \let\next\relax \fi
  \next}
\def \keepspacebis{\obeyspaces
  \afterassignment\keepspaceaux\let\next= }
{\obeyspaces%
\gdef\keepspaceaux{%
\ifx \next\space\let\next\ignorespaces\fi%
\catcode`\  =10\relax\next}}
%%%%
puis l'utiliser dans une autre définition  de macro.

Exemple :
\newcommand{\macro}{\textit{blablabla}\keepspace}

* Le package 'xspace' intègre ces définitions qui lui permette
de gérer automatiquement l'espace après une macro suivant le
contexte.

Exemple :
\newcommand{\macro}{aaaaa\xspace} dans le préambule 
et
\macro bbbb dans le texte
donne 
aaaaa bbbb à l'impression,
alors que
\macro, bbbb
donne 
aaaaa, bbbb à l'impression.

Note : à noter que le package xspace est conforme aux règles 
       de typographie anglo-saxones donc la gestion des 
       caractères de ponctuation composés tels que : où ; ne
       sera pas correcte en français. La solution consiste alors
       à utiliser le blanc insécable qui de toutes façons est
       recommandé dans ce cas : \macro~: blabla. On peut
       également se créer son propre package 'fxspace'.


 Où trouver les notices d'utilisation des packages ?
------------------------------------------

Généralement la notice d'utilisation d'un package est incluse
dans le package lui même (fichier .dtx). Il suffit alors de
compiler ce fichier. Il existe également le fichier doc
disponible sur CTAN dans /macros/latex/base/ qui décrit comment
exploiter ces notices.


 Comment obtenir des listes d'objets flottants ?
------------------------------------------

Certaines sont disponibles par défaut, il s'agit des listes de
figures et de tables, il suffit alors d'appeler les commandes 
\listoffigures et \listoftables. D'autres sont liées à certains
packages. Par exemple, le package 'algorithms' de P. Williams
offre la commande \listofalgorithms.

Exemple :
\documentclass{report}
\usepackage{algorithm,algorithmic}

\begin{document}

\listofalgorithms

\chapter{toto}

\begin{algorithm}[h!tp]
   \begin{algorithmic}[2]
      \REQUIRE $T_1$ et $T_2$ doivent \^etre des tableaux de
      valeurs additionnables, et doivent \^etre de la m\^eme
      longueur $n$. \ENSURE $\forall 0\leq k< n \;\;
      Result[k]=T_1[k]+T_2[k]$.
      \FOR{$i\rightarrow 0$ to $n$}
      \STATE $Result[i]\rightarrow T_1[i]+T_2[i]$
      \ENDFOR
   \end{algorithmic}
   \caption{Somme terme \`a terme de deux tableaux3}
   \label{algo-ex-3}
\end{algorithm}
[recopie en trois exemplaires]
\end{document}


 Comment connaître les versions des fichiers utilisés
------------------------------------------

lors d'une compilation ?
------------------------
Il suffit d'utiliser la commande \listfiles.

Exemple :
\documentclass{report}
\listfiles
\usepackage{french}
\usepackage{graphics}

\begin{document}
   texte  ...
\end{document}


 Pourquoi certaines commandes sont elles indépendantes ?
------------------------------------------

Les commandes LaTeX de type verbatim sont particulières du fait
qu'elles changent les codes de catégorie de leurs arguments. De
ce fait, elles ne sont plus compatibles avec d'autres commandes
et ne doivent pas être appelées par ces dernières. Certains
caractères spéciaux doivent alors être obtenus par \texttt.

Exemple :
\texttt{\char`\\}

On peut également utiliser l'environement lrbox de LaTeX2e pour
sauver le texte qui doit apparaître en verbatim dans une boîte 
et ensuite utiliser cette boîte.


 Comment installer un package ?
------------------------------------------

Lorsqu'il s'agit d'un package conçu pour LaTeX2e et utilisant
les procédures et outils définis dans ce cadre, c'est simple :
un package est fourni avec deux (ou plus, pour les packages plus
complexes) fichiers, package.ins et package.dtx

Il faut commencer par exécuter "latex package.ins" pour créer
le(s) fichier(s) de commandes lui(eux)-même(s) (typiquement
package.sty) puis "latex package.dtx" pour générer la
documentation.

Ces fichiers doivent ensuite être déclarés dans une variable
d'environnement afin d'être rendus visibles. 

Exemple (UNIX) :
TEXINPUTS=.:/usr/local/TEX/texmf/tex:/users/ADMIN/NG/LATEX:
/users/home/kluth/Section-equit/Modele

Deux remarques : 
* on ne peut évidemment générer la documentation en premier,
parce qu'elle aura toujours besoin du fichiers des commandes ; 
* si l'on ne veut obtenir que la documentation <>,
il faut ajouter la ligne "\OnlyDescription" dans le fichier
package.dtx, avant la ligne "\DocInput{package.dtx}". Sinon on
obtient aussi le listage commenté du code, ce dont on peut ne
pas vouloir se soucier et qui peut être volumineux pour les
packages importants.

Néanmoins, il est clair que ce mode d'organisation et de
distribution n'est que le mode encouragé par l'équipe de
développement de LaTeX et n'est en rien une obligation. Un bon
nombre de packages disponibles ne le suit d'ailleurs pas (!),
pour diverses raisons (packages qui se veulent utilisables aussi
bien avec Plain TeX qu'avec LaTeX, << vieux >> packages écrits
pour LaTeX209, packages écrits par des personnes qui veulent
faire << comme ça leur plait >>, etc.)... Dans ces différents
cas, il faut << aller y voir de plus près >> pour comprendre
l'organisation choisie.

C'est justement le cas pour slashbox (!) : un seul fichier .sty
est fourni. Il faut l'éditer pour extraire le fichier de
documentation qui se trouve après la ligne 80 "\endinput".


 Comment générer des codes barres ?
------------------------------------------

* Il existe les packages 'ean' et 'code128' disponibles sur CTAN
dans /macros/generic/ean/ et /macros/generic/code128/.

* De même les packages 'envelopes' et 'labels' sont disponibles
sur CTAN dans /macros/latex/contrib/supported/envlab/.

* On peut également trouver le package 'barcodes' sur CTAN dans
/fonts/barcodes/.


 Comment gérer des abbréviations ?
------------------------------------------

* Le package 'abbrevs' disponible sur CTAN dans
/macros/latex/contrib/supported/monster/ permet automatiquement
de remplacer des abbréviations par leur formule développée
notamment lors d'un premier appel.

* Le package 'acronym' disponible sur CTAN dans
/macros/latex/contrib/supported/acronym/ permet de s'assurer que
tous les acronymes utilisés dans un document apparaîssent au
moins une fois sous une forme développée.


 Comment imprimer un fichier postscript sur une imprimante non postscript ?
------------------------------------------

alladin est un excellent programme qui permet d'imprimer du
postscript sur à peu près n'importe quelle imprimante. Cet outil
est disponible par ftp à ftp.ibp.fr dans /pub/ghost/aladdin/.

Remarque : on ne peut pas dire que ce soit d'une très grande
	   facilité d'installation, mais une fois que ce mauvais
	   moment est passé, on ne peut plus s'en passer :-)


 Comment surcharger une commande déjà existante ?
------------------------------------------

* Pour pouvoir ajouter des définitions sur une commande
prédéfinie sans la réécrire complètement, il faut utiliser la
commande \let.

Exemple :
\let\standardsection=\section
\def\section{\newpage\standardsection}

* Il existe aussi la commande expandafter.

Exemples :
\toks0 \expandafter{\section}
\edef\section{\noexpand\newpage\the\toks0 }

\expandafter\def\expandafter\section\expandafter{\expandafter%
\newpage\section}

* Le package 'babel' propose également la commande \addto.


 Comment reporter l'exécution d'une commande à la fin d'une page ?
------------------------------------------

Il existe pour cela le package 'afterpage' disponible sur CTAN
dans /macros/latex/packages/tools/.


 Comment définir des scripts interactifs ?
------------------------------------------

Le package 'dialogl' disponible sur CTAN dans
/macros/latex/contrib/supported/dialogl/ permet de définir des
scripts LaTeX interactifs.


 Comment identifier une version provisoire ?
------------------------------------------

* Le package 'draftcopy' disponible sur CTAN dans
/macros/latex/contrib/supported/draftcopy/ permet par exemple
d'écrire le mot DRAFT en grisé en diagonale sur toutes ou
certaines pages d'un document. Il permet également de faire
moins voyant.

Exemple :
Ca donne à peu près ça mais en plus beau !
-------
|    T|
|   F |
|  A  |
| R   |
|D    |
-------

* Le package 'drafthead' également disponible sur CTAN dans
/macros/latex209/contrib/misc/ permet lui de n'agir que sur les
en-têtes des documents.

* On peut également utiliser la macro :
\special{
! userdict begin /bop-hook{
stroke
gsave 240 100 translate
65 rotate /Times-Roman findfont 220 scalefont setfont
0 0 moveto 0.9 setgray (DRAFT) show grestore
}def end}
dans le préambule de son document.


 Comment obtenir des caractères barrés ?
------------------------------------------

* Le package 'ulem' disponible sur CTAN dans
/macros/latex/contrib/other/misc/ permet de
barrer des caractères.

Exemple :
\sout{je barre tout}


 A quoi sert la commande \ensuremath ?
------------------------------------------

La commande \ensuremath assure que son argument sera imprimé en
mode mathématique quel que soit le mode courant.

Exemple :
dans le préambule :
\newcommand{\mc}{\ensuremath{(\alpha, \beta)}}
puis dans le texte :
Le couple \mc définit par $\mc = x+y, x-y$, ...


 A quoi sert la commande \newcommand ?
------------------------------------------

La commande \newcommand permet à l'utilisateur de définir ses
propres commandes voire de renommer des commandes LaTeX. Elle
s'utilise dans le préambule du document.

Exemple :
\newcommand{\be}{\begin{enumerate}}
\newcommand{\ee}{\end{enumerate}}

Remarque : La commande \renewcommand permet de redéfinir des
	   commandes déjà existantes.
	   
	   La commande \providecommand permet de ne prendre en
	   compte la nouvelle commande que si elle n'existe pas
	   déjà.


 Où trouver des hirondelles ?
------------------------------------------

On peut trouver la package 'cropmark' sur CTAN dans
/macros/generic/.


 Comment tracer des lignes épaisses ?
------------------------------------------

On peut mettre la définition suivante dans le préambule du
document :
\def\ligne#1{\leaders\hrule height #1\linethickness \hfill}
puis utiliser :
\ligne{5}
dans le texte.


 Comment imprimer le caractère ~ ?
------------------------------------------

* Il est possible de passer par le mode verbatim via la commande
\verb$~$.

* \~{} permet également d'obtenir un tilde.

* Une autre solution consiste à utiliser $\sim$.

* LaTeX2e fournit la commande \textasciitilde sous fonte T1.
Sous OT1, il faut définir
\ProvideTextCommand{\textasciitilde}{OT1}{\~{}}.

* Autre solution, utiliser : \char`\~


 Comment visualiser tous les caractères d'une fonte ?
------------------------------------------

Le plus simple consiste à utiliser :
latex nfssfont
nfssfont.tex est un fichier disponible sur CTAN.

Il faut ensuite donner le nom de la fonte à examiner par
exemple cmr10 puis construire la table des caractères en
utilisant la commande \table. Cela permet de construire un
fichier .dvi contenant les codes de tous les caractères.

L'appel d'un symbole se fait alors via la commande \symbol{x} où
x est le code associé au caractère voulu (c'est un entier
compris entre 0 et 255).

On peut également utiliser \char" ou \char^^
ou encore \char mais ses commandes sont moins robustes.


 A quoi sert % dans les macros ?
------------------------------------------

Le % en fin de ligne d'une macro permet d'indiquer à latex que
la définition qu'il est en train de lire n'est pas finie.


 Comment inclure l'heure dans un document ?
------------------------------------------

* La macro décrite ci-dessous, à inclure dans le préambule de
votre document définit la commande \timenow :

\makeatletter
\def\timenow{\@tempcnta\time
  \@tempcntb\@tempcnta
  \divide\@tempcntb60
  \ifnum10>\@tempcntb0\fi\number\@tempcntb
  \multiply\@tempcntb60
  \advance\@tempcnta-\@tempcntb
  :\ifnum10>\@tempcnta0\fi\number\@tempcnta}
\makeatother

* En ajoutant la macro suivante de H. Hanche-Olsen dans le
préambule de son document, on peut via la commande
\isodayandtime obtenir la date et l'heure courante :
\begingroup
\count0=\time \divide\count0by60 % Hour
\count2=\count0 \multiply\count2by-60 \advance\count2by\time % Min
\def\2#1{\ifnum#1<10 0\fi\the#1}
\xdef\isodayandtime{\the\year-\2\month-\2\day\space\2{\count0}:%
\2{\count2}}
\endgroup

Autre exemple d'utilisation (qui nécessite dvips) :
\special{!userdict begin /bop-hook
 {gsave -90 rotate -780 560 moveto 1.0 0.7 0.7 setrgbcolor 
 % 0.8 setgray 
 106 45 {dup mul exch dup mul add 1.0 exch sub} setscreen
 /Times-Roman findfont 25 scalefont setfont (DRAFT
 \isodayandtime)  show grestore} def}

* On peut également utiliser la macro \time qui donne l'heure
en minutes. Pour afficher l'heure en heures et minutes, il faut
les calculer puis les afficher avec la macro \the.

Exemple :
\documentclass{article}
\begin{document}
This run started at \the\time\ minutes after midnight on \today.
\end{document}


 Comment compter le nombre de mots d'un fichier ?
------------------------------------------

A priori, il n'y a pas de solution simple à ce problème :
* A partir du fichier source .tex, il faut enlever toutes les
commandes LaTeX avec un utilitaire tel que detex (disponible sur
CTAN dans /support/detex/) puis utiliser un autre utilitaire tel
que wc sous UNIX.

* On peut également travailler à partir du fichier .dvi en
appliquant dvitty (ou dvi2tty).

ATTENTION : aucune de ces méthodes n'est idéale comme l'indique
	    l'essai comparatif de T. Bouche :
	    	detex -i grfguide.tex | wc -w	donne 4420
	    	dvitty grfguide.dvi |wc -w	donne 4524
	    	wc -w grfguide.tex		donne 4539 
	    	ps2ascii grfguide.ps |wc -w	donne 5066


 Comment rendre inactif un caractère ?
------------------------------------------

La commande \string permet de désactiver un caractère rendu
actif par une macro.

Exemple :
En français le : est rendu actif pour que LaTeX gère lui même
l'espace à mettre avant. En revanche, dans une URL ou une
adresse mail cet espace est superflu. On utilise donc :
mailto\string:moi.meme@chez.mon.site


 Comment utiliser le mode verbatim dans une commande ?
------------------------------------------

LaTeX2e fournit l'environnement lrbox. 

Exemple :
\documentclass{article}
\usepackage{alltt}
\newsavebox{\inverbatim}

\begin{document}
\begin{lrbox}{\inverbatim}%
\noindent\begin{minipage}{\linewidth}
\begin{alltt}
                           1
 -8 + 7 Sqrt[2] ArcTanh[-------]
                        Sqrt[2]
\end{alltt}
\end{minipage}%
\end{lrbox}% box

\noindent
\usebox{\inverbatim}%rend le vide
\end{document}


 Comment redéfinir la commande \year ?
------------------------------------------

Pour ne faire apparaître que deux chiffres au lieu de quatre, il
faut utiliser :
\newcommand{\ignoretwo}[2]{}
\newcommand{\shortyear}{\expandafter\ignoretwo\the\year}
dans le préambule du document.


 Qu'est ce que la magnification ?
------------------------------------------

* La commande magnification permet de faire un zoom d'un
document.

Exemple :
\mag=1440
\documentclass{...}

* On peut également travailler sur le fichier postscript via
dvips avec l'option -x nombre où nombre vaut par exemple 1200
pour un agrandissement de 20%.


 Comment sont gérés les postscripts dans LaTeX ?
------------------------------------------

P. Perichon :
TeX/LaTeX + DVIPS procèdent en 2 temps : 
* Lorsque l'on a une figure PostScript (EPS : Encapsulated
PostScrit), on met les commandes nécessaires dans son source
(fichier .tex) pour dire à TeX/LaTeX que l'on veut cette figure
à telle place dans son document avec telles dimensions (en
incluant la bonne extension : graphicx, psfig, epsfig) . 

Lors de la phase d'interprètation du code TeX/LaTeX,
l'interprète TeX/LaTeX va seulement consulter dans le fichier
externe contenant la figure les dimensions/proportions de
l'image (les fameuses bounding box chères au PostScript
Encapsulé). 

En fin de course, dans le fichier DVI on aura le nom et les
dimensions de l'image mais PAS CELLE-CI (qui est toujours
externe). Donc si on visualise juste le fichier DVI, on voit
l'emplacement de l'image mais pas cette dernière (en particulier
la commande \psdraft génère un cadre genre \fbox{...} aux
dimensions de l'image à l'emplacement de celle-ci).

* Dans un deuxième temps, la commande DVIPS, transformera le
fichier DVI en PostScript et incluera à ce moment les fichiers
images EPS. Donc si on visualise/imprime le fichier PostScript,
on voit le texte et les images.

* Toutefois, certains visualiseurs DVI, par exemple XDVI sous
UNIX pour ne pas le citer, détectent quelque chose du genre :
     PSfile="toto.eps" llx=0 lly=-1 urx=57 ury=29 rwi=4252
dans le fichier DVI, et appellent gentiment GostScript/GhostView 
qui vont préparer une image bitmap que le visualiseur DVI
s'empressera de montrer (quand ça marche : avec psfig.sty pas de
problème, mais avec l'extension graphicx...). Mais d'autres
visualiseurs, comme DVIWIN, ne savent pas que faire d'un fichier
PS.

DVI signifie DeVice Independent : ceci est un format de document
indépendant du système d'impression. Donc non lié à un langage
de description de page utilisé en interne par une quelconque
imprimante (PostScript, PCL, HPGL, Windows GDI, ...). Il suffit
après d'utiliser un convertisseur qui transforme le DVI en
quelque chose que comprend votre imprimante (PostScript, HPGL,
PCL, ...). Cette méthode permet d'imprimer sur n'importe quel
type d'imprimante (il suffit d'avoir le bon convertisseur). Mais
hélas en DVI, il n'a point été prévu d'y mettre une image (le
problème est en effet pas simple). D'où l'utilisation de fichier
EPS.


 Pourquoi LaTeX n'accepte t'il pas tous les formats d'image ?
------------------------------------------

P. Perichon :
Le problème est le suivant : un fichier DVI + image TIFF, GIF,
TGA,... comment faire digérer tout ça par votre imprimante ????
Le fichier DVI peut se faire convertir en un langage
d'impression compréhensible par votre imprimante... mais vos
formats d'images... faudrait-il en plus prévoir un système de
conversion de TIFF, GIF, TGA, ... vers le langage d'impression
de votre imprimante (Alladin avec GhostScript/GhostView propose
quelques filtres, mais bon). A part quelques langages de
description de pages comme PostScript et un ou deux autres qui
pratiquent une compatibilité ascendante quant aux versions du
langage qu'ils utilisent, les autres fabricants d'imprimantes
changent de langage quasiment à chaque modèle ou série
d'imprimante (impossible de tenir à jour autant de
convertisseurs). 

Pour tout cela le langage PostScript (PS et EPS) nous rend
grandement service en nous simplifiant la vie (et pour d'autres
raisons aussi). C'est peut-être un peu plus lourd à manipuler,
mais plus simple pour tout le monde de convertir vos images en
EPS. De plus, si vous ne disposez pas d'imprimante PostScript,
GhostScript/GhostView se chargera d'interprèter le fichier PS
(texte + images), fabriquera une bitmap à la bonne résolution et
pourra l'envoyer à votre imprimante via votre pilote
d'impression. Tout cela automatisé dans une bonne distribution
est transparent à l'utilisateur.

Pour plus de détails, consulter le grfguide disponible sur CTAN
dans /macros/latex/packages/graphics/ et le document epslatex
disponible sur CTAN dans /info/.



 A quoi sert la commande \newenvironment ?
------------------------------------------

Les commandes \newenvironment et \renewenvironment permettent de
définir des actions qui seront appliquées entre les commandes
\begin{mon_environnement} et \end{mon_environnement}.

Exemple :
dans le préambule :
\newenvironment{Relief}[2]
{\mbox{\Large{\uppercase{#1}}}#2}
dans le corps du document:
\begin{Relief}
   {Il}~était une fois~\ldots
\end{Relief}
\begin{Relief}
   {U}n jour peut être~\ldots
\end{Relief}


 Comment récupérer le nom du fichier compilé ?
------------------------------------------

Pour récupérer le nom du fichier compilé et l'afficher lors
d'une compilation, il faut utiliser la commande \jobname.

Exemple :
Le fichier source est \texttt{\jobname.tex}.


================================================================
[30] AJOUTS
================================================================


 Quels sont les ajouts par rapport à la FAQ précédente ?
------------------------------------------

* 15.23 Comment changer l'espace entre les item ?
* 16.9 Qu'est ce que xindy ?
* 21.12 Comment obtenir un underscore ?
* 21.13 Où trouver le symbole radioactif ?
* 21.14 Comment obtenir le logo LaTeX ?
* 21.15 Comment obtenir le logo LateX2e ?
* 21.16 Où trouver des chiffres entourés ?
* 28.1 Que signifient les sigles T1, mf, fd etc. ?
* 28.2 Quels sont les attributs d'une fonte ?
* 28.3 Comment utiliser une fonte ?
* 28.4 Comment changer la forme d'une fonte ?
* 28.5 Comment changer la taille d'une fonte ?
* 28.6 Comment modifier la fonte des numéros de paragraphe ?
* 28.7 Comment modifier la fonte du mode verbatim ?
* 28.8 Comment réaliser des changements de fontes relatifs ?
* 28.9 Comment mettre en évidence une portion de texte ?
* 29.43 A quoi sert la commande \newenvironment ?
* 29.44 Comment récupérer le nom du fichier compilé ?


 Quelles sont les modifications effectuées ?
------------------------------------------

* 5.1 Quel éditeur de texte puis-je utiliser ?
* 6.7 Comment redéfinir les marges d'un document ?
* 6.27 Comment modifier les environnements de liste ?
* 6.51 Comment insérer un espace vertical dans une page ?
* 6.52 Comment insérer un espace horizontal dans un texte ?
* 13.2 Comment obtenir un renvoi à une page ?
* 15.1 Comment construire une bibliographie ?
* 23.10 Comment convertir du Excel en LaTeX ?
* 23.25 Comment convertir du LaTeX en PDF ?
* 25.1 Où trouver un logiciel de dessin ?
* 29.29 A quoi sert la commande \newcommand ?
* 29.35 Comment inclure l'heure dans un document ?


================================================================
[31] REMERCIEMENTS
================================================================

Je remercie vivement les personnes qui ont contribué de près
ou de loin à l'élaboration de cette FAQ. Je remercie également 
les personnes qui me soutiennent et m'encouragent.

================================================================




-- 

mpk.                                                   _  _
                                                      / |/ |
                                               _  _   () () \
                                              /o\/o\\\\\||\\/
                                              \vvvv/////|\\\\
Bad Wally.                                     wwww**** \\\\\\
---------------------------------------------------------------
 Marie-Paule KLUTH    Marie-Paule.Kluth@aar.alcatel-alsthom.fr
...............................................................
 Alcatel Alsthom Recherche       Tel : +33 (0)1 69 63 12 68
 Route de Nozay                  Fax : +33 (0)1 69 63 18 12
 91460 Marcoussis                			FRANCE
---------------------------------------------------------------

-- 
--
http://www.freenet.fr/villemin/